Vous êtes sur la page 1sur 174

PROFESSIONAL RESPONSIBILITY

ncbex.org (practice exam) --> MPRE

flabar.org --> Rules Regulating the Florida Bar --> Chapter 4 Rules of Professional Conduct
(mirror Model Rules, except for advertising and confidentiality); ethics hotline
No Florida rules on final

I. The Legal Profession--Background and Fundamental Issues

A. Development of the Legal Profession

B. Development of Standards of Professional Conduct

The earliest standards were statements of moral principles that had no legal effect. The ABA
adopted the Model Code of Professional Responsibility in 1969.

ABA Model Rules have only the status of proposed law. The ABA must lobby state and federal
courts to enact these Rules for them to become positive law. None of the ABA documents is
legally binding on anyone. They are models that must be adopted or rejected by individual state
supreme courts before they have an legal effect.

Four sources of authority and advice important to analysis of legal ethics issues:
1) Cases
2) ABA and state and local bar associations' ethics opinions (advisory opinions that respond to a
specific question or an assumed state of facts)
3) ALI's Restatement Third, The Law Governing Lawyers
4) Federal agencies' regulations that regulate the work of lawyers who appear before them

ABA Canons 1908 --> ABA Model Code of PR--> ABA Model Rules (Pre-2002) --> * ABA Model
Rules (2002-2003) *

C. Some Contributions from Moral Philosophy to the Study of Legal Ethics

1. the ethics of duty versus the ethics of aspiration

Morality of aspiration is the morality of good life, excellence, the fullest realization of human
powers. Morality of duty lays down the basic rules w/o which an ordered society is impossible; it
starts at the bottom of human achievement. It condemns for failing to respect the basic
requirements of social living.

The ABA was designed as an attempt to create propositions based on the morality of duty.
Aspirational rules are rare in the Model Code. Note: just b/c you violate an ethical rule does not
necessarily mean you are an immoral person!

2. Moral people versus moral actions

Professional ethics are not synonymous w/moral conduct.


The source of good decisions in settings in which a lawyer is unlikely to get caught for a bad
decision is usually called a person's "character."
Kohlberg's Six Stages of Moral Development:
1) Adherence to rules imposed b/c of understanding that wrongdoing is associated w/punishment
2) "I will be good to you if you are good to me"
3) Conforming behavior to something actor thinks others will approve of
4) Conforming acts to what is required by the social order; requires respect for authority (in legal
ethics, respect for courts and other legal institutions)
5) Conforming acts to what is required by the social contract; recognizes greater possibilities of
change in the social order and is identified as official morality of democracy
6) Person looks to universal ethical principles; must be self-chosen, but must appeal to logical
comprehensiveness, universality, and consistency

Lawyers must conform their behavior to rules applicable to their conduct.

3. Role ethics versus Common ethical standards

Role-differentiated behavior
The professional has a client whose interests must be represented or looked after by the
professional, which means that the role of the professional is to prefer, in a variety of ways, the
interests of the client over those of individuals generally.

4. Consequential versus deontological standards

Consequentialism asks whether the ethical analysis is based on achieving a good result (e.g.
utilitarianism), and deontology asks whether the analysis focuses on absolute values.

Two categories of utilitarianism: act utilitarianism and rule utilitarianism. Act asks which behavior
will lead to more happiness or well being in a particular situation. Rule takes the view that there
is value in establishing appropriate standards of behavior for particular classes of cases.

Deontological approaches can also be divided into two categories. 1) based on duty, says that
there are particular general principles of moral responsibility that can be derived logically and
applied universally (e.g. do unto others as you would have done to yourself). 2) based on rights;
individuals have certain human rights that lawyers should help preserve and protect.

deontological: having to do with moral obligation

5. The Ethic of Care

This view disagrees with Kohlberg. Stresses enhancing the quality of the relationship b/t the
lawyer and all those affected by a given situation, not on analyzing the inherent propriety of
particular conduct of the lawyer.
Suggests that neutrality and objectivity are not possible for lawyers and clients.

Basic focus of ethic of care is on lawyer's acting as a healer who takes a comprehensive view of
her situation and seeks to make everyone better off.
6. Personal versus Social Ethics

D. The Matter of Professionalism

Risk of labeling the practice of law as a profession is that the label is then used to justify restraints
of trade that would otherwise not be accepted.

Society plays a dual role on lawyers; citizens want lawyers to be understanding and socially
responsible, but when they have a problem, they want a lawyer that will play hardball. Evidence
shows that the primary way some people learn about lawyers is through watching fictionalized
portrayals of lawyers (Law & Order).

Rules of professional conduct attempt to accommodate at least five interests: those of 1) the
lawyers of individuals, 2) lawyers in their relationships w/each other, 3) lawyers' clients, 4) non-
clients w/whom the lawyers deal, and 5) institutions of the legal system through which the lawyers
work.

E. Introductory Problem

Page 26

Lawyer knows client is innocent of murder w/which he is charged, but knows prosecutor has
convincing eyewitness who will testify that saw client commit crime. Lawyer knows can obtain
forged hotel register from city far way that will "prove" that client was in other city at time of event;
lawyer firmly believes will not be caught if engages in this fraud. What do you do if you are the
lawyer?

If I was the lawyer, I would be facing competing duties: first, the duty to zealously represent my
client, and second, my duty to be candid toward the tribunal. I would uphold my duty to the
tribunal first. Even though I know my client is not guilty, I can't procure false evidence to get him
off. I could get disbarred for that. Instead, I would do my best to create the best defense possible
for my client, and work extra hard at impeaching the state's "eyewitness," either intrinsically or
extrinsically. If my client is convicted, I will appeal.

1)

2) How would a utilitarian respond?


a. An act utilitarian might argue to forge the document to achieve the greater good
(i.e. allowing an innocent man to escape incarceration).
b. A rule utilitarian, on the other hand, says that we have established appropriate
standards and they should be followed.

3)

4) What if client's alibi is embarrassing to the client and was told you in confidence so that the
information is legally protected. Do you tell the prosecutor the information if it will exonerate your
client? Should not do anything that effects your client w/o asking first. Perhaps should have
client evaluated for competency if chooses death over telling where she was and risking
embarrassment, b/c not a rational choice.

6) What does the concept of professionalism add to your analysis?


Professionalism is an attempt by bar associations to improving the image of the lawyer
among society.

II. Regulation of the Legal Profession

Entry into the legal profession and the conduct of lawyers once admitted are regulated by the
highest court of each state.

Under rule 8.4, a lawyer should be professionally answerable only for offenses that indicate lack
of those characteristics relevant to law practice. Offenses involving violence, dishonesty, breach
of trust, or serious interference with the administration of justice are in that category.

8.4(c) is very vague, and makes it a violation to engage in conduct involving dishonesty, fraud,
deceit, or misrepresentation.

A. Problem 1

Admission to the Bar


(Page 32)

Issues
Will the bar refuse to admit him?
If they admit him, will they find out later and disbar him? (Remember: there is no statute of
limitations!)
What are the dean's obligations?

A. Character and Fitness for Admission to the Bar


1) The point of the requirement that lawyers have the necessary character and
fitness to practice law is that lawyers owe fiduciary duties to their clients, and often their clients
are depending upon them (their clients usually trust them may even be vulnerable to them and
their knowledge to get them through a serious problem). If a lawyer does not possess the proper
character, they can do serious damage (financially or otherwise) to their clients' lives.

2) Gerald Smith's cheating in law school may demonstrate that he lacks good moral
character, [but I don't think it's conclusive]. Cheating is an act of dishonesty, however, and ideally
we want lawyers to be honest with their clients and with the court.
a) We should be more concerned that Smith tried to cover up the fact that
he cheated than we should be about the mere fact that he cheated. Everyone makes mistakes,
but lying about it is not to learn from it. Bar organizations typically admit people who have made
dishonest mistakes, but are less forgiving with those who later lie about those mistakes when
asked about them. The mentality is, if you're lying to us now, you're going to lie as a lawyer.
b) Smith's drug conviction is not conclusive evidence of a lack of character
and fitness, especially if it was only once. Everyone makes mistakes in college. However, Smith
has to be honest about it to keep within rule 8.2. If Smith had been convicted of a drug-related
felony, still not sure that should prevent him from admission to the bar, as long as he is truthful
(forthcoming) about it. If has repeated convictions, however, may indicate he has a disregard for
the law.
c) (Note: when you violate a rule, it is an automatic violation of 8.4 as well).
Rule 8.4 applies to lawyers, and Smith is not technically a lawyer, so perhaps he did not
technically violate the rules. However, cheating does involve dishonesty, so if Smith were a
lawyer, it appears that on that ground he would have violated rule 8.4. With regard to whether
marijuana possession reflects adversely on a lawyer's honesty, trustworthiness, or fitness as a
lawyer in other respects: it only reflects on their honesty and trustworthiness if they lie about it;
with regard to fitness, it may mean they have some kind of dependency problem, but if does not
adversely affect work product, and does not occur often, may not necessarily mean lawyer is
unfit. If occurs often, may mean lawyer has a disregard for the law (since marijuana possession
is illegal, and any disagreement w/that law is no excuse w/regard to criminal liability).
d) Does not necessarily have to be admitted, but if is forthcoming about it
on application and does not appear to have a habit of getting arrested for possession (i.e. has not
shown blatant disregard for legal obligation), Smith should not have a problem.

3) If a person has repeatedly been dishonest in the handling of money, this may be
enough to defeat a person's bar admission, b/c reflects on person's trustworthiness. Lawyers
must be trustworthy, especially those who will be handling their clients' funds. If person was
dishonest one time in an emergency situation and is forthcoming and honest about it, perhaps
should still be admitted. Mustafa The bar is very hard on people who cannot handle money.

4) There should definitely be a statute of limitations on how long prior incidents can
affect current bar admission, especially taking into account that most people do stupid things
when they are minors and/or in college.
a) It should make a difference that some of Smith's problems occurred when he
was a freshman in college, as long as his behavior since then has been more responsible (i.e. he
seems to have been "rehabilitated"). Might have a problem here since he's in his last semester of
law school and decided to cheat on an exam and then lie about it.

5) Perhaps there is some risk today of misuse of the requirement that people
disclose as a matter of character and fitness whether they have ever been members of groups
whose purpose is to overthrow the U.S. government. Mere membership does not necessarily
mean active participation; on the other hand, we do have a lot of terrorists running around and we
do not want any of them every coming into any kind of power.
b) I would never admit an admitted white supremacist to the bar. That
would allow him to one day pursue a position as a judge, and if he were to gain that position,
justice could seriously be interfered with for many defendants who aren't "white."

B. Candor in the Bar Application Process


1) Bar applicants must be completely honest in the application process. It's not
what you have done, it's whether you tell the truth about what you have done. Rule 8.1: an
applicant shall not knowingly make a false statement of material fact, or fail to disclose a fact
necessary to correct a misapprehension. However, rule does not require disclosure of
information otherwise protected by rule 1.6. (Smith cheating on his test is not covered by 1.6).
Bar application does not ask whether an applicant has ever cheated, and the dean decided not to
press charges against Smith, so he will probably not be caught if he does not disclose it on his
application. He would only really be required to disclose any kind of procedure (i.e. honor court)
against him if the Dean had decided to press charges.
a) An applicant is more likely to be denied admission for covering up his
past than for what that past contains.
b) This approach by the bar examiners (denial of admission for lack of
candor) does seem reasonable. Similar to impeachment; if applicant lies on his application, he
will probably lie during his career as an attorney.
e) Reason the person who cheated and admitted it, got an F in the course,
and then was candid about it on his bar application was admitted, while person who got off for
shoplifting by denying he intended to steal, and then admitted that he had intended to steal on his
application was not admitted was that the latter person tried to cover it up at first.
4) Rule 8.1, comment 1: The duty imposed by this rule extends to lawyers.
Comment 3: A lawyer representing an applicant for admission to the bar, or representing a
lawyer who is the subject of a disciplinary inquiry or proceeding, is governed by the rules
applicable to the client-lawyer relationship including 1.6 and in some cases 3.3 (duty to disclose
to tribunal; cannot allow your client to commit perjury). *Lying to court lose 1.6 protection
a) Even if Smith talks to you as his friend who is a lawyer, bottom line is,
you are a lawyer, and therefore attorney-client privilege attaches, and lawyer is bound by
confidentiality under 1.6.
5) Obligation of law schools and their deans in cooperating with investigations
conducted by admission officials:
a) Under rule 8.3, the dean only has to report offenses of another lawyer
and must himself be a lawyer to be within the rule. Here, Smith is not a lawyer. With regard to
whether the law school has a duty to report to the bar, remember that the school has a strong
interest in taking care of these matters on their own (e.g. through its honor court). However,
under 8.1, a lawyer in connection with an applicant's bar application must not fail to disclose a
fact necessary to correct a misapprehension known by the lawyer to have arisen in the matter.
Thus, we would need to know how much the Dean knew about Smith cheating (and whether the
dean is himself a lawyer) before we know if he violated 8.1.

Chemical dependency problems: New for the ABA

C. Educational and Knowledge Standards for Admission to the Bar


5) Whether the taking of particular law school courses should be a prerequisite to
becoming a lawyer:
c) Do not think that lawyers who have not taken PR in law school are any
less apt or ethical. Should not be a bar to admission.
6) Requirement of taking and passing the bar:
c) Should there be a limit on how many times an applicant may take the
bar? Might indicate low level of competency; might not. Does not seem to be a high correlation
between a person's ethics and whether they can pass the bar or the MPRE on the first, second,
or third try.

B. Problem 2- (Rules, 1.1, 1.3, 8.3, 8.4, 8.5, 2.15 code of Jud. Conduct)

Lawyer Discipline and the Disabled Lawyer


(Page 46)

Once a lawyer is a member of the bar, the court retains jurisdiction to sanction that lawyer for
violations of the court's rules of professional conduct.

Issues: one lawyer is alcoholic and is ineffectively assisting his clients, while the other takes on
too many cases (more than he can handle) and may also be ineffectively assisting his clients.

A. Conduct that can Subject a Lawyer to Professional Discipline


1. In Florida, a lawyer looks to the Rules Regulating the Florida Bar, Rules of
Professional Conduct, to see what can justify professional discipline.
a. Under the ABA, professional discipline is imposed for violating rule 8.4,
and if a lawyer violates any of the ABA model rules, it is an automatic violation of 8.4(a).
b. 8.4(b)-(f) are the catch all provisions; include committing criminal acts,
engaging in any conduct involving dishonesty, etc.
2. Discipline process has three functions: (1) to identify and remove from the
profession all seriously deviant members (the "cleansing" function); (2) to deter normative
deviance and maximize compliance with norms among attorneys (the deterrence function); and
(3) to maintain a level of response to deviance sufficient to forestall public dissatisfaction (the
public image function).
c. The effectiveness of deterrence might depend on the likelihood of getting
caught, b/c if lawyers did not believe that they could be caught (i.e. if the bar did not make
examples out of some lawyers), then they might be inclined to believe that the bar rules are not
enforced, and will not be deterred by fear of punishment from breaking breaking them.

3. Does using the discipline process to pursue Andrews and Black seem desirable?
a. Andrews and Black are not necessarily "bad" people. Andrews is in a
difficult position: not wanting to turn away any clients, on the one hand, and not being able to
devote the requisite amount of time to each client b/c he takes on too many, on the other hand.
b. Rule 1.1: Competence. A lawyer shall provide competent
representation to a client. Competent representation requires the legal knowledge, skill,
thoroughness and preparation reasonably necessary for the representation. Rule 1.3: Diligence.
A lawyer shall act with reasonable diligence and promptness in representing a client. Note also
that if a lawyer violates these rules, he/she automatically violates 8.4, but note also that 1.1. and
1.3 are the least enforced rules.
Andrews has violated the competence rule b/c under 1.1 he must be adequately
prepared to handle his clients' legal needs. By biting off more than he can chew, he is unable to
adequately prepare himself for what is required of him for each client's case. With regard to
diligence, Comment 2 to rule 1.3 expressly states that a lawyer's work load must be controlled so
that each matter can be handled competently. Thus, it appears that Andrews has violated 1.3.
Also, b/c of Andrews' workload, it is doubtful that he is able to dedicate himself to the interests of
his clients with the zeal required in Comment 1 of 1.3. However, these rules are the least
enforced. Here, if Andrews' clients happen to be satisfied with his work, the bar is never going to
discipline him for violating these rules. They will only come after him if one of his clients
complains to the bar.
c. We do not have evidence that Andrews has neglected his clients; only
his thoughts that he takes on too much, along w/Black's spiteful opinion that Andrews is always
juggling too much and asking for continuances. Not sure that Opinion 1273 has necessarily been
overruled by rule 1.3, but rule 1.3 is inconsistent w/Opinion 1273 b/c it seems to hold that one
instance of an act or omission (such as overlooking a statute of limitations) is enough to violate
the rule, and seems to punish for negligence instead of requiring a "conscious disregard."
Opinion 1273 is a much higher standard than 1.3

4. Criminal acts that should be held to reflect adversely on the lawyer's honesty,
trustworthiness, or fitness as a lawyer in other respects include theft, bribery, embezzlement,
lying, and may also include drug and alcohol abuse (b/c if a lawyer is under the influence of mind-
altering substances they likely are not "fit" to advocate for the needs/rights of their clients).

5. State supreme courts might sanction lawyer misconduct even when the lawyer
was not acting as a lawyer at the time b/c
a. I think that whether "killers" should be allowed to practice law depends
on the circumstances surrounding the killing. If a person kills her husband b/c he abused her,
then no, I don't think that allowing that person to practice law would undermine public confidence
in the bar. With regard to whether the bar's public image is a sufficient concern to keep an
otherwise qualified person from practicing, I think that perhaps it is. The reason we have rules of
professional conduct at all is in large part to improve the public's image of lawyers. Violent
crimes reflect adversely on our profession.
b. Yes, buying the child pornography reflects on the lawyer's
trustworthiness because it shows his willingness to break the law. This could be indicative of his
willingness to break the law in other areas.
d. Telling gross lies on your resume or stealing another student's academic
identity is certainly grounds for a lawyer losing his license. It's completely dishonest, fraudulent, a
blatant violation of rule 8.4, and shows that the lawyer is willing to lie and alter documents.
e. Yes, sanctions for saying you attended a CLE seminar when in fact, you
showed up near the end shows lawyers that the bar has zero tolerance for any kind of lie
whatsoever. With regard to whether the lawyer actually has to listen to the CLE speaker at the
seminar, I think it would be hard to prove that the lawyer did or did not, short of testing him.

6. Conduct sufficient to violate the rule 8.4(d) prohibition of conduct that is


prejudicial to the administration of justice includes bribery of judges, congressmen, perpetrating
fraud on the court (e.g. plagiarizing work submitted and then refusing to tell court where
information came from).

B. Aggravating and Mitigating Factors in Discipline Cases; Problem of Alcohol and Drug
Abuse

1. Drug and alcohol abuse problems among lawyers:


b. Perhaps Black should not be disciplined so much as he should have to
undergo mandatory rehabilitation. That way, it's punishment in the sense that it is involuntary, but
it is beneficial to him as well and recognizes that his conduct may the result of a disease over
which he may lack control.

2. Yes, Black's alcoholism should be a factor that affects the nature of his discipline.
On the other hand, if we do not punish Black or punish him less b/c he is an alcoholic, this may
reflect poorly on the profession when the damage to the client is the same as to a client whose
lawyer simply did not have time to show up. This is a strong argument for punishing Black and
Andrews to the same degree.
c. ABA has a rule that provides for placing lawyers on an indefinite period
of disability inactive status in case of their mental or physical incapacity. Proceedings are to be
conducted in the manner of a discipline case, but are to be confidential. See rule 8.3.(c): the rule
requiring lawyers to report professional misconduct of other lawyers does not require disclosure
of information gained by a lawyer or judge while participating in an approved lawyers assistance
program.

3. Yes, other kinds of mental disease should affect a lawyer's level of discipline; e.g.
if a lawyer is addicted to gambling and misuses client or firm funds.

C. CHOICE OF LAW 8.5: Interstate Discipline: Jurisdictions that May Sanction and the Law They
Apply- rule 5.5 (can work in a different state) prohov vj??

1. Under rule 8.5, a lawyer admitted to practice in a certain jurisdiction is subject to


the disciplinary authority of that jurisdiction, regardless of where the lawyer's conduct occurs.
Even a state to which a lawyer travels for temporary practice may try the lawyer for conduct that
occurs there; therefore, a lawyer may be subject to the disciplinary authority of two jurisdictions.
Does not matter where you commit the violation, or whether you are in the jurisdiction in which
you are licensed! This is reciprocal jurisdiction.

2. Each jurisdiction applies its own standards to evaluate the conduct if a lawyer is
licensed in one state and conduct occurs in another, unless standards of another state with
regard to the lawyer's conduct there relieve the lawyer of the risk of being disciplined. Same
disciplinary measures don’t have to be impose.
a. Under rule 8.5(b)(2), the lawyer must follow the New Jersey rules
governing disclosure, b/c the predominant effect of the conduct is in NJ. He will not be subject to
discipline under 8.5(b) if his conduct conforms to the rules of NJ, and he reasonably believes the
predominant effect of his conduct will occur there. Thus, he should disclose, per NJ law, that his
client lied about certain important facts in the commercial transaction. Possible to violate your
state's rule and not violate another state's rule, but cannot be subject to discipline under 8.5(b)(2)
if lawyer's conduct conforms to rules of a jurisdiction in which lawyer reasonably believes
predominant effect of lawyer's conduct will occur. ("Predominant effect" means the place in
which your transaction has meaning. E.g. if we negotiate the contract in FL, but it is for a sale in
NJ, the predominant effect is the sale, which will occur in NJ).

3. The effect of lawyer discipline in one state should have the affect on a lawyer's
status in other states where the lawyer is admitted to practice law of
a. The two jurisdictions should seek to apply the same rule to the lawyer's
conduct, and in all cases should avoid proceedings against a lawyer based on two inconsistent
rules. Thus, the sanction imposed by the first state to try the lawyer should be binding on the
other states, b/c both jurisdictions should agree that the same rule applies, and there should be
no proceedings at all if the two jurisdictions have inconsistent rules w/each other.
c. Under rule 8.5(b)(2), such a case would be determined under the rules of
the jurisdiction the conduct occurred in.

Step 1- is the matter pending before a tribunal


Step2- Where did the conduct occur? Where is the impact most raised? (predominate effect
rule)
Safe harbor – possible out for the lawyer here

D. The Duty to Report Another Lawyer's Misconduct: The Discipline Process Itself

1. Under rule 8.3, comment 3, a lawyer is not obliged to report every violation of the
Rules, and the comment acknowledges that such a requirement would be unenforceable. Rule
8.3. limits the reporting obligation to those offenses that raise substantial questions as to the
other lawyer's fitness (substantial = seriousness of offense, not the quantum of evidence of which
the lawyer is aware). Does rule 8.3 mandate that Andrews and Black report each other? Yes. If
you do not report, or make a deal with another lawyer w/regard to reporting them, you then
become the regulator of the conduct instead of the bar being the regulator. On the other hand,
however, this is also one of the least enforced rules.

Knowledge/knows= actual knowledge, can’t turn your head


Substantial= material matter of clear and weighty (even if you don’t have a quantum of evidence)

2. The lawyer must make a report against a fellow lawyer as soon as he/she can be
relatively sure that the lawyer committed some kind of violation of the rules.
a. If there is a civil or criminal action pending involving the same conduct,
disciplinary authorities often prefer that the lawyer wait until that action is completed.
c. Rule 8.3 requires simply that the lawyer have some evidence of a
substantial offense, and expressly states that "substantial" does not refer to the amount of
evidence of which the lawyer is aware.

3. Yes; Andrews cannot avoid reporting Black simply because his client does not
want him to, unless the information the client told Andrews is privileged.
a. Remember that 1.6 is much broader than the attorney-client privilege.
See comment 3: the confidentiality rule applies not only in matters communicated in confidence
by the client but also to all information relating to the representation, whatever its source.
Although the fact that Andrews won b/c of Black's alcoholism certainly relates to Andrews'
representation of his client, it does not appear from rule 1.6 that it was intended to protect
information such as that the client won b/c the lawyer for the other side is an alcoholic. In this
case, perhaps the concept of confidentiality should be read narrowly for this purpose.

4. A judge having knowledge, or a substantial likelihood should take appropriate action


Judge has to turn in a judge and a lawyer should turn in a judge
Pg. 574  2.15(d)

A judge, under Canon 3D(2) of the code of judicial conduct, must report attorney misconduct of
which he has knowledge. Witnessing Black's impaired state or Andrews' inability to keep up with
his caseload would constitute knowledge w/in the meaning of the canon.

5.
b. A state's disciplinary counsel, operating under the authority of the
jurisdiction's highest court and to which complaint about a lawyer can be made, is required to
evaluate all complaints filed by clients or others about a lawyer. Note that only five percent of
complaints have any merit at all, and only half of those result in any sanctions. (In Florida, the
bar makes it very easy for clients to complain about their lawyer).

6.
d. A lawyer should be permitted to resign for any reason he chooses, as long as he
wraps up any pending legal matters for his clients first. If he has been charged with a violation of
the professional conduct rules, then perhaps resigning can be like pleading no contest in criminal
law, and he will simply have to do something to, in some way, make the victim whole. In making
these decisions, it is certainly helpful to know how easy it is for an attorney who has resigned to
apply for readmission to the bar.
e. Sanctions on the entire law firm seems unfair, b/c while it would certainly be a better
incentive for firms to police their own policies and the practices of their individual members, when
you get down to it, each lawyer is responsible for his/her individual actions, and there is only so
much the firm can truly control w/regard to those actions.

7. Some say lawyer discipline should not be considered the equivalent of a criminal sanction and
lawyers should therefore not be entitled to the constitutional guarantees inherent in a criminal
process.
d. A lawyer can be disciplined based on testimony given under a grant of immunity from
criminal prosecution.
e. A presidential pardon might not even be enough to eliminate a lawyer's exposure to
professional discipline.

C. Problem 3

Regulating Lawyers Outside of the Formal Disciplinary System


Page 63

Actions for professional malpractice may seek damages against a lawyer for wrongs
characterized as 1) a tort committed by the lawyer against the client, 2) a breach of the contract
the client made for the lawyer's services, or 3) a breach of fiduciary duties that the lawyer owes to
the client.

Issues
1. Should Field be liable for not interviewing important witness who could have verified that client
had valid claim for higher settlement?
2. Should Field be liable for not referring the tax case to an expert in tax law who could have
ensured that the trusts would not be subject to taxes?
3. Should Field be liable for her criminal client's sentence to imprisonment?

A. The Standard of Care and Conduct in a Malpractice Case


1. Standards upon which lawyers should be judged in malpractice cases:
a. Essential elements of a cause of action for professional negligence are:
1) the employment of the attorney or other basis for imposing a duty;
2) the failure of the attorney to exercise ordinary skill and knowledge; and
3) that such negligence was the proximate cause of damage to the plaintiff.
b. Restatement 3d, Law Governing Lawyers, § 52(1): a lawyer who owes a
duty of care must exercise the competence and diligence normally exercised by lawyers in similar
circumstances.
2. Field committed malpractice by recommending the inadequate settlement to the
victim of medical malpractice b/c she made the recommendation without exercising the diligence
that was necessary for her to discover that the victim was entitled to much more damages. That
is a violation of rule 1.1.
a. Clients tend to follow their lawyers' advice and do not have independent
bases on which to evaluate a settlement; therefore recommending an inadequate settlement can
sometimes constitute malpractice. (have to prove that had she done what she was supposed to
do, the client would have gotten more money/ the client must be aware of this at the time)
- we know the lawyer fell below the ordinary skill and knowledge
Proximate cause and damage
d. There is no malpractice liability for an honest exercise of professional
judgment such as whether to call a particular person as a witness. However, that rule does not
protect a lawyer's decision not to interview a potentially material witness.
3. Field should have been obliged to refer the tax case to a specialist under rule
1.1, comment 1 (expertise in a particular field of law may be required in some circumstances).
Also see comment 2: A lawyer can provide adequate representation in a wholly novel field
through necessary study. If she was going to take on the tax case, she should have brought
herself up to speed on that area of the law.
4. Field should not be liable to the criminal defendant whose defense she handled
badly:
a. The majority rule is that one of the elements of a suit for lawyer
malpractice is that a convicted criminal defendant must prove himself actually innocent of the
charges against him.
e. Reason for treating criminal and civil clients differently w/regard to
attorney malpractice is that it might be harder to find lawyers willing to act as criminal defense
counsel if the law permitted more suits against criminal defense attorneys by their clients who
have been convicted.
5. Liability for professional malpractice to persons other than clients:
a. Restatement 3d, Law Governing Lawyers, § 51: Duty of Care to Certain
Nonclients. Liability may be found in four situations:
b. 1) to a prospective client for revealing confidential information
communicated to the lawyer, or if the lawyer fails to tell the prospective client that the statute of
limitations on his claim will soon run out;
c. 2) to beneficiaries named in a client's will if, due to lawyer's negligence,
will does not carry out testator's intention;
d. 3) to a non-client to whom the lawyer expressly assumes an obligation to
investigate facts and accurately report them to the non-client (example: insurance company); and
e. 4) a lwyer who aides a trustee-like fiduciary to breach an obligation to the
intended beneficiary of the fiduciary's duty may be liable to that beneficiary.

B. Proving a Malpractice Case


1. How trier of fact should know what skill and knowledge a lawyer would ordinarily
employ in these circumstances:
a. Plaintiff must ordinarily present expert testimony a/b the duty of care in a
suit for professional malpractice in both jury and bench trials b/c must be on the record and not
left to subjective standard of the judge.
b. Expert testimony not necessary if issue simple enough or lack of skill
obvious enough as to be w/in range of ordinary experience of lay people.
2. Whether state's rules of professional conduct be determinative of the standards
of care and conduct:
b. Rules have disclaimer (page 9, paragraph 20) that says the rules should
not be treated as the standard of care for lawyers, and that they are not designed to be a basis
for civil liability. However, most courts allow a violation of the rules in as some evidence of a
lawyer's professional duties, and the disclaimer does acknowledge that a lawyer's violation may
be evidence of a breach of the applicable standard of conduct.
d. Restatement 3d, § 52 (proof of a violation of a rule or statute regulating
conduct of lawyers does not give rise to implied cause of action for professional negligence or
breach of fiduciary duty, but may be considered as an aid in understanding and applying
applicable standard of care or conduct).
f. The rules of professional conduct should not be treated as per se rules
with regard to negligence or malpractice. When we say they should be considered by the trier of
fact, we mean that they are one more piece of information to consider with regard to whether the
preponderance of the evidence standard has been met.
3. With regard to causation, client should not be able to recover damages if the
client would have lost the case no matter what the lawyer had done. If client would have lost
anyway, then the lawyer was not even the but-for cause.
a. suit-within-a-suit: traditional requirement of proof of what a non-negligent
lawyer would have obtained for the client if that lawyer had represented the client instead
c. Courts generally do not allow attorney to claim client was contributorily
negligent when attorney's negligence harms plaintiff (e.g. not reading document drafted by lawyer
before signing b/c lawyer represents to you what is in the document). Sophisticated clients with
experience in the type of transaction are treated differently, however, and may be held
contributorily negligent.

C. Malpractice Remedies; Advance Waivers of a Lawyer's Malpractice


1. Remedies that should be available to redress Field's professional malpractice:
a. Range of possible remedies is substantial depending on injury.
b. Lawyers can be subject to punitive damages for malpractice.
2. Sometimes in cases of breach of fiduciary duty, the court will excuse the client
from paying all or part of the lawyer's fee even if the client can show no actual damages (remedy
called forfeiture of fees).
3. Adding clause to standard retainer agreement pursuant to which the client
waives any malpractice claims against attorney for purpose of reducing malpractice liability:
a. Under rule 1.8(h)(1), a lawyer is prohibited from doing this unless the
client is represented separately at the time he/she makes the agreement. It would be very rare,
however, for any attorney to advise a client to waive their malpractice rights.
b. On the one hand, client may be "assuming the risk" b/c the attorney is
being honest and forthcoming about her inexperience in that area of law, but on the other hand,
the client's interests are still at risk, and a lawyer has a fiduciary duty to protect his/her client, not
endanger the client. Under rule 1.1, lawyer has a duty to either bring him/herself up to speed on
that area of the law, or else refer the client to someone with expertise in that area.
d. Under Restatement 3d, § 54, it is ok for a lawyer to require arbitration of
all claims against the lawyer as long as client receives proper notice of scope and effect of the
agreement and if the law of the jurisdiction allow such agreements to be enforceable. It is also ok
under rule 1.8, comment 14, as long as you advise the client exactly of what arbitration is and
what its effects are.- cheaper and quicker…less time
4. Whether a lawyer has the duty to tell a client about the lawyer's own malpractice
in the client's case:
a. Most courts require lawyer to give client notice in writing that client must
obtain independent representation with regard to whether they should/will sue lawyer for
malpractice. Under rule 1.8(h)(2), a lawyer cannot settle a claim for liability w/an unrepresented
or former client unless that person is advised in writing of the desirability of seeking and is given
reasonable opportunity to seek advice of independent legal counsel in connection w/that lawyer's
liability to the client. – give them the amount of time to find a lawyer and if they don’t find one then
you become their lawyer again
d.
5. Whether law firms should be permitted to shield each of the lawyers from
vicarious liability for malpractice of other lawyers in the firm:
a. Sure, if they are formed as a LLC or LLP.
b. Note that rule 1.8(h)(1) does not address vicarious liability, and no other
model rule requires a lawyer to be liable for his or her partner's malpractice.

D. Other Significant Consequences of Negligence or Misconduct by Lawyers


1. Although Field cannot successfully be sued for malpractice, for missing the first date in
the criminal case Field might expect to be held in contempt by the court, and/or fined.
2. Whether the criminal defendant should be able to cite Field's poor representation as a
basis of reversing the conviction:
a. Rule on ineffective assistance of council from Strickland (U.S. Supreme Court):
to justify an issuance of a write of habeas corpus, court must find that lawyer's acts or omissions
were not only outside the wide range of professionally competent assistance, but also that the
ineffectiveness caused actual prejudice.
c. It is ineffective assistance of council to fail to tell a criminal client about a plea
offer from the prosecutor and to fail to give client the right to decide whether to accept it.
3. Lawyers can be criminally charged as accomplices of their clients:
B. Representing client with zeal required by 1.3 is not an excuse for breaking the
law to help your client.
d. While an attorney must do the best job he can for his client, he cannot engage
in dishonest or fraudulent conduct.
4. Lawyers can be sanctioned by the federal Fair Debt Collection Act for engaging in
abusive and unfair practices for sending threatening letters/messages to a debtor of the lawyer's
client or failure to pay money owed to the lawyer's client.
5. Other kinds of remedies and sanctions for improper conduct:
a. Disqualification of the lawyer from participation in the case
b. Court may void transactions made in violation of the lawyer's
professional obligations
III. Fundamentals of the Lawyer-Client Relationship

The lawyer-client relationship is based on law. Some elements of the relationship are subject of a
contract b/t lawyer and client (i.e. the retainer) like any other service contract, and other
obligations are inherent in the status of a lawyer as a fiduciary and are not entirely subject to
amendment by lawyer and client.

Separate the issues into those regarding informed consent of the client and those which require a
more absolute and general prohibition.

A. Problem 4

Undertaking to Represent a Client


Page 84

Once representation begins, a lawyer assumes duties not easily shed.

Issues
Whether your client's interest in you not showing any "mercy" to the client's broker is an actual
duty that you must uphold?
Whether it was ethical for you to refuse to bring suit against the client's broker in any way but
your own way, with the threat of the statute of limitations hanging over the client's head to force
him into a decision?
Whether it was ethical for you to accept the client's broker's offer of settlement without first
discussing it with your client?

A. The Lawyer's Duties to a Prospective Client

- Variation one question 4 Cannell tells you(i) the name of the broker and (ii) his interest in suing.
You do a conflicts check and learn that one of your partners represents the broker and inform
Cannell you can't proceed.
- Is Cannell a "prospective client"? Yes
- Can your partner represent the broker in a subsequent lawsuit by Cannell to recover his
investment losses? Yes (1.18 Paragraph allows Because there is not significant harmful
information)
- can you assist your partner in this lawsuit? Yes (1.18 paragraph c
There is not significantly harmful information exchanged between
The client and the lawyer so there is no prohibition)
- what does this suggest about how a lawyer should proceed (should
Initially inquire the last amount of information, to do a conflicts
Check before you go forward)
- Variation two: Before you have a conflicts check, Cannell blurts out that he (i) knew what the
broker was doing, (ii) could have stopped him but (iii) wanted to get rich. You do the conflicts
check, learn that your partner represents the broker and inform Cannell you can't proceed.
- Can your partner represent the broker in a subsequent lawsuit by
Cannell to recover his investment losses?
- what must happen?
- No Rule 1.18 (d)(2)(i) you must be screened
(must prevent the lawyer who spoke with cannell
From having access to the brokers file, and the
Lawyers in the firm and the original lawyer who
Spoke with Cannell from speaking about the case)

- either _provide notice to Cannell that a screen has


been established or not share any income from the fee
income generated by representation of the broker.
- Can you assist your partner in this lawsuit?
- What must happen?
- only yes: Consent Mr. Cannell must give his
Consent
- Variation 3: Cannell blurts out the same things but this time you ask detailed follow-up questions
and elicit detailed answers about each of the admissions. You do the conflicts check, learn that
your partner represents the broker and tell Cannell you can't proceed.
• Can your partner represent the broker in a subsequent lawsuit by Cannell to recover his
investment losses?
• What must happen?
• Only: No since the lawyer didnt stop, and asked detailed questions and illicited
harmful information; under rule 1.18(d)(2) the alwyer did not take reasonable
measures to avoid exposure to more significantly harmful information, than was
reasonably necessary to see if he could take the case. Since the lawyer illicited
more harmful information he prevents himself from representing the broker, but no
one in the firm can either.
• Only under 1.18(d)(1) with consent of Cannell which is unlikely
• Can you assist your partner in this lawsuit?
• What must happen?
• Only under 1.18(d)(1) with the consent of Cannell which is highly unlikely can
someone represent the Broker.
Variation 4: Cannell blurts out the same things and, again, you ask detailed follow up questions
and elicit detailed ansers about each of the admissions. You do the conflicts check, learn that
your partner represents the broker and tell Cannell you can't proceed.
• Can your partner represent the broker in a subsequent lawsuit by Cannell to recover for his
injuries in an automobile accident with the broker?
• Yes: 1.18(c) allows becasue the information has to be material to the lawsuit at hand
and if nothing which was learned earlier affects the new case then it can proceed
• Can you assist your partner in this lawsuit?
• Yes: the two matters are not substantially related (rule 1.18(c).
• Why the different between variation 4 and variation 3?
• The matter in variation 4 is not substantially related, and in variation 3 we were
talking about the same matter, and for the purposes of rule 1.18 analysis that
makes all the difference.

Until Model Rule 1.18 was adopted, The Law Governing Lawyers, has been clear on the
following: (page 86)
1. Communications from a prospective client are legally privileged and protected by the
lawyer’s duty of confidentiality as if they were communications from an actual client;
2. If the lawyer takes possession of documents or other property of a prospective client, the
lawyer must protect those items as if they were documents or property of a client; and
3. If the lawyer gives advice to a prospective client, the lawyer may be responsible to the
prospective client for malpractice if that advice is wrong.

Rule 1.8
- (h)(1) – more stringent, needs individual representation
- (h)(2) – settlement of

RULE 1.18: duty of


- CONFIDENTIALITY
- SAFEGUARDING
- PROVIDING GOOD ADVICE

1. Legally, Mr. Cannell was not a stranger before you agreed to represent him:
a. Restatement 3d, § 15: 1) Communications from a prospective client are legally
privileged and protected by the lawyer's duty of confidentiality as if they were communications
from an actual client; 2) if the lawyer takes possession of documents or other property of a
prospective client, the lawyer must protect those items as if they were documents or property of a
client; and 3) if the lawyer gives advice to a prospective client with regard to the validity of a
prospective client's claim, that lawyer may be responsible for malpractice if that advice is wrong.
b. Rule 1.18: a person who discusses w/a lawyer the possibility of forming a client-lawyer
relationship w/respect to a matter is a prospective comment. Under comment 9, a lawyer is
subject to the competency requirements of rule 1.1 with regard to advice on the merits of the
prospective client's claim.

2. Under 1.18(c), a lawyer cannot represent a client with interests materially adverse to those of a
prospective client in the same/substantially related matter if the lawyer received information from
the prospective client that could be harmful to that person in the matter. Under rule 1.7, the firm
would not be able to represent Mr. Cannell at all, b/c he is only a prospective client and his
interests conflict with an actual client's.
a. Under rules 1.9 and 1.10, the fact that a lawyer has learned confidential information
from an actual client typically means that neither the lawyer nor anyone else in the lawyer's firm
may oppose that client in the same or a substantially related matter.
d. Under rule 1.18(d)(2)(i), the lawyer must be timely screened from participation in the
case of the actual client whose interests are adverse to the prospective client the lawyer
interviewed. "Screened" means isolation of a lawyer from any participation in a matter through
the timely imposition of procedures within a firm that are reasonably adequate under the
circumstances to protect information that the isolated lawyer is obliged to protect.
e. Under comment 8 to rule 1.18, notice that the lawyer has been screened must be
given to the prospective client as soon after the need for screening becomes apparent as is
practicable.

3. If in legal services setting and met Cannell, who did not want to hire, just wanted basic advice
on how to file the claim himself, Mr. Cannell is an actual client, because under comment 1 to rule
6.5, a lawyer-client relationship has formed.
a. You are permitted under rule 6.5, comment 1 to provide advice to Cannell w/o usual
conflicts review, b/c it is not feasible for a lawyer to systematically screen for conflicts of interest
in that setting.
b. If you found out later that your firm represents the broker, your firm is not disqualified
from continuing that representation b/c you had no way of knowing when you gave Cannell the
advice that the person Cannell wants to sue had retained your firm as his defense council.
c. The lawyer's knowledge of a conflict becomes disqualifying when the lawyer
undertakes to represent the prospective client he met at the legal aid place.
4. Whether online chat rooms or the internet are "safe" places to try to get clients:
a. Using a web site to invite communications with the firm does not constitute an
invitation to burden the firm with confidential information from a prospective client. However, the
firm must treat the information from the prospective client as confidential and not use it to
advantage the firm's client who may be in dispute with the prospective client.
b. Chat room conversations between lawyers and prospective clients are ok as long as
the clients know from the beginning that the lawyer is only giving legal information, and not legal
advice.

B. The Decision to Represent a Client


1. Whether you had an obligation to accept Cannell as your client:
a. A lawyer is not obligated to act as advocate for every person who wishes to
become his client, but he should not lightly decline proffered employment. (Model Code of PR)
6.2
b. No such rule in the Model Rules; closest is w/regard to court appointments to
represent indigent clients.
-take him on as your client- yes case is good
-no he is angry,
2. Note rule 1.2(b) states that a lawyer's representation of a client, including
representation by appointment, does not constitute an endorsement of the client's political,
economic, social or moral views or activities.

3. Whether there should be legal limits on the gourds a lawyer may rely upon to reject a case:
a. A lawyer's absolute right to decline a case for any or no reason ended with the
Massachusetts Commission Against Discrimination's decision Stropnicky in 1997 (firm would
represent only women; no men).

4. Whether you should give weight to your client's anger toward his adversary in deciding
whether to take a case: under rule 3.1, the lawyer must consider whether the action is
meritorious, or whether it is frivolous and merely aimed at revenge against the adversary (i.e. the
broker, in this case). 1.16 allows a lawyer to withdrawl from the case if he is opposed to
representing the client on basis of disagreement,

C. Documenting the Decision to Undertake a Representation


1. According to Restatement 3d, § 14, a prospective client becomes an actual client
when: a person manifests to a lawyer the person's intent that the lawyer provide legal services for
that person and either: a) the lawyer manifests to the person consent to do so; or b) the lawyer
fails to manifest lack of consent, and the lawyer knows or reasonably should know that the person
reasonably relies on the lawyer to provide the services.

2. Lawyers bear the risk of ambiguity about whether the lawyer-client relation has been
formed.

3. Consent required for limits on the scope of representation:


a. A lawyer who fails to get such consent may later be accused of malpractice.
When a lawyer takes on a case, the client may not know the range of remedies that is possible,
and it is the lawyer's duty to bring these actions as well or advise the client that he may get other
counsel to do so.
b. Under rule 3.1, a lawyer cannot file a claim on behalf of a client that is frivolous
and not in good faith. Lawyers must inform themselves of the facts of their clients' cases and the
applicable law and determine whether they can made good faith arguments in support of their
clients' positions. A claim is not frivolous even though the lawyer believes that the client's
position ultimately will not prevail. Comment 2. Under rule 1.2(c), a lawyer may limit the scope of
representation of a client if the client gives informed consent. Under comment 7, the limitation
must be reasonable under the circumstances, and is a factor to be considered when determining
whether the lawyer has the legal knowledge, skill, thoroughness, and preparation to be
considered competent under 1.1. to take the case.

4. Matters to address in any engagement letter with client:


a. Engagement letter is prepared by lawyer and counter-signed by client,
containing:
1) who is or is not the client;
2) the fee for representation and outline of expenses for which client will be
responsible (when bills will be rendered, when they must be paid, whether interest will be
charged);
3) scope of representation (what the lawyer agrees to do or not do) under rules
1.2(c) and 1.5(b)
4) conflicts of interest lawyer may have and time w/in which client must give
informed consent if willing to waive conflicts;
5) departures from usual assumptions a/b handling confidential information,
under rule 1.6; and
6) undertakings the client will be asked to make in connection w/the
representation (such as candor).
b. If lawyer decides not represent a client, should send client a letter to that
effect so can document that fact and avoid claims later by client that client reasonably believed
lawyer was working on client's case.

D. Decision-making During Representation--Issues the Lawyer is To Decide and Issues


Reserved to the Client
1. If you want to make the argument that your client has a lack of sophistication
when it comes to these kind of things (in this case, stocks), do you have to get his permission
first? Under rule 1.2(a), you are impliedly authorized to make this argument, and it does not
seem to fall w/in the confidential information relating to the representation that rule 1.6 seeks to
protect. However, the argument may embarrass the client, so you should get your client's
consent (sit down with him and explain to him that the argument is part of your strategy).
2. Under rule 1.2, comment 2, normally, legal, technical, and tactical matters are left
to the lawyer to determine. If the client disagrees, and tells you he does not want you to make
the argument that he is unsophisticated in stock transactions, you cannot make the argument.
You can w/draw if you have a fundamental disagreement (or the client may fire you). However,
most likely the best way to handle it is to put in writing that you wanted to make the argument and
that the client forbade it, that you believe the case is weaker without that argument, and then
proceed the best you can w/o it.
Under rule 1.4, which is the communication rule, a lawyer must, under (a)(2),
reasonably consult w/the client about the means by which the client's objectives are to be
accomplished. Comment 2 makes clear that in some situations you might have to consult with
your client prior to taking action, and that at others, the exigency of the situation may require the
lawyer to act w/o prior consultation. (Think of 1.2 and 1.4 as working together).
3. Client has the authority to decide whether or not to settle a civil case under rule
1.2(a). Comment [blank] states that if the client gives you permission to settle for a certain
amount, you may do so if offered a settlement that meets that criteria without consulting the client
again. Under Restatement 3d, § 30(3), lawyer may have liability to third persons for unauthorized
conduct on behalf of a client.
4. Under rule 1.2(a), a client has the exclusive authority to decide what plea to enter
in a criminal case. Remember that there is a fine line between advising and deciding. A lawyer
cannot make decisions for his or her clients. In a lawyer's role as an advisor, a lawyer can
certainly give the client his/her opinion about what to do, but the client must make the ultimate
decision. Thus, it is ok for a client to choose the death penalty over life in prison and the lawyer
will be absolved of liability if the client later changes his mind, especially if the lawyer advised
against the death penalty to begin with. Just make sure that you put your advice in writing and
that the client signs that they are making their decision against your advice.
5. Who controls the litigation:
c. Restatement 3d, § 23 identifies only two matters that are beyond the
reach of client control: (1) the decision of the lawyer to refuse to act in a way that the lawyer
reasonably believes to be unlawful, and (2) the lawyer's decision to take actions that the lawyer
reasonably believes to be required by law or an order of a tribunal.

Bottom line: A client's authority to direct a lawyer depends in part on restrictions the law imposes
on the client's conduct. If the lawyer knowingly accepts a direction the client does not have the
legal authority to give, the lawyer may be liable to the party to whom the client owed duties.

B. Problem 5

Billing for Legal Services


Page 99

Issues
When must a fee agreement be reached?
What limit does the law impose on the size of a lawyer's fee? (Some states, like FL, limit the fee
a lawyer can collect in contingency fee cases)
Does the client have the option to pay by the hour instead of on a contingent fee basis?

MUST BE REASONABLE!

Duties of Attorney
1. confidentiality
2. duty as an advisor
3. competence
4. diligent
5. communicate
6. reasonable fee
7. advocate’s duty
8. duty of loyalty/conflicts

A. The Fee Agreement Between Lawyer and Client


1. Whether a lawyer's fee must be in writing:
a. Under rule 1.5(c), contingent fee agreements must be in writing and signed by
the client. Rule 1.5(b) only states that it is preferable to have any other kind of retainer
agreement in writing. Thus, it would not be a violation of 1.5 to create an hourly billing agreement
that was not reduced to writing.
c. Novak would not be wise to rely on an oral argument to be paid an hourly fee,
because it would be hard to prove the terms of the agreement if a dispute arose.
2. Under rule 1.5(a), the expenses charged by a lawyer must be made part of the fee
agreement. A lawyer shall not charge or collect an unreasonable fee or an unreasonable amount
for expenses.
b. Under rule 1.5, comment 1, the amount that the lawyer charges the client has
to be the amount actually incurred by the lawyer, and that amount has to be reasonable. With
regard to whether the expense/charge is reasonable, 1.5(a)(1) through (8) are factors to consider.
Most importantly, the client has to agree to pay expenses in advance. Thus, a $1 per page of
photocopying is not reasonable when it costs $0.10 at the grocery store. It is likewise
unreasonable for the lawyer to charge over $1000 for a first class airline ticket when the coach
ticket probably costs less than $250.
d. ABA Formal Opinion: absent a separate agreement, if the form treats work by
a contract attorney as an expense item (over and above its fee), the firm must bill only what it
actually paid for the contract lawyer's services. If the contract lawyer is billed as just another
lawyer whose work makes up the fee for the matter, the firm may bill any reasonable rate for the
services just as it does for one of its associates.

3. Whether a lawyer may charge a non-refundable retainer:


c. When the fee agreement is proposed by the client, non-refundable retainers
are ok, especially where the client is sophisticated and the law firm has performed properly. A
client can terminate the relationship at any time, he just might have to forfeit the retainer fee. A
lawyer may be able to justify the non-refundable retainer under 1.5(a), or if the lawyer is able to
show that by taking this client's case, the lawyer forwent making money from another client and
should therefore be able to keep the retainer as an opportunity cost. (Non-refundable retainers
are often charged in the area of family law to deter litigants from lawyer-shopping). Some courts
say that a lawyer is entitled to a fee only after doing the work to earn it, and that "engagement
retainers" maybe justified if the lawyer turns down other work or otherwise benefits the client.

1.5(a)(2): consider precluding other employment


cmt 4: may require advance payment but must return unearned portion
In FL you can, usually in family matters: if its reasonable and it must be in writing
The money should be put into his account if it’s a true refundable retainer.
To determine if a fee is reasonable look at the factors in 1.5.

4. Whether a lawyer may increase her fees during the course of representation:
a. It depends on the understanding of a reasonable client under the
circumstances. Some courts say it's ok, as long as the lawyer gives the client notice.
b. A lawyer and client may revise their agreement as to scope of work and fees
after the representation has begun, but at that point the lawyer assumes a greater burden of
showing the agreement to be fair and reasonable. Restatement 3d, § 18(1)(a).

B. The Requirement That a Lawyer Charge Only a "Reasonable" Fee


1. Novak's proposed fee of 44% seems very unreasonable, considering that by his own
admission most lawyers charge only 33%. (Remember that true contingency fees are only for the
fee, but the client still has to pay for all of the expenses). On the other hand, if the lawyer really is
better than most, perhaps he will be able to get the client a larger recovery and then his 44%
contingent fee may not be so exorbitant.
Make sure that you keep track of the amount you work on a contingency fee
case, because if the client fires you, you may be able to collect from the client on a quantum
meruit theory.
2. One reason for limiting the freedom of contract between a lawyer and client is that the
lawyer is to avoid a negative image of lawyers in the public eye. Plus, a lawyer is supposed to
act as the client's fiduciary, and limitations on the freedom to contract with the client helps to
ensure that the lawyer will not charge the client for unreasonable expenses (such as the fact that
the lawyer does not have an assistant and must do all the clerical work on the case himself).
3. The Supreme Court has held that it is a violation of antitrust law for a local bar
association to adopt a schedule of fees that lawyers would be required to charge for particular
services. A purely advisory fee schedule issued to provide guidelines would be ok.
b. The Supreme Court has also held that it is illegal to agree to maximum fees.
5. Whether the lawyer may properly consider the client's ability to pay in setting a
"reasonable" fee:
b. Rule 1.5, comment 5: "it is proper to define the extent of services in light of
client's ability to pay." This comment should be read in light of rule 1.1. (diligence).
On the flip side, the lawyer could take the case for nothing (pro bono) b/c the
client is indigent. It is not proper to raise your rates just b/c the client is wealthy, but it seems to
be ok to charge a poor client less than your normal fee. The lawyer is still subject to 1.1 and 1.3
(competency and diligence), however.

6. Reasonableness is accessed at the time the K is made, sometimes if the fee ends up
being excessive, the judge will sometimes reduce it
7. Trial judges are not permitted to raise sua sponte whether an excessive fee is being
charged by counsel.

C. Special Rules Applicable to Contingent Fees


1.c. When the ABA changed rule 1.5, comment 5, it seemed to say that a lawyer does not
have to offer the client an alternative between a contingent fee and a non-contingent fee. A
lawyer who insisted on contingent fees, even if they are not in their client's best interest, may be
breaching his or her fiduciary obligations to the client, because he is putting his own financial
well-being before the client's. On the other hand, the client does have a choice, and can always
leave and find a lawyer who charges an hourly fee.
2. Whether a lawyer may charge a contingent fee based on a settlement award if the
case is settled before the lawyer even begins work: fiduciary problem b/c lawyer is putting his
own financial interests before the client's, b/c he knows that the case is worth more than the
settlement offer. However, if the lawyer fully discloses to the client that the case may be worth
more, but that it may take longer to get more and the client then agrees to settle, that would be
ok. The lawyer must, however, always put the client's needs before his own needs. Rule 1.2(a)
the lawyer would have to abide by the clients decision on settlement offers.
a. Some argue that collecting contingent fees in settlement cases is unethical, as
the lawyer is unlikely to have done more than 15 hours of work, and that with trial awards, there is
little, if any relationship between the efforts of the lawyer and the size of the verdict. The size of
the verdict is determined by the nature and extent of the plaintiff's injury and resulting damages,
not by the brilliance of the lawyer. The rationale that a lawyer should be entitled to collect his/her
third in all cases that win to make up for the ones that lose is also unethical, b/c one client cannot
be properly surcharged to compensate for deficiencies in another client's case.
3. Cases in which a contingent fee should not be proper:
a. Rule 1.5(d)(1): A lawyer shall not enter into an arrangement for, charge, or
collect any fee in a domestic relations matter, the payment or amount of which is contingent upon
the securing of a divorce or upon the amount of alimony or support, or property settlement in lieu
thereof. This is because of the major policy concerns involved (see comment 6) (e.g.
encouraging couples to reconcile). Underlying public policy is to protect and promote marriage,
and if lawyers take contingency fees in domestic relations cases they will be concerned with
getting their fees (i.e. with getting the divorce decree), not with protecting the clients' best
interests, which may include reconciling the marriage.
b. Rule 1.5(d)(2): A lawyer shall not enter into an arrangement for, charge, or
collect a contingent fee for representing a defendant in a criminal case. Contingency fees in
criminal cases might deter attorneys from taking many cases.
4. Prohibition of contingent fees in other kinds of cases:
D. The Alternative of the Hourly-Rate Fee
1. Hourly billing rates:
a. Still subject to some abuses, such as filing as many motions as possible, doing
more research, going to more hearings, etc. than would be done in a contingent fee case.
b. You are only supposed to bill what you actually do, so if something takes you
half an hour, you should only bill half an hour, even if what you did took you five hours the very
first time you did it. Also, you cannot bill two clients for the same hour (i.e. one hour is one hour,
not two!).
2. Collateral effects on the lawyers of requiring them to bill a certain amount of hours?

Florida Bar Rules


Contingency fee agreements are governed by 4-1.5. Gives a statement of client rights.

Slide show

RULE 1.5
Attorney’s fees and expenses must be reasonable

1.5(a)(1)-(8) factors are not exclusive factors, but are a guideline


Lawyers may be reimburse for copies etc but it must be reasonable

Rule 1.5(b)
Scope of representation and basis of rate shall be communicated to the client.
Preferably in writing, before or with a reasonable time after representation.
· For cont. fees, go to 1.5© must be in writing and other requirements
· Desirable to furnish client with memorandum explaining fees (cmt 2)
· If a regularly represented client, ordinarily fees are understood if under same
rate and basis (cmt 2)

1.5 (c) Contingency Fees


Lawyer receieves a fee only if matter is resolved in client’s favor. Most often
expressed as a percentage of the recovery.
· Must be reasonable
· Must be in writing, signed by a client
· Fully explained, including expenses
· Upon conclusion of matter, lawyer must provide client w. written statement
stating outcome, and remittance if recovery and method of determination.
1.5 (d)
Contingent fees are prohibited in:
· Domestic relations matters, contingent on securing divorce or amount of
alimony and support or property settlement
· Criminal cases, when defending a criminal defendant
1.5(e)
A division of a fee between lawyers who are not in the same firm may be made:
• Encourages lawyers to refer clients to more competent lawyers
• Tax lawyer, with client who suffers personal injury, can refer client to
plaintiff's lawyer and still receive "a piece of the action"
• Encourages lawyers to associate with more experiences lawyers
• Two scenarios Envisioned in Rule 1.5(e)
• Fee split in relation to work performed by each lawyer
• Proportionate to services rendered
• Lawyer A= 50% of work = 50% of fee
• Lawyer B - 50% of work = 50% of fee
• Fee split not in relation to work performed by each lawyer
• Disproportionate to services rendered
• Lawyer A = 100% or work = 75% of fee
• Lawyer B =0% or work = 25% of fee
• ("pure referral fees are permissable")
• Scenario one: Proportinate Division
• Lawyers agree to divide fee in proportion to services they will render
• Lawyer A = pleadings, discovery, motions =40%
• Lawyer B= Mediation, trial prep, trial= 60%
• Client agrees to arrangement 1.5(e)(2)
• Participation of each lawyer
• Share that will be received by each lawyer
• Client's agreement must be "confirmed in writing"
• Document signed by client OR
• Document prepared by lawyer and forwarded to client
memorializing client's prior oral agreement RPC 1.0(b)
• Total fee is reasonable
• Scenario two: Disproportionate Division
• Lawyers agree to divide fee not in proportion to services rendered
• Lawyer A= 100% of the work =75% of the fee
• Lawyer B= 0% of the work =25% of the fee
• Each lawyer assumes"joint responsibility" for representation
• Meaning=joint liability for any malpractice and ethical violations
• Lawyers treated as if they were partners with duty to oversee
one another ad obligations to accept responsibility if something
goes wrong
• Better ensures that referring lawyer will make a good referral
• Client agrees to the arrangement
• Participation of each lawyer
• Share that each lawyer will receive
• Client's agreement "confirmed by writing"
• Document signed by client OR
• Document prepared by lawyer and forwarded to client
memorializing clients prior oral consent
• Total fee is reasonable
• Reminds working lawyer not to increase contingency fee to offset
amount paid to non-working lawyer

Types of fee arrangements
· Hourly-may only bill for actual hours
o ABA Formal Opinion 93-379 (pg. 114)
· Fixed Fees
o 1.5(b) says it is preferred to have hourly rates memorialized in writing but
is not required
o for an hourly rate Rule 1.5(a)(7) is most helpful in figuring out what is
appropriate and reasonable.
o a person can increase the hourly rate during the course of representation
but rule 1.5(b) says there must be written notice to a client but it is not
expressly prohibited.
· Contingent fees
o Fee earned only when a particular outcome/recovery is achieved
normally it is a percentage.
o If no recovery is garnered then the attorney receives no compensation.
o must be memorialized in writing
· Refundable v. non-refundable retainers
o Lawyers may require advance payment of a fee, but must return any
unearned portion (cmt 4)
·

Trust accounts 1.15 (a)


1.15(d)
Safekeeping Property- disputed amounts
· Upon receiving funds that client or third party has an interest, lawyer must
promptly notify the client or third person and deliver the funds and provide a
full accounting 1.15(d)
· If there is a disputed portion must be kept in trust account. Undisputed
portion must be promptly distributed (1.15(e) and cmt 3&4)

****Distribution of disputed amounts


· 100,000 recovery for pun. Damages
· Fee K- Lawyers get 40%
· Client wants to pay lawyer 25%
· In dispute 40-25= 15%= 15,000
· Not disputed= 85%
· 25% to lawyer (client agrees)= 25,000
· 85-25=60%= 60,000 to client
· 25,000 (to lawyer)+ 60,000 (to client)+ 15,000 (to trust account)= 100,000

C. Problem 6

Handling Client Property and Withdrawing from Representation


Page 117

Issues
What is the lawyer's duty to protect and account for a client's property?
How should lawyers deal with clients who fail to pay the lawyer's fee?
What possible rights does a lawyer have to a lien on client property and papers?

A. Handling Client Property


1. Why lawyers must pay attention to their handling of client property:
a. A lawyer has a fiduciary relationship with a client. Thus, the trustworthy
management of a client's property is one of the most fundamental duties of a lawyer.
b. Violation of fiduciary standards w/respect to client property is a definite way for
a lawyer to be disbarred, in part b/c it constitutes the serious offense of stealing from a client. In
another part, it is b/c proof of a violation of accounting rules is typically much more straightforward
than proof of failing to be forthcoming in disclosure of information.
2. Whether depositing a client's settlement check into the lawyer's trust account is a
violation of Rule 1.15: Yes, under 1.15(e), even where property in which both the lawyer and the
client have an interest is in dispute, the lawyer must promptly distribute all portions of the property
as to which the interests are not in dispute; thus, Jackson must at least give her client the ring,
b/c Jackson has no interest in the ring. She only has an interest in the punitive damages award.
OK to deposit in client's trust account.
a. A lawyer may sign the lawyer's own name on a settlement check made out to
the lawyer, but the lawyer may sign a client's name on a settlement check only if she has the
actual authority to do so.
c. W/o such authority, signing the client's name on a check may also constitute
the crime of conversion.
ci.when Jackson takes the check to the bank attorney should deposit the check
in the Client’s trust account. then the lawyer should dispurse the entitled
amount to the client and the attorney’s fees to herself.
cii. one cannot take money out of an account (i.e., and advance) even if a
check has yet to clear. attorney can only withdrawal money when a check
clears and there are actual funds in the account. There are two violations of
rule 1.15(a) a comingling of accounts first when you take money out of the
account, and then again when you return money to the account.
4. Whether Jackson was required to give the client the full amount that the client thought
was due:
a. Rule 1.15(d) states that except as permitted by agreement with the client, a
lawyer must promptly deliver to the client any funds or other property that the client is entitled to
receive, and upon the client's request shall render a full accounting regarding such property.
$15,000 should stay in the trust account, b/c the client only wants to give the attorney $25K, and
the attorney believes she should get $40,000 under the agreement (and the difference is
$15,000). Thus, the $60,000 that is not in dispute should be distributed to the client, $25,000
should go to Jackson, and only the $15,000 should stay in the trust account. Thus, Jackson is
violating 1.15(d) by keeping the entire $100,000 in the trust account. The client is entitled to the
$60,000 now, and the attorney is entitled to at least $25,000 now; the attorney must disburse
what is not in dispute! Plus, keeping the entire $100,000 in the trust account is actually a
commingling of attorney and client money, which is against the rules! [Think F. Lee Bailey]
1.15(a)
b. Under 1.15(a), a lawyer shall hold property of clients that is in a lawyer's
possession in connection w/a representation separate from the lawyer's own property. So
Jackson would have to keep her portion of the award in a different account. Note, however, that
Jackson does not need to create a million different accounts for all her clients; she can keep all
client money in one account as long as she records each sum as belonging to a certain client. By
wearing the ring jackson violates 1.15(d) and (a) Under 1.15(b), a lawyer can only deposit the
lawyer's own funds in a client trust account to pay bank service charges on that account and only
in the amount necessary for that purpose.
c. Under 1.15(e), the amount of the fee that remains in dispute must be kept
separate by the lawyer. (See (a) above).
d. 1.15(e). A lawyer who pays settlement proceeds over to the client in knowing
disregard of a creditor’s lie may become directly liable to the lien holder & basically required to
pay twice.
5. Not proper for Jackson to wear the client's ring while it was in her possession.
However, if client had given the ring to Jackson to safeguard it, Restatement third section 44
comment(e) says Jackson would have to take reasonable measures for safe keeping of objects
such as keeping the ring in a safe deposit box. She is not allowed to wear the ring!
a. See Restatement 3d, § 44, comment e.
b. Must protect client property while in lawyer's possession.
6. The bar will subject a lawyer to discipline for the dishonesty of the lawyer's office staff
or associated lawyers under rule 5.3.
7. Traditionally there can be no audit unless there is probable cause to believe the
accounts have not been properly maintained. In 1992 ABA delegates voted and approved
proposals calling for random audits however states have yet to adopt these rules.
C. Where the audit determines a lawyer has stolen from his/her own law firm and
not a client, disbarment of the lawyer may follow.

B. Withdrawal When the Client Becomes Too Difficult


1. When a lawyer should be required to terminate a representation:
a. Where the lawyer's physical or mental condition materially impairs the lawyer's
ability to represent the client, rule 1.16(a)(2) makes withdrawal mandatory. [Also, a lawyer must
withdraw if (1) continuing to represent the client would violate another model rule, or if (3) the
lawyer is fired. (duh)]. In this case, Jackson cannot withdraw from representing the difficult client
under (a), and can only withdraw under (b) if she complies with (b)(1) (no material effect on the
interests of the client) or (b)(7) ("other good cause for w/drawal exists"). The later in the
representation that the lawyer w/draws, the more likely the client will be adversely affected. Note
(b)(4) (repugnant to the lawyer, or lawyer has fundamental disagreement) is a high standard. It is
not meant to excuse lawyers who simply do not like their clients.
b. Withdrawal is also required by rule 1.15(a)(1) where continued representation
will violate another rule. (E.g. conflicts of interest, or situations involving uncorrected client fraud).
2. Ability of lawyer to terminate representation over client's objection:
a. Rule 1.16(b)(1) authorizes a lawyer to w/draw from representation at any time
for any reason if the w/drawal affects no material interests of the client adversely.
b. Under rule 1.16(b)(3), a client may discharge the lawyer at any time and for
any reason, as long as the client pays for work the lawyer has done to that time.
3. Whether lawyer should be permitted to w/draw under 1.16(b)(4) whenever client insists
on taking action w/which lawyer has fundamental disagreement: yes a lawyer may withdrawal if
there is fundamental disagreement however that standard is a rather high standard, and cannot
be used merely to escape a high maintenance client, or a client who may be difficult to work with.
a. The client under 1.2(a) has the authority to define the objectives of
representation, and the lawyer may only resign if the high standard is met, and a client may also
resign at any point therefore the relationship is mutual in respect to withdrawal.
b. Imprudent conduct is a much lower standard than conduct to which a lawyer is
in fundamental disagreement with.
4. Whether lawyer should be permitted wo w/draw from a case that has become
financially unprofitable for lawyer to pursue:
a. Under rule 1.16(b)(6), if representation results in unreasonable financial
burden, lawyer can w/draw.
b. In 1.16(b)(1) a lawyer may withdrawal for any cause as long as it doesnt
adversely affect a client; but 1.16(b)(2)-(6) requires good cause from the
lawyer but it doesnt matter if it adversely affects the client.
c. 1.16(b)(5) allows a lawyer to withdrawal if the client fails substanitally to fulfill
an obligation to the lawyer regarding the lawyer’s services after giving the
client notice and provide a client an opportunity to cure the defect.
d. 1.16(c) tells lawyers sometimes they may have good reason for withdrawal but
must still obtain permission from the trial judge and if the trial judge says no
then you must continue to represent the client.
e. there is nothing in the RPC which prohibits a lawyer from suing a former client
for unpaid fees, but most dont do it because a client is likely to countersue for
legal malpractice.
ONLY HAVE TO REVEAL INFORMATION ABOUT CLIENT THAT IS NECESSARY
5. What role courts should play in reviewing attempted withdrawal.
a. If the matter for which the lawyer is trying to withdrawal from is already pending before a
tribunal the rules of that tribunal will ordinarily require the tribunals permission to withdrawal under
rule 1.16(c).
a.
C. Limitations on a Lawyer's Efforts to Collect a Fee
1. Whether a lawyer should be permitted to sue a client to collect a fee:
b. ABA's formal opinion 250 upheld suits against clients, but without enthusiasm.
Restatement 3d, § 42(1) allows suits by lawyers against clients to collect legal fees under a
breach of contract theory. Lawyer can also put in fee agreement that the client will be subject to
arbitration for fee disputes.
2. Whether a lawyer should be bound by and obligation to preserve client confidence and
secrets in establishing the elements of the lawyer's claim: yes most of the time, however rule
1.6(b)(5) allows information which is reasonably necessary to defend or represent the lawyer in a
dispute to be revealed about the client which might otherwise be prohibited from being revealed.
3. Cannot threaten to reveal more client confidences than necessary to force clients to
pay outstanding fees. See rule 1.6 comment 4.
4. Whether a fee dispute between client and lawyer is subject to mandatory arbitration:
Some states do require that but not all.
5. Whether a lawyers engagement letter can specify non-fee complaints must be
submitted to arbitration as well: rule 1.8(h) forbids limiting the lawyer's liability for malpractice or
settling a claim for malpractice without advising his client for the need of independent counsel.
(California would allow it, ABA formal opinion said an arbitration clause does not violate 1.8(h) but
the attorney would still under 1.4(b) need to advise his/her client of the benefits, and adverse
consequences that may arise from execution of such agreement.
6. In absence of agreement, lawyer has the right to collect from client under quantum
meruit when the client terminates the relationship under circumstances where significant time has
been invested, but no results yet achieved. In the case of a contingent fee, the first lawyer can
usually recover only if the second lawyer wins the case, (i.e., only if plaintiff ultimately wins the
case)
7. You can according to the ABA (ABA Formal opinion 338) charge interest and accept
credit cards (and let the credit card company deal with not getting paid) or charge interest for late
payments.
(b) court in Illinois said that a law firm charging a client 1% of its fee interest for
late payment plus additional attorneys fees if the fee had to be collected in court was not allowed
as it put the firm in a position of prosecuting its own client, but they could be used to "silence a
client" who wanted to protest an improper fee.
(c) even where original agreement made no provision for interest on past due
fees if a client is late on payment an interest fee may be charged, however under strict scrutiny.

D. Attorneys' Liens and Other Security Interests


1. Retaining lien gives the lawyer a possessory interest in the papers and documents in
the client's position (but does not apply to property left in the attorney's possession for
safekeeping). Rule 1.8(i) says that a lawyer may not acquire a proprietary interest in the cause of
action or subject matter of litigation the lawyer is conducting for a client, except that (a) the lawyer
may acquire a lien authorized by law to secure the lawyer's fee.
2. Charging lien gives the lawyer a right to have the recovery in a case applied to
payment of his or her fees. The attorney must give notice to the person paying the judgement or
settlement. Once notice is given, person paying the judgement or settlement is liable for the
attorney's fee if that person pays the entire judgment or settlement directly to the attorney's client.
[Florida recognizes both types of liens].
Sometimes judges force attorneys to turn over papers if they determine that the client
really needs them and cannot possibly pay the lawyer's fee.
3. A contingency fee is not a security or proprietary interest b/c the ABA gives the client
the right to fire you. If the lawyer had a true proprietary interest, he/she would be more like a co-
party to the lawsuit. The ABA does not like this b/c it fears that the lawyer will sway the litigation
the way he/she wants it b/c he/she has an interest in the outcome.
4. The other rule governing the lawyer taking an interest in the client's property is rule
1.8(a). The difference between the two rules is that if you have a proprietary interest in the
proceeds of the litigation (i.e. the subject matter of the litigation), you are like a co-party, which
the ABA does not want (e.g. do not want you to prevent settlement b/c you are an owner of the
outcome of the lawsuit). The exceptions are liens and true contingent fee agreements. However,
under 1.8(a), it is ok, for example, to get an ownership interest in filing a patent agreement.
5. under rule 1.8(i) only applies in a litigation context and therefore if a person hires
jackson and is short changed then Jackson may accept a 5% profit of a persons net income for
his services of helping prepare articles of incorporation for a company. (1.8(i) only applies to a
litigation lawyer not a transactional one. Rule 1.5 still applies regardless however as the fee must
always be reasonable.
6. Jackson represents the plaintiff in a personal injury action on a 33% contingency fee
basis. After jackson works hard on the case for six months, the client fires her and hires another
lawyer to handle the case through trial. At this point Jackson has no complaint as she took the
case on a contingency fee basis, so the lawyer is not entitled to any attorney fee even under
Quantam Meruit until the

D. Problem 7

The Duty of Confidentiality


Page 133

The lawyer's confidentiality obligation is made up of three different bodies of law:


1) The attorney-client privilege, which comes from the law of evidence.
2) Work product immunity (discovery and civil procedure).
3) The professional duty of confidentiality (obligations of fiduciaries).

A. Information Protected by the Attorney-Client Privilege


1. The attorney-client privilege protects 1) a communication (written or oral), 2) made b/t
privileged persons (i.e. the client/prospective client, the lawyer, and agents of both in facilitating
the communication b/t them), 3) in confidence, 4) for the purposes of obtaining or providing legal
assistance for the client. Purpose of attorney -client privilege is to allow the client to speak freely
to his/her lawyer. (all 4 elements must exist for there to be attorney client privilege) (There can be
other conversations about nay number of topics but as long as it doesnt relate to legal advice
then it is not ocvered by the attorney client privilege)
Note that Rule 1.6 is very broad. Lawyer cannot reveal any information that relates to the
representation (unless the lawyer is impliedly authorized to do so), whatever its source (i.e. the
information does not have to come from the client. See comment 3).
3. When Carter told attorney that he lied to the buyer about the basement flooding, that
conversation was protected by the attorney-client privilege since it is a confidential
communication made b/t lawyer and client for the purposes of obtaining legal assistance. The
fact that the client committed a fraud is not material to the privilege, b/c the fraud was committed
in the past. (Note, however, that if the client had told the lawyer that he was going to commit a
fraud, that conversation would not be privileged). Attorney-client privilege will protect the content
of a conversation b/t attorney and client, but not the fact that attorney and client spoke about a
certain subject. Will not protect notes from an interview in which you discover from someone
other than the client the confidential information. Will not protect unfavorable rumors others told
you about your client. Privilege does not protect underlying facts, so if opposing counsel takes
the client's deposition and asks about such a fact he must answer truthfully at the risk of perjuring
himself.
4. Variation two: The buyer sues carter for fraud. Carter tells you he spoke with a lawyer
before the sale attorney client privilege is there to protect the client, if a client injects a prior
communication the element which no longer exists is the confidentiality on a voluntarily and
volitional basis, so once the client injects a privleged communication to someone new, the client
can no longer rely on the privelege.
5. SUBJECT MATTER WAIVER: is the consequence for making a selective decision to
disclose some conversations, so in the end you may end up waiving attorney client privilege for
other conversations as well. Does not however waive confidentiality on subsequent conversations
relating to different subjects however with a new lawyer.
3. you assert a third party claim for contribution against Carter;s former lawyer: Under
RPC 1.6(b)(5)says when a lawyer is sued by a client a lawyer may disclose information relating to
the client even if it is normally protected by attorney client privilege. But cannot disclose
something unless it is reasonably necessary.
With regard to rule 1.6, any information relating to the representation of a client is
protected unless: the client gives the lawyer informed consent to disclose the information, or
disclosure is impliedly authorized.
1.6 b: An attorney may reveal confidential information, at the lawyer's discretion, if
disclosure is necessary:
1) to prevent death or bodily harm;
2) to prevent client from committing a crime or fraud;
3) to prevent, mitigate, or rectify substantial injury to the financial interests or property of
another of which the client has used the lawyer's services;
4) to secure legal advice about the lawyer's compliance with the Rules;
5) to establish a claim or defense on behalf of the lawyer in a controversy b/t the lawyer
and the client; or
6) to comply with other law or a court order.

Thus, the attorney-client privilege does not protect the conversation the attorney had with
the previous owner, b/c the previous owner is not the client! it is, however, protected by Rule 1.6.
The notes the attorney took w/regard to his interview with the previous owner are protected by
work product immunity; however, since the previous owner died, opposing counsel may be able
to show a substantial need for those notes, at least with regard to the underlying facts (not the
mental impressions of the attorney).

B. Information Protected by Work Product Immunity


1. Work product consists of tangible material or its intangible equivalent in unwritten or
oral form, other than underlying facts, prepared by a lawyer for litigation then in progress or in
reasonable anticipation of future litigation. Opinion work product consists of the opinions or
mental impressions of the lawyer; all other work product is ordinary work product. Except for
material which by applicable law is not so protected, work product is immune from discovery or
other compelled disclosure.
2. R. Civ. P. 26(b)(3): a party seeking discovery may show it has substantial need of the
materials in preparation of the party's case and that the party is unable, w/o undue hardship, to
obtain the substantial equivalent of the material by other means.
3. Opinion work product is never discoverable, unless extraordinary circumstances justify
disclosure.
4.

C. The Lawyer's Profession Obligation of Confidentiality


1. Under rule 1.6(a), the lawyer cannot reveal information relating to the representation of
a client (unless the client gives informed consent). Here, the fact that client may be broke relates
to the representation with regard to the client's ability as a defendant to make a settlement offer.
2. A lawyer may be required to testified about information that is protected only by the
obligation of confidentiality, as opposed to information that is privileged. Under rule 1.6(b)(6) an
attorney may disclose information to comply with other law or a court order. Comment 13 also
states that a lawyer may be ordered to reveal information relating to the representation of a client
by a court.

D. How Legal Protection Against Disclosure Can Be Lost


3. Whether a lawyer can get ethics advice about his representation of a client from
another lawyer in a different firm, and discuss the client's case with that lawyer, w/o violating rule
1.6: yes, under rule 1.6(4) ("to secure legal advice about the lawyer's compliance with the
Rules").
4. How much affirmative effort must a lawyer undertake to preserve the confidentiality of
communications w/clients and others (e.g. with emails): Under comment 17 to rule 1.6, when
transmitting a communication that includes information relating to the representation of a client,
the lawyer must take reasonable precautions to prevent the information from coming into the
hands of unintended recipients. Duty does not, however, require that lawyer use special security
measures if the method of communication affords a reasonable expectation of privacy. (So
regular emails are ok; they do not have to be encypted!)
5. What should happen if a lawyer or client inadvertently reveals confidential client
information (e.g. secretary faxes confidential document to opposing counsel instead of to client):
Three ways that courts can look at this situation. 1) The "never waived" rule, which requires
giving the documents back; 2) the "strict accountability" rule, which treats all such situations as
waivers; and 3) a middle ground that looks at reasonableness of precautions taken, time taken to
discover the error, scope of the production, extent of the disclosure, and the interests of fairness
in the situation.
Rule 4.4(b) puts the obligation on the recipient to promptly notify the sender that
he/she has received documents relating to the representation of the lawyer's client and knows
that they were inadvertently sent to the recipient.
6. Whether attorney-client privilege, work product immunity, or lawyer's duty of
confidentiality (under 1.6) should be lost by the passage of time or by the client's death: No. The
attorney-client privilege continues even after death. The principle behind allowing the privilege to
continue is b/c there may be ramifications post-death, or the information given by the client to the
attorney might not have been accurate.

9/20/10

• Variation three:
• you interview the prior owner who confirms that he told carter the basement
floods after a heavy rain. You take verbatim written notes of what he says and
include a statement regarding your impression of the former owner ... (see slides)
• What is the diference between “fact” work product and “opinion” product?
• Fact: notes, a lawyers summary of testimony, connecting documents
and testimony
• Opinion: the lawyers impressions, conclusions, and legal theories
• Variation 3 Contd.:
• The former owner dies before the buyer’s lawyer interviews him. The buyer’s
lawyer learns that you have notes of an interview and she serves you with a
subpoena duces tecum.
• what is the likelihood that you will be able to avoid producing the notes
altogether?
• not very good, this information is not protected by attorney client
privilege but by the work product doctrine
• can you draw a distinction, for discovery purposes between your notes of
what the former owner said and your impressions of him?
• what is the standard for discovering another party’s “fact” work product?
• substantial need so it would probably be discoverable because the
former owner is now dead, and therefore the lawyer has no way of
interviewing and getting or recreating a similar work product.
• what is the standard for discovering another party’s “opinion” work product?
• opinion work product is almost always insulated and protected under
almost any circumstance. (only discoverable under exceptional or
extrodinary circumstances)
• Variation four: While you are at a party a banker mentions Carter is having
financial problems and is behind on several of his loans. You make a mental note
of this information because it could be helpful in settlement negotiations.
• Does RPC 1.6(a) cover the bankers statement?
• yes
• Does the attorney client privilege cover the bankers statement
• no, it is a communication between lawyer and non client
• would you be required to obtain Carter’s permission prior to disclosing this
information during settlement negotiations.

• how might your answer change if Carter previously told you to “do whatever
it takes to get his case settled”?
• pursuant to 1.6(a) there could be implied consent and a lawyer can
disclose information via informed consent, or implied consent
• you speak with carter and he confirms that he is struggling financially.
• does RPC 1.6(a) cover carters statement?
• yes
• does the attorney client privilege cover carters statement?
• yes
• Does the fact that you learned the same information from the banker at an
earlier affect whether the attorney-client privilege does or does not attach to
this communication
• no it is still privileged
• variation four (contd:
• you are at another party and a friend tells you she heard that Carter is having
financial problems and asks whether the rumor is true
• which of the following would be more appropriate response in this context:
• I can’t disclose information relating to the representation of a client OR
• I cant disclose information covered by the attorney-client privilege (only
applies in a courtroom and here this is a party and there is no courtroom)
• How would your answer change if you were being deposed and asked the
same question about carters financial situation?
• this time the second answer rather than the first would be more
appropriate, so you are in a court setting.
• Extrapolate from these answers and fill in the blanks:
• the attorney client privilege applies only in a litigation setting in which a
lawyer/client has been asked what did client say to you or lawyer say to
you.
• the professional duty of confidentiality applies in a non litigation setting
(all other settings)
• Variation five:
• Carter dies unexpectedly while the lawsuit is pending.
• does carters death have any affect on your professional duty of
confidentiality?
• no, duty of confidentiality survives a clients death 1.6(a) is carried
forward into rules 1.9(c) in regard to former clients.
• does his death have any affect on the application of the attorney-client
privilege?
• no, it continues to block an attorney from sharing information relating to
the client attorney relationship. the policy underlying this is that a client
has the confidence in talking to an attorney candidly and do so without
fear that what they share will not come back to bite them down the road.
• (problem set 2)

E. Problem 8

Confidentiality and the Organization as a Client


Page 141

A. Privilege and Confidentiality Rules When the Client Is an Organization


1. Whether the results of internal investigation will be protected by the attorney-
client privilege or work product immunity: Upjohn is the leading case. Rejects the "control group"
theory, which stated that attorney-client privilege only applied to persons who controlled the
corporation or organization. Therefore, after Upjohn, confidential information given to the
attorneys by regular employees is protected by the attorney-client privilege.
6. Whether the former managers of a corporation can control whether the attorney
turns over a report to new management: basic rule is that neither the privilege nor the duty of
confidentiality prevents successor corporate management from learning the content of
discussions of former management with corporate counsel.

B. The Common Interest Privilege Among Multiple Organizations


1. Ordinarily defendants w/a common interest may hold joint discussions at which
they and the various lawyers talk freely w/each other about matters relevant to their common
interest. Any client who is part of that discussion or similar information exchange may assert the
privilege just as it could if the client had been talking to its lawyer alone.
2. The privilege continues to apply to what the attorney and the client said. If there
are two clients with a common interest, and one wants to waive attorney-client privilege
3. McCormick: information is privileged when the two clients have a common
interest and are on the same side of the litigation. When two or more persons, each having an
interest in some problem or situation, jointly consult an attorney, their confidential
communications w/the attorney, though know to each other, will be privileged in a controversy of
either or both of the clients w/the outside world. If, however, the clients have a "falling out" and
go against each other, the privilege is lost.

C. Confidentiality and Risks to Public Health and Safety


1. Difference b/t the old rule 1.6(b)(1) and new rule 1.6(b)(1) is that the new rule is
broader. It protects anyone, not only the client, who is at risk of certain death or substantial bodily
harm. Remember, rule 1.6(b)(1) is not a mandatory rule; it is discretionary. So if the lawyer does
not turn over the memo to the EPA, nothing will happen to him as far as disciplinary action for
violating the rules. If the lawyer was going to turn over the memo, he would have to be
reasonably certain that someone was going to die.
Comment 6 to rule 1.6 states that a lawyer who knows that a client has accidentally
discharged toxic waste into a town's water supply may reveal this information to the authorities if
there is a present and substantial risk that a person who drinks the water will contract a life-
threatening or debilitating disease and the lawyer's disclosure is necessary to eliminate the threat
or reduction of the number of victims. Here, it seems that the 5 per cent increase in the risk of
cancer would result in at least one person dying, and that it would be ethical for the lawyer to turn
the memo over to the EPA and reveal the risk.
Florida's Rule 1.6 (4-1.6): much stricter than the ABA rules with regard to what a lawyer
must disclose.
b) When Lawyer Must Reveal Information. A lawyer shall reveal such information to the extent
the lawyer reasonably believes necessary:
(1) to prevent a client from committing a crime; or
(2) to prevent a death or substantial bodily harm to another.
(c) When Lawyer May Reveal Information. A lawyer may reveal such information to the extent the
lawyer reasonably believes necessary:
(1) to serve the client's interest unless it is information the client specifically requires not to be
disclosed;
(2) to establish a claim or defense on behalf of the lawyer in a controversy between the lawyer
and client;
(3) to establish a defense to a criminal charge or civil claim against the lawyer based upon
conduct in which the client was involved;
(4) to respond to allegations in any proceeding concerning the lawyer's representation of the
client; or
(5) to comply with the Rules of Professional Conduct (allows a lawyer to reveal confidential
information if doing so would keep the lawyer from violating another rule; very broad).
Conflicts of Interest Family
1.18- Prospective Client
1.7 Current Clients
1.9 Former Clients
1.8 Specific Conflicts
1.11 Gov Conflicts
1.12 Judge, Arbitrator
1.10 Imputed Disq

Conflicts
1. I.D Conflicts
2. Does Conflict exist
3. Is it consentable
4. Can obtain informed consent, confirmed in writing
5. Is it imputed to firm
6. Is screening available

IV. Requirement of Loyalty to the Client


• A. Problem 9

Representing Multiple Parties Dealing With Each Other


Page 159

A. Determining Whether a Lawyer Has a Conflict of Interest


1. Rule 1.7: a lawyer may not represent a client if the representation involves a
concurrent conflict of interest (i.e. two current clients who have a conflict). Such a conflict exists if
1) the representation of one client will be directly adverse to another client; OR 2) there is
significant risk that the representation of one or more clients will be materially limited by the
lawyer's responsibilities to another client.
Whether simultaneous representation of both Wilsons involves their attorney in a
conflict of interest: Comment 6 says that . Plus, to secure a divorce, the parties do have to go to
court, and therefore they will be deemed as adverse to each other. Plus, if each was
represented by a separate lawyer, each might be better aware of what each is entitled to (e.g.
perhaps if the wife had her own lawyer, he would push for her to receive more in child support
each month, whereas if they have the same lawyer, the lawyer just wants them to come to an
agreement, even if that agreement might not be the best for one of the parties).
We worry about loyalty, communication, and confidences; we cannot be loyal to
both sides. We must be competent as attorneys and inform each client of what they are entitled
to. The attorney has to be an advisor, which will be a problem for an attorney who represents
both of the Wilsons.
3. Whether the attorney should be able to avoid the conflict problem by
representing Mr. Wilson while having his law partner represent Mrs. Wilson: Rule 1.10 treats the
law firm as one unit; therefore, if one attorney in the firm is representing one side, no other
attorney in the firm may represent the other side.

B. Waiver of a Conflict of Interest; The Requirement of Informed Consent


In most states, joint representation of spouses in a dissolution action is treated as a non-
consentable conflict in all instances. If your state categorizes divorce as an adversarial
proceeding, then joint representation of those clients is non-consentable.
Comment 18 to rule 1.7 states that informed consent requires each affected client be
aware of the relevant circumstances and of the material and reasonably foreseeable ways that
the conflict could have adverse effects on the interests of that client. (Informed consent is also
defined in rule 1.0(e)). The requirement of informed consent prevents many waivers.
3. Whether the model rules should require that any client consent to a conflict be in
writing: rule 1.0(b) defines "confirmed in writing," and states that when used in reference to the
informed consent of a person, denotes informed consent that is given in writing by the person or a
writing that a lawyer promptly transmits to the person to the person confirming an oral informed
consent. Note that there is no requirement that the confirmation be signed.
5. Whether a client should be able to revoke consent and then require that the
lawyer not represent either of the clients: comment 21 to 1.7 states that a client who has given
consent to a conflict may revoke the consent and, like any other client, may terminate the
lawyer's representation at any time. [A client can always fire a lawyer]. The problem arises when
revoking consent may unduly burden the other client that the lawyer was jointly representing. It
depends on the circumstances, including the nature of the conflict, whether the client revoked
consent b/c of a material change
Generally, however, if client 1 revokes consent, the lawyer must cease to
represent both, b/c otherwise that lawyer will have an unfair advantage in representing client 2
(who did not revoke consent) against client 1 and client 1's new lawyer.

C. Conflicts for Which Consent Is Not Effective


5. Whether a lawyer may represent both husband and wife for estate planning
advice: yes. However, there may be a potential conflict under 1.7(a)(2) whereby the attorney
may be materially limited in representing both spouses. Comment 27 to rule 1.7 states that a
conflict may arise in estate planning and estate administration. To comply w/conflict of interests
rules, the lawyer should make clear the lawyer's relationship to the parties involved. Under
comment 31, continued common representation will almost certainly be inadequate if one client
asks the lawyer not to disclose to the other client information relevant to the common
representation. The lawyer should, at the outset of the common representation and as part of the
process of obtaining each client's informed consent, advise each client that information wil be
shared and that the lawyer will have to withdraw if one client decides that some matter material to
the representation should be kept from the other.
Better to not represent a client than to represent them and then withdraw.
Under comment 22, a lawyer can require waiver of future conflicts if the waiver is
very specific. It cannot be boilerplate language or leave open-ended the conflicts that the client
has consented to waive, b/c it is unlikely that the client will have understood the material risks
involved in the waiver. In any case, advance consent cannot be effective if the circumstances
that materialize in the future are such as would make the conflict non-consentable under 1.7(b).

• Variation One: Advantages and Disadvantages


• the Wilsons want a divorce. They agree between themselves on custody and support
and then ask green top represent them.
• what are the potential advantages to the Wilsons from having one lawyer represent
them both.
• both can get the lawyer they want, if they must have separate lawyers then one
person will not get the lawyer of their choosing.
• if there is a separate lawyer then there may be unnecessary conflict by bringing
in a second lawyer.
• both will save money and costs by only needing one lawyer
• what are the potential disadvantages to the Wilsons from having one lawyer represent
both of them.
• if things don't work out as contemplated, the lawyer must withdrawal from each,
and then they must both hire separate attorney’s
• Attorney client privilege is not in effect, and the attorneys normal duty of
confidentiality is not there, and the Attorney must share all information with each
party.
• difficult if not impossible to be independent professional judgment to both parties
• Green agrees to represent the Wilsons with respect to their divorce.
• what happens if green concludes that Mr. Wilson should pay more for child support
because Mr. Wilson makes substantially more than Mrs. Wilson
• What would Green do if he represented Mr. WIlson Only?
• he would not say anything and remain silent in the hopes that the other side would
not raise the issue.
• What would green do if he represented Mrs. WIlson only?
• he would raise the issue and tell her that she is entitled to more money
• since he represented them both then he can not raise either issue.
• What critical issue have the Wilsons completely missed?
• how the property/assets should be divided.
• What should green do?
• he should go to the Wilsons and tell them to talk between themselves and see
if they can work out a property agreement.
• What should green do if the Wilsons agree as follows
• Mrs. Wilson gets their house worth 1 million dollars in this market.
• Mr. Wilson gets his 401(k) retirement account comprised of securities worth 1
million at todays prices
• this in not an equal trade, Mrs. WIlson can sell the house at any time and
receive the million dollars but Mr. WIlson can only get his 1 million dollar
40-1(k) at the time of his retirement in the future at some point. However
because this is unequal Green cannot point this fact out because he would
jepordize his independent professional representation.
• Will the joint representation of Mr. and Mrs. WIlson involves a “concurrent conflict of
interest” Under RPC 1.7(a)(1) and/or 1.7(a)(2)
• 1.7(a)(1)-will the representation of one client (Mr. WIlson) be directly adverse to
another client (Mrs. Wilson)
• Direct adversity = one client will benefit at the other clients expense
• 1.7(a)(2)-is there a significant risk that the representation of one client (Mr. WIlson) will
be materially limited by the lawyers responsibilities to another client (Mrs. Wilson)
• Material representation- will lawyer be tempted to pull her punches when
representing one lcient to avoid hurting the other client
• Significance of risk = intensity of temptation
• practical question (always ask when deciding a 1.7(a)(2) question - is the other
client the “preferred” client?
• length of relationship with other client
• frequency of representation of other client
• amount of fee income generated by other client
• Can green represent Mr. and Mrs. WIlson notwithstanding the presence of a
concurrent conflict of interest?
• yes 1.7(b) tells lawyers they can represent clients notwithstanding a current conflict of
interest so long as 4 conditions are satisfied
• lawyer reasonably believes he/she will be able to provide competent and diligent
representation to each affected client
• whether the particular representation is not prohibited by law
• the representation does not involve the assertion of a claim by one client against
another client represented by the lawyer in the same litigation or other proceeding
before a tribunal
• each affected client gives informed consent, confirmed in writing
• Can green reasonably believe that he can provide competent and diligent
representation to Mr. and Mrs. Wilson? RPC 1.7(b)(1)
• What is a reasonable belief?
• it is both a subjective and objective belief the lawyer can provide reasonable and
diligent advice to the clients. Rule 1.0(i)
• How do we get this definition?
• reasonably believes- is defined in RPC 1.0(i)
• lawyer believes the matter in question RPC 1.0(a) (subjective belief)
• circumstances are such that the belief is reasonable RPC 1.0(h) (objective
belief)
• Practical definition of reasonable belief for conflicts purposes
• lawyer actually believes he/she can do a good job for each client
• disinterested lawyer, on the outside looking in, would agree with this assessment
• Is the represntation of Mr. and Mrs. Wilson in the divorce proceeding prohibited by law?
• how does comment [16] explain representations that are “prohibited by law”
• in many states it is prohibited by law for a lawyers to represent multiple defendants in a
capital case
• also prevents a former high ranking government official who then leaves and
represents a client in a matter involving an issue to which the official was previously
involved.
• does it matter whether the lawyer believes he can do a good job for each client?
• does it matter...
• Would the representation of Mr. and Mrs. WIlson result in Green representing the
plaintiff and the defendant, at the same time, in the same lawsuit? RPC 1.7(b)(3)
• what is the difference between 1.7(b)(3) and 1.7(a)(1) in the litigation context?
• 1.7(b)(3) covers the situation in which a lawyer wishes to represent a plaintiff and
defendant in the same litigation at the same time (client + matter focused)
• 1.7(a)(1) covers this situation and it also covers the additional situation in which
focuses on clients who interest are directly adverse but it does not matter if it
has to do with the same litigation, it focuses on the clients interest period
without regard to whether it is in the same litigation (client focused)
• Why can’t the same lawyer represent the plaintiff and the defendant at trial?
• does it matter whether the lawyer believes he can do a good job for each client?
• does it matter that the clients are willing to proceed with the same lawyer?
• there is an institutional interest that each sides presentation is as forceful as
possible because that is the premise of our adversarial legal system. 1.7(b)(3)
exists to continue the adversarial practice and allows each side to be as forceful as
possible to ensure a judge/jury makes the correct ruling under the law.
• Has green obtained Mr. Wilson’s and Mrs. Wilson’s informed consent, confirmed in
writing? 1.7(b)(4)
• what is the definition of informed consent?
• 1.0(e) defines this term - lawyer must law out for the client all the material risks in a
course of action as well as the reasonably available alternatives to that course of action
so the client may make an informed choice of whether the lawyers choice is the one in
which she will agree
• the material risks of joint representation are : loyalty, confidentiality, and the
consequence of green having to withdrawal and both having to hire separate and
new representation
• what is a reasonably available alternative to joint representation?
• the Wilson’s could get separate counsel
• potential benefits would be: each is impartial and can exercise independent
professional judgment, and each will gain the confidence of confidentiality.
• What is the definition of “confirmed in writing”?
• can be accomplished in either of 2 ways; (preferred) obtain the clients signature on a
piece of paper; or obtaining oral consent and then memorializing the oral consent on a
piece of paper or writing and then delivered to the client.
• does comment [20] contemplate that the lawyer will speak with the clients in person
about risks and alternative? tells lawyer in affect the lawyer should speak in person to
the client about all material risks and reasonably available alternatives; allows the
lawyer to gage the client and the client understands exactly what is going on before
giving her informed consent, the client can also ask follow up questions to which the
lawyer can answer, and then the consent given will be as informed as possible.
• How might Rule 1.6(a) get in the way of obtaining another client’s informed consent to
waive conflict of interest: May limit the extent to which the layer may speak to the other
client in an attempt to get the consent of the existing client to bring on conflicting new
representation.
• what are the two “consents” that the lawyer must obtain from each client?
• consent to release information about that client to the other
• consent to joint representation
• what happens if the lawyer can’t get a client’s consent to disclose?
• then you are done, each consent is a minimum requirement and in its absence the
other client will never have adequate representation to give informed consent.

B. Problem 10

The Duty of Loyalty


Page 171

A. Taking a Case Against a Current Client


1. Whether a client has a legal right to keep you from representing another client whose
interests are directly adverse to the client's: Rule 1.7 comment 6, absent consent a lawyer may
not act as an advocate in one matter against a person the lawyer represents in some other
matter, even when the matters are wholly unrelated. This is b/c the client to whom the
representation is directly adverse is likely to feel betrayed; breach of duty of loyalty.
The attorney may be able to get around 1.7(a) if he can fit into an exception in 1.7(b).
However, the attorney is never going to fit into the fourth exception, 1.7(b)(4), "each affected
client gives informed consent, confirmed in writing." International Bolts will never consent; they
have already told the lawyer that they do not want their lawyer to represent the company that is
foreclosing on them. The president has even stated that he would be offended if the lawyer
represented the other company.
2. This does not appear to be an issue of confidences. It is an issue of loyalty.
3. Whether there should be a concern that the lawyer might represent one client less
vigorously so as to not offend another. This also raises issues of loyalty. It seems as well that
this raises a conflict of interest under 1.7(a)(2), the materially limited rule.
B. Ascertaining Who Is a Current Client
The rules for former clients are more flexible than those for current clients. Whether a
client is a current client is a case-by-case determination. The pattern of repeated retainers could
be a factor.

D. The Problem of Positional Conflicts


1. The lawyer's neighbor who wants him to sue Second National Bank does not pose a
1.7(a)(1) problem, because the lawyer's client is First National Bank, not Second National Bank.
He may have a 1.7(a)(2) problem because First National Bank (a current client) does not want
him to file the suit b/c it is afraid of the possible ruling that could come out of it. Therefore, the
lawyer may not be as vigorous in representing the neighbor, who wants prepayment penalties to
be declared illegal, because he does not want to make First National angry.
Positional conflicts: A positional conflict occurs when a lawyer takes a position in
litigation on behalf of one client which could, if successful, have an adverse impact on the legal
position of another client. This situation most overtly arises when the same lawyer is arguing
antagonistic positions in litigation simultaneously conducted on behalf of two different clients.
Comment 24 to rule 1.7 states that a conflict of interest exists if there is a significant risk that a
lawyer's action on behalf of one will materially limit the lawyer's effectiveness in representing
another client in a different case; for example, when a decision favoring one client will create a
precedent likely to seriously weaken the position taken on behalf of the other client.
Factors to consider when deciding whether to client needs to be advised of the risk:
where the cases are pending; whether the issue is substantive or procedural, the temporal
relationship b/t the matters; significance of the issue to the immediate and long-term interests of
the clients involved; and the clients' reasonable expectations in retaining a lawyer. If there is
significant risk of material limitation, then absent informed consent of the affected clients, the
lawyer must refuse one of the representations, or w/draw from one or both matters.

C. Problem 11

Conflicts of Interest in Criminal Litigation


Page 189

Facts
Barbara Bentley represents Bitter Creek, a corporate-D charged with price fixing. Chuck Carson
is manager of widget division at Bitter Creek and conspired with Mary, who worked for Bitter
Creek's main competitor Carson asked Bentley to represent him and Mary. U.S. attorney wanting
to run for governor will do anything to obtain conviction so makes deal with Bentley: if Carson
and Mary plead guilty, no jail and he will drop case against Bitter Creek. Bentley tells them to
take the deal. She then writes an article called "There's Nothing I won't do to see that at least
one of my clients goes Free."

A. Limits on One Lawyer's Representing Co-defendants In a Criminal Case


1. Why courts consider it important that criminal defendants be warned about possible
conflicts of interest:
a. Federal rules of criminal procedure requires a judge to "promptly" inquire about joint
representation of two or more clients and must personally advise each defendant of his right to
the effective assistance of counsel, including separate representation. Additionally,
representation of co-defendants presents a rule 1.7(a)(2) conflict, because the lawyer is
materially limited in representing one client for the benefit of another (or in this case, two (Mary
and Carson) for the benefit of one (Bitter Creek). Comment 23 states that such conflicts often
arise in criminal cases, and that the potential for conflicts of interest in representing multiple
defendants in criminal case is so grave that ordinarily a lawyer should decline to represent more
than one defendant. (Might be ok in civil cases, if the lawyer gets informed consent from all
affected parties).
Rule 1.13 governs organizations as clients, and subsection (g) is subject to rule 1.7.
1.13(g) states that a lawyer representing an organization can also represent any of its employees
or shareholders. If consent is required by the organization under 1.7, consent can be given by
the appropriate official.

C. Conflicts of Interest Faced by Prosecutors


1. Prosecutor is highly motivated to get convictions that will tend to further his political
ambitions (he is running for governor). There is a conflict of interest with his duties as a public
prosecutor under rule 3.8 (which governs the special responsibilities of a prosecutor). 3.8(a)
states that a prosecutor in a criminal case shall refrain from prosecuting a charge that the
prosecutor knows is not supported by probable cause. Thus, in this case at least the prosecutor
has probable cause to go after conviction of Carson and Mary. However, he does not care who
takes the fall as long as he gets a conviction. Comment 1 states that a prosecutor has the
responsibility of a minister of justice and not simply that of an advocate. This responsibility
carries with it specific obligations to see that the defendant is accorded procedural justice and
that guilt is decided upon the basis of sufficient evidence. Thus, he may be violating his duties as
a prosecutor.
Under 1.7(a)(2), the prosecutor has a conflict of interest because he is materially limited
from representing his client (in his case, the government; the state) by his personal interest of
becoming governor.

D. Conflicts of Interest of Defense Counsel; Publication Rights and Social Entanglements


1. Whether a defense attorney may personally and financially benefit from the publicity
surrounding a major trial: Rule 1.8(d) states that prior to the conclusion of representation of a
client, a lawyer shall not make or negotiate an agreement giving the lawyer literary or media
rights to a portrayal or account based in substantial part on information relating to the
representation. [This is generally a defense counsel rule, because prosecutors do not technically
have clients]. Comment 9 states that an agreement by which a lawyer acquires literacy or media
rights concerning the conduct of the representation creates a conflict between the interests of the
client and the personal interests of the lawyer. After representation, 1.8(d) seems to allow
lawyers to write books and make movies, but they may also have an issue with confidentiality,
even after the representation has ended, because the attorney certainly knows confidential
information relating to representation of the client.
2. Whether a client should be able to waive the protection of rule 1.8(d): generally, no.
There is no waiver exception written into rule 1.8(d), because the ABA does not want the client to
be able to simply allow his lawyer to start writing a book or making a movie about the
representation. Not to mention the public interest: it is not good for the criminal justice system.
The public has an interest in seeing defendants receive a fair trial; plus, the jury may need to be
sequestered if there is info about the case flying around in the media.

Reading: 166-179
Rules: 1.7
Rule 1.7 → Conflict of Interest

(a) Except as provided in paragraph (b), a lawyer shall not represent a client if
the representation involves a concurrent conflict of interest. A concurrent
conflict of interest exists if:
(1) the representation of one client will be directly adverse to another client;
or
(2) there is a significant risk that the representation of one or more clients will
be materially limited by the lawyer's responsibilities to another client, a
former client or a third person or by a personal interest of the lawyer.
(b) Notwithstanding the existence of a concurrent conflict of interest under
paragraph (a), a lawyer may represent a client if:
(1) the lawyer reasonably believes that the lawyer will be able to provide
competent and diligent representation to each affected client;
(2) the representation is not prohibited by law;
(3) the representation does not involve the assertion of a claim by one client
against another client represented by the lawyer in the same litigation or
other proceeding before a tribunal; and
(4) each affected client gives informed consent, confirmed in writing.

PROFESSIONAL RESPONSIBILITY
9/22/10

Reading: 166-179
Rules: 1.7

Rule 1.7 → Conflict of Interest

(a) Except as provided in paragraph (b), a lawyer shall not represent a client if
the representation involves a concurrent conflict of interest. A concurrent
conflict of interest exists if:
• the representation of one client will be directly adverse to another client;
or
• there is a significant risk that the representation of one or more clients will
be materially limited by the lawyer's responsibilities to another client, a
former client or a third person or by a personal interest of the lawyer.
(b) Notwithstanding the existence of a concurrent conflict of interest under
paragraph (a), a lawyer may represent a client if:
• the lawyer reasonably believes that the lawyer will be able to provide
competent and diligent representation to each affected client;
• the representation is not prohibited by law;
• the representation does not involve the assertion of a claim by one client
against another client represented by the lawyer in the same litigation or
other proceeding before a tribunal; and
(4) each affected client gives informed consent, confirmed in writing.

Variation 1:

INSERT NOTES FROM MONDAY

Variation 2: RPC 1.7(a) Analysis

• Green represents Dan in a boundary dispute w/ Sarah. Polly


thereafter asks Green to represent her in a lawsuit against Dan
who broad-sided her car at an intersection.
o Existing Representation of Dan = boundary dispute
Dan (EC)(Green) → boundary dispute → Sarah (other
lawyer)

o Potential New Representation of Polly = Negligence Action


Polly (PC)(Green) → Negligence → Dan (EC)(?)

• Will Green’s representation of Polly involve a concurrent conflict


of interest?
o What does it mean to represent a client who is directly
adverse to another client?
 1 client benefits at the other client’s expense
o What does it mean to represent a client under the
circumstances in which the lawyer’s efforts that the client
might be materially limited by her responsibilities to
another client?
 The lawyer will be tempted to pull his punches with
representation of 1 client to another

Polly (PC)(Green) → Negligence → Dan (EC)(?)


1.7(a)(1)? YES 1.7(a)(1)? YES
1.7(a)(2)? YES 1.7(a)(2)? NO (Maybe)

o Dan 1.7(a)(1) issue? YES


 Why might Dan feel betrayed if Green represents
Polly in car accident?
 If client who has been represented by attorney for a
while & trust will feel betrayed if gets a summons
from his attorney in another case.
 Because of betrayal → Dan may not be as forth
coming w/ info to Green
o Polly 1.7(a)(1) issue? YES
 From a textual perspective → if she becomes a client
then her interests are directly adverse to another
client (Dan)
 BUT → Dan feels 1.7(a)(1) conflict more significantly
than Polly b/c he is already an existing client (EC
feels the betrayal)

o Dan 1.7(a)(2) issue? NO (Maybe)


 Focusing just on (a)(2) → the EC will rarely have any
concern that representation of the new client will
have any effect on the lawyer’s representation of him
in existing matter
 NOTE: Theoretically possible that there can be
an (a)(2) EC when the nature of the
allegations in the lawsuit b/t EC & NC that
cast the EC in such a light that it may erode
the lawyer’s ability to represent the EC in an
existing matter
EX: Polly sues Dan for sexually abusing her son

o Polly 1.7(a)(2) issue? YES


 Feel that her representation will be limited in a
material way b/c Dan already client of Green & be
concerned that Green will not aggressively represent
her against Dan b/c doesn’t want to chap off Dan
Variation 1: RPC 1.7 (b)(1) Analysis

• Does Green reasonably believe that he can provide competent &


diligent representation to Polly in the PI action & also to Dan in
the boundary dispute? What should Green take into
consideration?

o Representation of Polly in negligence action → Any


potential competence or diligence problems?
 Green should decide → whether or not he would be
able to do a good job for Polly OR whether his work
for Polly will be adversely effected by his
representation of Dan
** Would Green Pull any Punches representing
Polly?
 Things that might affect Green’s ability to represent
Polly:
• length of his relationship w/ Dan
• # of times he has represented Dan in past
• Amount of fee income Dan has generated in
past
• Amount of fee income Dan may generate in
future

 If Dan was 1 time representation → then Green will


probably have NO problem representing Polly
 BUT if Dan is a constant client (cash cow) → then
Green probably would not want to upset Dan & would
be likely to pull his punches representing Polly
(THUS: adversely affecting his representation of
Polly)

o Representation of Dan in boundary dispute → Any


potential competence or diligence problems?
 Not really, b/c boundary dispute is completely
unrelated matter! Green has no reason to quit
representing Dan aggressively in boundary dispute
Variation 1: RPC 1.7(b)(2) Analysis

• Is Green’s representation of Polly prohibited by law given that he


already represents Dan in an unrelated matter? NO → (b)(2) is
NOT an issue
o EX: (b)(2) prohibited representations
Cannot represent multiple ∆s in a capital criminal case
Federal Gov’t employees are prohibited in
representing clients in agency matters after they leave
agency employment

Variation 1: RPC 1.7(b)(3) Analysis

• Would Green’s representation of Polly result in Green


representing the ∏ & ∆ at the same time & in the same litigation
matter? NO
o NO → b/c 2 matters are wholly unrelated

Variation 1: RPC 1.7(b)(4) Analysis

• Has Green obtained Polly’s & Dan’s informed consent, confirmed


in writing?
o Informed Consent 1.0(e) → Must lay out all the risks &
all the alternatives
o Confirmed in Writing 1.0(b) → Document actually
signed by client & letter memorializes a previous oral
agreement in a letter/memo to the client

o What should Green talk about w/ Polly?


 Need to discuss reasonable available alternatives →
IE: hire another lawyer w/ no conflict
 Need to discuss risks of conflicted representation

o What would you want to know if you were in Polly’s


position?
 The extent that Green is connected to Dan
• Is Dan a cash-cow client?
• Length of Green’s representation of Dan
• How aggressive will Green be in representing
her?
• Assurance of confidentiality in info she shares
w/ Green
 NOTE: Before Green has conversation w/ Polly
about his relationship w/ Dan → Green must
get Dan’s consent to share the info re:
Green’s representation of Dan
1.6(a) → must obtain existing client’s
informed consent
If Dan says NO → Green CANNOT share the
info w/ Polly & Green will have to decline
Polly’s representation!!

o What should Green talk about w/ Dan?


o What would you want to know if you were in Dan’s


position?
 Reassurance of confidentiality → that info shared in
past/future of representation will NOT be shared w/
Polly
 Reassurance of continued aggressive representation
in existing representation

- Reasonableness of the hourly rate and the reasonableness of the hours worked
- The attorney client privilege does not exist when the attorney represents 2 parties in joint
representation.
• Rule 1.7 Analysis: Lawyer’s believe = Subjective &
Objective
o Lawyer must believe that he can represent competently &
diligently AND Lawyer looking in must believe that Lawyer
can represent competently & diligently!!!

• Rule 1.7(b)(1) Analysis: Adversely effects representation

subjective –adjective
1. existing in the mind; belonging to the thinking subject rather than to the object of thought (opposed to
objective ).
2. pertaining to or characteristic of an individual; personal; individual: a subjective evaluation.
3. placing excessive emphasis on one's own moods, attitudes, opinions, etc.; unduly egocentric.
4. Philosophy. relating to or of the nature of an object as it is known in the mind as distinct from a thing in
itself.
5. relating to properties or specific conditions of the mind as distinguished from general or universal
experience.
6. pertaining to the subject or substance in which attributes inhere; essential.

objective –noun
1. something that one's efforts or actions are intended to attain or accomplish; purpose; goal; target: the
objective of a military attack; the objective of a fund-raising drive.
–adjective
5. not influenced by personal feelings, interpretations, or prejudice; based on facts; unbiased: an objective
opinion.
6. intent upon or dealing with things external to the mind rather than with thoughts or feelings, as a person
or a book.
7. being the object of perception or thought; belonging to the object of thought rather than to the thinking
subject (opposed to subjective ).
8. of or pertaining to something that can be known, or to something that is an object or a part of an object;
existing independent of thought or an observer as part of reality.

Reading: 180-198
Rules: 1.7 1.9(a)

• In any joint presentation, the attorney may not be able to keep


all information confidential.
• Once clients ask for “that protection” (privacy protection of
specific information from the other co-client) there is probably
some reason for them to have separate representation.

Problem 10: International Bolt & FNB

Variation 1:

• FNB asks you to sue IB to foreclose a loan. You represent FNB in


all of its commercial lending work & have represented IB in labor
law matters from time to time. You are not doing work for IB at
current time.

o Is IB a current or former client?


 1.3 Com. 4 & 1.16 Com. 1 → Whenever a lawyer
does piece work for a client (ex: labor law advice)
when the work is done the representation ceases
 If IB came to you like clock-work every year to do a
labor law issue would IB be more like a current
client? Probably yes

o Why would you prefer to characterize IB as a former client?


Why would IB prefer to characterize itself as a current
client?
 If IB is former client → Gives you more leeway in
representing potential adverse parties
 1.9(a) → same or substantially related matter
 If an entity is characterized as a former client → 1.9
contains lawyer’s obligations
 If entity characterized as current client → 1.7
contains lawyer’s obligations
 If IB a former client → under 1.9 you do NOT need
IB’s consent to represent
BUT if IB a current client → under 1.7 you need to
get IB’s consent

o What can a lawyer do to avoid these sorts of problems?


 Send disengagement letters OR Ask for advance
waivers of future conflicts

o “Disengagement Letters” → INSERT


o Why do you suppose lawyers typically do NOT send
disengagement letters?
 Bad marketing!! If send that letter then lawyer is
putting in client’s mind that they do not want any
more business

o Advance waivers of future conflicts → Ask client to waive


conflicts of interests in advance that may arise in the
future
 1.7 Com. 22 → usually NOT valid or enforceable
BUT sometimes maybe valid & enforceable only if 3
conditions met:
• Client giving waiver is a sophisticated
purchaser of legal services – IE: recidivist
litigator
• Sophisticated “legal” client giving waiver has
independent representation in looking at issue
of waiver
• Potential future conflicts must be able to be
identifiable in advance what those
consequences are at time client is asked to
waive conflicts

o Why do you suppose lawyers CANNOT typically get


advance waivers?
 B/c generally not feasible that all 3 conditions are
met to allow waiver to be valid & enforceable.
 More frequently → clients almost NEVER want to sign
the waivers!!!

• Same facts but assume IB is a former client so its consent is NOT


required.
o Is there a Rule 1.7(a)(2) conflict lurking in the facts that
might require you to obtain FNB’s informed consent? YES
o Whose interest creates the conflict problem in this
circumstance → FNB/IB/You? You
 Your personal interest in making $$ in future off
representing IB so creates conflict w/ FNB b/c you
might pull your punches representing FNB to curry
favor w/ IB to get future business
o What should you think about for purposes of determining
whether there is a “significant risk” that you will pull your
punches in connection w/ the representation of FNB?

• Assume IB is a current client so the foreclosure action is FNB vs. IB


o Can you terminate your attorney client relationship with IB and this
transform IB into a former client?
 The Hot Potato Doctrine (lawyer cant drop a client just to
take on a more lucrative client)
 Consistent with RPC 1.16(b)(1) (lawyer can withdrawal from
representing a client as long as there is no adverse affect to
the client)? Yes a lawyer under 1.16(b)(1) can drop a client
but that act per the hot potato doctrine would preclude the
attorney from treating the dropped client for conflicts
purposes as a “Former Client” under RPC 1.9 and must
continue to treat the dropped client as a current client in
regards to conflicts purposes.
o What are the RPC 1.7(a) issues that must be taken into
consideration…
 On the FNB side?
• Hint- FNB occupies the role of Polly (conflicted
client/new kid on the block) in this context and its
issues are primarily (a)(2) issues (significant risk that
the representation would be materially limited by the
lawyers responsibility to the other client) but there is a
technical (a)(1) issue as well (the clients would be
directly adverse to each other)
 On the IB side?
• Hint- IB occupies the role of Dan (Conflicting client,
old kid on the block) in this context and its issues are
nearly always (a)(1) issues only (its interest are
directly adverse to those of another clients of a
lawyer). (have to make sure they don’t feel betrayed).
o What are the RPC 1.7(b)(1) (lawyer must reasonably believe they
can provide competent and diligent representation) issues that
must be taken into consideration…
 Hint- the (b)(1) issues on FNB’s side are nearly always the
same as the (a)(2) issues. (because if there is a significant
risk that the lawyers ability to representation of the new client
will inhibit his ability to represent the old client there is a
serious issue as to whether the lawyer can reasonably
believe he can provide competent and diligent
representation)
• Why- because these are the issues a neutral and
detached lawyer would focus on (look at the
economic issues- to verify the lawyer really can do a
good job for FNB)
 Hint- the (b)(1) issues on IB’s side are rarely a problem
assuming the work for IB is truly unrelated to the work the
lawyer wants to do for FNB. (correct because there are no
overlapping issues and therefore the lawyer would have no
reason to represent the client just as well and just as
zealously as the lawyer would have in the absence of the
new relationship)
• Assume the load was made ot national Bolts, a wholly-owned subsidiary
of IB. You have never done any work for NB but you are currently
representing IB in a labor law matter.
o Is NB deemed to be a client for conflicts purposes simply because it
is a wholly-owned subsidiary of your client, IB?
 Just because it is a subsidiary for conflicts purposes it is not
deemed a client.
 (regarding corporate families) 2 things to consider for
purposes of determining whether IB is deemed “client” for
conflicts purposes
• (1) ABA suggest lawyers consider whether lawyer in
advance has agreed to treat all members of the
corporate family as 1 for conflicts purposes.
• (2) whether or not as a practical matter looking in it
appears that these two entities appear to be separate
and distinct (separate employees, directors, officers,
employees, etc.)
• whether the parent and subsidiary are represented as
a single entity or separate entity.
• 2 entities may be recognized under the law to
represent separate entities, may seem as 1 and are
allowed to share the same office, board of directors,
employee’s etc.)
o Assume you determine that NB is a “deemed client” for conflicts
purposes.
 Are the RPC 1.7(a) and RPC1.7(b) issues exactly the same
for FNB and NB/IB as they are whenever a lawyer finds
herself representing one client against another? If they are
one entity you have to use the same analysis under 1.7(a)
with FNB and NM/IB. they can be characterized them as
either the new kid on the block, or the old kid on the block.
o Assume you determine that NB is not a client for conflicts purposes.
 Is there a RPC1.7(a)(1) issue because a judgment against
NB would affect IB’s economic interests, thus requiring IB’s
informed consent and FNB’s informed consent? (It would be
a indirect affect, so they would only be indirectly adverse
(ABA says this is not the type of direct adverse affect and
therefore no direct conflict)
 Is there a RPC(a)(2) issue that may nevertheless require you
to obtain FNB’s informed consent. (Yes, the lawyers interest
himself creates the conflict in this instance (bank v.
Subsidiary) because the lawyer is connected to the parent
company and the the lawyer may pull his punches in
representing the bank as not to alienate the parent
company)
• Variation 3
• Your neighbor asks you to seek a declaratory judgment that the
prepayment penalty in her home mortgage loan from Second National
Bank is unenforceable. You have never represented SNB.
o On these limited fact without assuming any others, do you have a
1.7(a) analysis? No
• The president of FNB calls and tells you she is concerned about the
potentioal for an adverse decision regarding prepayment penalties and
asks you to withdrawl from the neighbor’s case. She concedes, however,
that FNB’s loans do not have prepayment penalties.
o On these changed facts is there a RPC 1.7(a)(2) issue that should
at least be considered. Yes, the lawyer’s personal interest
 What risk do you run if you do not ask the neighbor for her
informed consent? (you run the risk of malpractice, that you
will lose both clients, disciplined by Bar association, and
losing the fee’s) RPC 1.7(a)(2) says when there is a
significant risk that the lawyers rep of one or more clients will
be materially affected then there is a concurrent client, on
these facts there is not a significant risk because the bank
has no prepayment penalties.
 What risk do you run if you ask the neighbor for her informed
consent? (and the client says no) then the lawyer cant
represent her in the lawsuitif you were not required to obtain
your neighbors consent in the first places, and out of a an
abundance of caution you do so anyways then the client
says no, then there is no practical way to represent the
neighbor, because you essentially toled her you need her
consent to proceed and she said no, so you must get the
1.7(a) analysis correct in the first instance becaasue if
informed consent is required then you want to get it, and if
you don’t need informed consent then you don’t want to ask
for it.
• You receive a second phone call from the president of FNB. She tells you
if you win the declaratory judgment action, FNB will pull all of its existing
work and not refer any further work to you. FNB’s work has represented
15% of you annual revenue, on average, for each of the last 10 years.
o On these changed facts is there a RPC 1.7(a)(2) issues that must
be taken into consideration? Yes there is a personal interest
 Whose interest creates the problem?
o How does this new information affect the RPB1.7(b)(1) analysis?
 Is this conflict waivable or not waivable? This issue is not
consentable because the lawyer cannot reasonably believe
he can provide competent and professional representation.
• Variation 4
• You are one of the nation’s leading employment lawyers. You represent
both plaintiffs and defendants in title VII actions.
o Is there anything in the RPC that prohibits you from representing
both plaintiffs and defendants in title VII cases? NO
 When does a positional conflict exist? Comment 24:
 There is a significant risk
 That the lawyer’s litigation position for one client
 Will materially limit the lawyers ability to effectively litigate
another matter for another client.
o What should a lawyer look for in particular?
 Both litigation matters involves the same issue of
substantive law
 Both litigation matters are pending in the same jurisdiction
 Both litigation matters are pending at the same time.
• If all 3 factors are present then there is probably a
positional conflict, if any are missing then there
probably is not a positional conflict.
o How must the lawyer proceed if a positional conflict exist? No
different then any other conflict of interest, must go through the very
same 1.7(b) analysis, if the lawyer gets through that analysis then
he must go through a 1.7(b)(4) analysis, and the lawyer must
confirm the clients informed consent in writing.

D. Problem 12
Conflicts Between Client Interests and the Lawyer's Personal Interest
Page 203

Facts
Attorney Doe went to high school with Johnson. Johnson does not have a lot of money, but
asked Doe to help set up his business and see if he could pay over time. But instead, on Doe's
suggestion, Johnson agreed to Doe taking a 10% interest in the company in which Doe sets up.
Doe's knows business is doing well and that Johnson will be purchasing plant in area which will
increase in property value, so Doe buys land there. Johnson wants to thank Doe for all that she
did so he gives her his Mercedes. Then they fall in love and they are together when the aren't at
work.

A. Accepting Payment in the Form of Stock; business Transactions With a Client


1. Whether the law prohibits lawyers from taking all or part of their fee in the form of stock
in the client's business: comment 1 to rule 1.8(a) states that rule 1.8(a) does not apply to
ordinary fee arrangements b/t client and lawyer, which are governed by rule 1.5. However, 1.5
must be met when the lawyer accepts an interest in the client's business (i.e. stocks) or other
non-monetary property as payment of all or part of a fee. Rule 1.5, comment 4 states that a
lawyer may accept property in payment for services, as long as doing so does not create an
ownership/proprietary interest in the litigation for the lawyer in violation of Rule 1.8(i). Rule 1.8(i)
is not going to be a problem here, however, b/c the lawyer is just setting up the articles of
incorporation; there is no litigation involved. However, comment 4 to rule 1.5 also states that the
payment may be subject to rule 1.8(a). 1.8(a) is definitely involved, because the transfer of
stocks in exchange for the legal services is certainly a business transaction.
However, the fee must be reasonable under 1.5. The attorney should put
everything in writing, and advise the client to get assistance of outside counsel with regard to
whether the fee is reasonable.
4. Comment 1 states that rule 1.8(a) does not apply to standard commercial
transactions between the lawyer and the client for products or services that the client generally
markets to others (e.g. buying a car from the client's dealership, or getting a meal at the client's
restaurant). In such transactions, the lawyer has no advantage in dealing with the client, and the
restrictions in the rule are unnecessary and impracticable.
Such transactions seem to potentially present ethical problems if the client gives
the lawyer the product or service for considerably less, perhaps in exchange for future legal
services. However, professor says that this does not count as a business transaction; it is merely
a fee and is therefore not subject to 1.8. The "payment" must be valued at that time so that it can
be measured to determine whether it is a reasonable fee.

B. Using Confidential Client Information to Make Private Investments


1. Whether lawyer, w/o client's consent, may invest in a parcel of land near the
industrial park: Rule 1.8(b) says that a lawyer shall not use information relating to representation
of a client to the disadvantage of the client unless the client gives informed consent, except as
permitted or required by the Rules. Comment 5 states that if a lawyer learns that a client intends
to purchase and develop several parcels of land, the lawyer may not use that information to
purchase on of the parcels in competition w/the client or to recommend that another client makes
such a person. The rule does not prohibit uses that do not disadvantage the client.
In this case, investing in the parcel of land near the park would not disadvantage
the client, but the lawyer still might as well get the client's consent. Compare to agency law.

C. Accepting Unsolicited Gifts from Happy Clients


1. Whether the lawyer may follow the client's direction to prepare the documents
necessary to give herself title to the Mercedes Benz: lawyers are allowed to receive gifts from
their clients. If the gift is a "token" gift, that is always going to be ok. However, rule 1.8(c) states
that a lawyer shall not solicit any substantial gift from a client, or prepare on behalf of a client an
instrument giving the lawyer (or person related to the lawyer) any substantial gift unless the
lawyer or other recipient of the fits is related to the client. Comment 6 says that 1.8(c) does not
prohibit the lawyer from accepting a substantial gift, but it might be voidable by the client b/c of
undue influence. The doctrine of undue influence treats gifts from clients as presumptively
fraudulent.
Here, if the lawyer draws up the papers giving herself the Mercedes it will look
suspect because the client is not related to the lawyer, and the lawyer is the beneficiary of the
document. There is a family exception b/c the rules realize that most family members of lawyers
would want to use their family member b/c they trust them and it will save them money.
4. Whether lawyer's appointment as a compensate executor of a client's estate (or
a similar fiduciary position) is a gift from the client w/in the meaning of 1.8(c): Comment 8 states
that 1.8 does not prohibit a lawyer from seeking to have the lawyer named as executor of the
client's estate or to another potentially lucrative fiduciary position. However, these appointments
will be subject to the general conflict of interest provisions in rule 1.7 when there is a significant
risk that the lawyer's interest in obtaining the appointment will materially limit the lawyer's
independent profession judgment in advising the client concerning the choice of executor or other
fiduciary.
Thus, under 1.7(b), it seems that the only way that the attorney could act as executor is if
she receives consent from the client while the client is alive, and at the time the document is
created. If there is an issue later when the will is to be probated, the probate court will deal with
it. If the client consents before he/she dies, the lawyer has complied with the ethical rules.

D. Intimate Relationships Between Lawyers and Clients


1. Whether the lawyer's license to practice should be put at risk if the lawyer and
client enter into a consensual relationship involving physical intimacy: Rule 1.8(j) states that a
lawyer shall not have sexual relations with a client unless the consensual sexual relationship
already existed between when the client-lawyer relationship commenced. Comment 17 says that
the relationship b/t the lawyer and client is a fiduciary one in which the lawyer occupies the
highest position of trust and confidence. The relationship is almost always unequal. The rule
prohibits the lawyer from having sex with a client regardless of whether the relationship is
consensual, and regardless of the absence of prejudice to the client.
In this case, the lawyer and client did not start having sex with each other until
after the lawyer-client relationship started. Thus, under this rule, their sexual relationship is per
se unethical and it is unprofessional for the lawyer to continue in it.
[Florida's rule is 4-8.4(i)].
Note that 1.8(k) states that rules (a) through (i) apply to all the lawyers in the firm
(i.e. if one lawyer cannot do (a), then no other lawyer in the firm can either). However, (j) is not
included, which leads to the inference that if one lawyer in the firm is having sex with a client,
another lawyer in the firm can represent that client without a problem.
- Can’t have sex with your client unless you were having sex before there was an
atty/client relationship….look at rules.
- This hits the divorce lawyers.
- Personal interests + interests of client issue

• Variation on Problem 12 (RPC 1.5, 1.7, 1.8


• Variation 1
o Johnson askss doe to set up a corporation for him. Johnson explains he does
not have sufficient cash and would like to pay over time. Doe counters by
asking for a 10% stake in the company.
 Why would clients prefer to pay with stock rather than cash
• Inexpensive way to secure legal service, client doesnt have to
pay cash up front
 Why would lawyers like Doe prefer stock rather than cash?
• There is a substantial potential for upside gain over and above
what the lawyer would have made if she had taken an hourly
rate wage or contingency fee, if the stock appreciates in value
substantially.
 Which RPC’s are implicated by this simple fact pattern of stock for
services? 1.8(a), 1.7(a)(2)(b), 1.5(a)
• When a lawyer is paid in money, the lawyer must satisfy the
requirements of 1.5(a)
• When a lawyer is paid in property other than money, the
laywer must satisfy the requirements of both 1.5(a) and 1.8(a)
• When a lawyer is paid in stock, and will provide legal services
to the entity that issued the stock, then the lawyer must
additionally satisfy 1.7(a)(2)
o This additional rule is implicated because the lawyer
has a greater personal interest,
o Doe will receive 10% of the comapny’s stock in exchange for (i) preparing
articles, by-laws, and a shareholders agreement and (ii) preforming whatever
additional work may be needed during the entity’s first year.
 Is the stock a reasonable frr for these reasonable services? It is
unclear to say whether or not the fee is reasonable.
• At what point in time do we value the stock for purposes of
making this determination?
 Is the stock-services exchange fair and reasonable transaction from
the client’s persepctive? Again it is unclear from the facts provided
• If the fee satisfies RPC 1.5(a), is the transaction fair and
reasonable under RPC1.8(a)?
o Yes, most of the time but in 1.8(a) there is an additional
word fairness, which is not included in 1.5(a) and
therefore textually alone there is a difference. If you
accurately value what the lawyer gets for the servides
provided and that deal is reasonable then one could
say almost reflexively that the deal is fair and
reasonable under 1.8(a). (same alaysis that adresses
one rule(1.5(a) should adress the other 1.8(a))
 What three things must Doe do to satisfy RPC1.8(a) assuming the
transaction is fair and reasonable to the client?
• Doe must: Disclose in writing the terms of the transaction, and
how the lawyer will secure his/her interest in language that the
client can understand.
• Doe must: advise the client in writing of the desriability of
securing independent legal counsel regarding the transasction
and provide the client a reasonable amount of time to secure
outside counsel.
• Doe must: Client must give informed consent in writing and
this wriiting must disclose the essential terms of the
transaction, the lawyers involvement in the transaction, and
whether the lawyer represented the client in this transaction
(must memorialize the deal)
o What is the difference about documenting consent in
RPC 1.8(a)(3) and RPC 1.7(b)(4)? Under 1.7(b)(4)
requires lawyer to acquire clients informed consent
(which can be made orally) confirmed in writing and
1.8(a)(3) it is not permissable because the only way a
lawyer can obtain informed consent in a writing by the
client.
 How must Doe proceed if she wishes to comply with rule 1.7(a)(2) and
1.7(b)? consider whether there is a significant risk if her ability to do
good work for the client will be compromised by her interest in the
entity. Under (b) if there is significant risk the lawyer then must get
informed consent from the client.
• Is the RPC 1.7(b)(4) discussion different from the RPC 1.8(a)
(3) discussion? No, the analysis of informed consent does
double duty because the same analysis can be used for both
rules.
• Variation 2
• Doe larns that Johnson’s company is secretly contemplating the construction of a
new plant. She knows where the plant will be located and anticipates that adjacent
property values will increase once the company’s plans are disclosed. She
purchases and adjacent property after concluding the client will not need it.
o Does the purchase violate RPC 1.8(b)? no because it only prohbits things
which will disadvantage the client and if the company doesnt need the
adjacent property then there will not be a disadvantage to the client then Doe
is free to purchase the proerty.
 How does RPC 1.8(b) differ from RPC 1.6(a)? 1.8 prohibits the lawyer
from using the information to the disadvantage, and 1.6 prohibits the
lawyer from revealing information relating to the representation of a
client.
• Is “using” information different from “revealing” information?
No
• Would doe violate RPC 1.6(a) if she purchased an adjacent
parcel but did not say anything about her property? No, so
long as she purchases the property without revealing or saying
why the lawyer is purchasing the property
 What principle of agency law might create a problem for Doe even if
her purchase does not violate RPC 1.8(a)?
• Variation 2 contd.
• Doe secretly purchases an adjacent parcel and then Johnson calls a meeting and
asks Doe to attend. He announces he has located another piece of property for the
plant on the other side of town and want input from those in attendance about which
site they prefer
o Which RPC is implicated when Johnson asks Doe for her opinion about the
better site? Rule 1.7(a)(2)
 What must doe do at this point if she wishes to comply with the RPC’s
to the fullest extent possible?
• She would have to obtain informed consent and disclose to the
client that she has made the purchase of the adjacent parcel.
• Variation 3
• Johnson is pleased with does work and wishes to give her a Mercedes-Benz as a
gift. Johnson asks Doe to perpare the documents necessary to make the transfer.
o Which RPC is implicated in this circumstance? 1.8(c) tells lawyers they
cannot solicit substantila giflts from a client unless the lawyer and lcient are
related, and the lawyer cannot prepare a document conveying a gift from the
lcient to the lawyer unless they are related
 Can doe prepare the documents? No, because 1.8(c) says doe
cannot prepare documents conveying a gift to the lawyer from the
client
• Would your answer change if Johnson was willing to give
informed consent? No, doe still cannot ask for it, or prepare
the documents
 Can does law partner prepare the documents? No
• Which additional portion of RPB 1.8 is implicated in this
cirumstance? 1.8(a) tells lawyers with one limited exception if
the lawyer cant do it becasue of something in 1.8 (a)(b)(c) etc.
no other lawyer in that lawyers firm cant do it
• Variation 3 (contd)
• Johnson instead gives Doe a suitcase filled with 75,000 in cash.
o Is there anything in the RPC that would preclude Doe from accepting the
cash? No, there is nothing that precludes the lawyer from accepting a gift. 1.8
merely states a lawyer may not solicit but does not prevent a lawyer from
accepting a gift.
o Why do you suppose RPC 1.8(c) draws a distinction between gifts made
through an instrument and gifts not made through and instrument?
• Variation 3 (contd.)
• Johnson has a baseball autographed by nearly every player who has hit 500 or more
home runs during his career. Doe is a huge baseball fan and has repeatedly asked
Johnson to give it to her.
o Has doe acted consistently with RPC 1.8(c) assuming she and Johnsons are
unrelated? No
 What else might we want to know before answering this question?
The worth of the baseball to determine if the gift is usbstantial or an
insubstantial gift.
 Does comment [6] provide any help regarding what is, and what is
not, a “substantial gift”? the second and thrird sentences provide
insight as to what is an insubstantial gift (a simple gift as would be
given at a holiday, or a token of appreciation) (term substantial is not
expressly defined)
o How might your RPC 1.8(c) analysis change is Doe and Johnson are...
 Child and Parent? This is an exception to 1.8(c) which prohibits
solicitation of gifts from unrelated persons but does not preclude
solicitation of gifts from lineal ancestors, and decendents, and
spouses, therefore this is allowed.
 Brother and Sister? May be related or may be unrealted depends on
whether these non linear related persons continue to have a close
familiar relationship with the lawyer and if so they are deemed related
as per the rule.
• Variation 4
• Johnson and doe become involved at a personal level and are headed toward an
intimate relationship.
o What are Doe’s two options in this circumstance, assuming she wished to
comply with the RPC’s? continue to do legal work with Johnson and not have
an intimate relationship with him; or discontinue legal work and she may have
an intimate relationship with him.
 Does your answer change if we identify the client as the company
rather that as Johnson? RPC1.8 Comment 19 says if the individual
either direct or supervises the lawyers work on beahlf of the client or
someone who the lawyer regualrly consults then the individual
becomes someone the lawyer may not have an intimate relationship
with unless the lawyers stops representing the client.
 Does your answer change if Johnson is willing to give his informed
consent to Doe’s continued service as his lawyer? No because rule
1.8(j) does not indicate that conflcit cannot be cured by consent
o If Doe decides to have an intimate relationship with Johnson, can she ask
one of her law partners to handle his legal work? Yes
o Why isnt’ the conflict imputed to her partners in this circumstance? 1.8(k)
typically refers to all the other lawyers in the firm with the exception of 1.8(j)
therfore allowing a person in the firm to represent the client
• Variation Five
• Tex is one of the nation’s leading criminal defense lawyers. He is writing a book
about his exploits and wants to include a chapter about his current representation of
a professional athlete accused of using performance enhancing drugs.
 Can tex ask the client, at this point in time, for permission to include
information aboiut the representation in his book? No, prior to the
conclusion of this particular representation tex cannot ask for
permission to include information about this representation in his book
1.8(d) par. 3
o What do you suppose is the rationale for this rule? rationale in the absence of
an outright prohibition, a conflict of the lawyers interest and the clients
interest may arise.
 (lawyers book/movie rights would have the greatest value if the case
goes to trial, and if the client pled out then the book/movie rights
would be significantly reduced)
 1.8(d) does not allow for informed consent for the conflict of interest
• Variation 6
• Tex is handeling a personal injury matter for a client who is struggling financially. The
client asks for a $5000 loan so that she can pay rent until her case is resolved?
o Can tex make the loan consistently with the RPCs? 1.8(e) says there are only
two instances when a lawyer can provide financial assistance to a client.
o How might/would your answer change if Tex were helping the client set up a
corporation rather than handling a personal injury matter? It would change
because the rule (1.8(e) only specifically names lawyers in a litigation
standpoint therefore a transactional, real estate, or other kinds of non
litigation attorney could make a loan to a client.
 Does this difference make sense to you? Yes because the litigator
would have to much of a stake in the litigation, and could possibly
influence the lawyers decision in regards to the client in litigation. In
contrast that conflict is not necessarily present in the transactional
context.
 Is there another portion of rule 1.8 that tex would have to satisfy if he
wished to make a loan to this client? Rule 1.8(a) deals with lawyers
entering into a loan outside litigation. When a transactional lawyer
wants to give a loan a lawyer must comply with all the requirements of
1.8(a), therefore he must look at the requirements of (a) 1-3
• transaction/terms on which the lawyer makes the loan to the
client must be fair and reasonable from the clients perspective,
• transaction, and the terms which the lawyer acquires the
interest must be fully disclosed (in writing, in words the client
can understand)
• tex must advise client in writing of the desireability of seeking
independent legal counsel and provide the client a reasonable
amount of time to secure the legal counsel,
• informed consent must be in a writing signed by a client
• Variation 7
• Tex is defending a motorist in an automobile negligence case.The motorist’s
insurance company hired Tex to defend the motorist and is paying ....
o What portion of 1.8 is uniquely applicable in this circumstance? 1.8(f)
 What must Tex do to ensure that he complies with this portion of Rule
1.8?
 Tex must Obtain the clients informed consent (doesnt have to be in
writing therefore it can be Oral informed consent)
 Tex must ensure during the course of the representaion there is no
interference with his professional judgment because he is being
payed by someone other than the injured client and ensure there is no
interference with his attorney client privilege since he is being payed
by someone other thatn the client.
• What is the “independence of professional judgment” problem
in the insurance defense context?
o How might the interest of the insurer differ from the
interest of the insured with respect to the amount of
effort the lawyer puts in defending the case?
o Tex must information regarding to the representation to
the client must be protected, 1.6(a) protects the
information regarding the client from the insurance
company,(cannot reveal information relating to the
representation of a client, but can in 2 circumstances
 (1) when the client gives informed consent

 (2) the disclosure is impliedly authorized by the


nature of the representation.
• VaRIATION 8
• Tex rep...
o Can tex immediately accept the offer if he thinks it reflects a good settlement?
No 1.8(g)
 Tex must speak with his clients about the offer pursuant to RPC 1.4(4)
 Tex must abide by their decision regarding settlement pursuant to
RPC 1.2(a)
o Because the offer covers the cliams of both plaintiffs what protion of RPC 1.8
is also applicable? 1.8(g) (multiple parties in a civil action and those parties
make or intend to make a settlement offer.
 How would you summarize the requirements of this rule?
• Tex must obtain
• In this context, informed consent requires: a writing signed by
each client (oral informed consent is not enough, nor is oral
consent confirmed in writing)
o A discussion of the nature and basis of each claim
o Client agreement to overall settlement amount and to
what amount each client will receive
 What happens if either client rfuses to give
informed consent to either of these things? If
these are not reached then a settlement cannot
be reached, each client essentailly can veto
either the big number or their individual number.

E. Problem 13

Representing the Insured and the Insurer


Page 215

Facts
Terry Tennant son in law of Larry Landlord rented an apt where he slipped on ice. Larry saw and
ran to him to help and Terry said he was all right so Larry didn't report the accident to All-Mutual.
Tenant later sued his landlord father-in-law for 175K All Mutual requires notice of accident
(failure means waiver), it will provide the lawyer to defend the insured, policy requires the insured
to cooperate with All Mutual in the defense. All Mutual hired Henderson to defend Landlord.
Henderson is suspicious of Tenant's stories. Tenant has offered to settle for 50k. Landlord wants
All Mutual to agree rather than go to trial and put Landlord at risk for the 75 K in excess of his
policy coverage. Henderson believes Landlord should be denied coverage for failing to promptly
notify. Henderson also thinks Landlord isn't cooperating since Landlord thinks fondly of Tenant.
Henderson hasn't told All Mutual any of this. All Mutual has asked Henderson's advice on
whether they should agree to the settlement.

A. A Lawyer's Client When Retained by an Insurer to Represent an Insured


1. In this case, the insured is the client. Rule 1.8(f) states that a lawyer shall not
accept compensation for representing a client from on other than the client unless the client gives
informed consent, there is no interference with the lawyer's independence of professional
judgment or w/the client-lawyer relationship, and information relating to representation is
protected as required under rule 1.6. (Remember: this rule applies with any person other than
the client who pays for the client's legal services; including parents). However, this can cause a
conflict under 1.7(a)(2) with the insurance company, especially if their goals are not aligned with
the insured's. Conflict can be waived, however, if client gives informed consent.

B. A Lawyer's Obligation to Protect Confidential Information of the Insured


3. It may be fraudulent for the lawyer not to say anything to the insurance company
at the request of the insured. Under 1.6(b)(2), a lawyer may reveal information to prevent the
client from committing a fraud. However, there is no evidence that the client conspired with
someone to arrange the slip and fall; therefore, the lawyer may have to refrain from telling the
insurance company under 1.6 b/c the information is confidential and relates to the representation
of the client.
If there is a conflict b/t insured and insurer, the lawyer may have to withdraw,
under the Florida rules.

• Variation 3
• W & X represents World Wide in a lawsuit against National Gasket. Willis is the only lawyers
working on the case Xerex handled a case for NG while he was a solo practitioner. The case
involved the smae facts
• Can Xeres represent World Wide consistently with RPC 1.9(a) Yes, unless informed
consent in writing is given
• Can Willis represent World Wide Consistently with RPC 1.10(a)
• What can W & X do that would allow Willis to representWorld Wide even though Xeres
cannot represent World Wide?
• How would your answer change if Xeres represented...
• How might your answeeer change is Xeres represented National Gasket Whle he was a
partner at W&X but left the firm of W&X before woirl wide approached W ad asked for his
help? Gerrys departure takes with him the taint of national gasket
o How does RPC 1.10(B) work? When a lawyer is terminated (leaves the firm) the
firm is not prohibites from representing a person with interests materially adverse
to those of the client represented by the formally assoiated lawyer and not
currently represented by the firm, unless (1) the matter is the same or
substantially related to that in which the formerly associated lawyer represented
the client, and (2) any awyer remaining in that firm has information protected by
rule 1.6 and 1.9*c) this is material to the client.
• Variation 4
• A lawyer was sexually abused by a foster parent while in a foster home as a child...
o Can the lawyer represent the foster parent assuming he does not subjectively
believe he can provie competent and diligent representation? No there is a
conflict between the lwyers own personal interest and the person he is wishing to
represent. (concurrent conflict of interest)
 Which RPC precludes the lawuyer from handeling this case? 1.7(a)(2)
o Would other lawyers in the same firm likely be precluded from representing the
foster parent? No 1.10(a)(1) says in general a personal interest conflit of one
lawyer are not imputed to the other lawyers in the firm.
 How might your answer change if the disqualifies lawyer trained an
promoted all the other lawyers in the firm and, as a result, they owe their
professional status to him? They will probably not be able to provide
competent and professional representation to the client, because of their
own personal interest to the lawyer who trained them, then they have
their own personal interest conflict by way of their loyalty to the lawyer
who trained them.
 How might your answer change if the disqualified lawyer has unilateral
control over the compensation received bu all the other lawyers in the
firm? Then the remaining have an even bigger personal interest congflict,
because the representation of the foster parent ay alienate the lawyer so
much that their own economic interest may be affected.

F. Problem 14

The Lawyer and Her Former Client


Page 230

Facts
Heath (well known attorney) has been retained by Linda Parker with a claim against Dr. Abraham.
However 5 years earlier, Heath represented doctor in the routine adoption of his wife's children.
Doctor is pissed and moved to disqualify Heath as counsel for Parker
Rule 1.9(a)
1. Same or Substantially Related Matter (comment 3 and 2)
2. Interest are materially adverse
3. Former client informed consent
A. Matters as to Which Disqualification Is Required
1. Whether a lawyer owes the same duty of loyalty to a former client as to a current one;
what standards define when a lawyer may undertake a matter that is contrary to the interest of a
former client: According to Judge Weinfield, former client must show that the matters in the
pending suit are substantially related to the previous representation. Rule 1.9(a) also says that
they must be substantially related. To decide if the matter is the same or substantially related,
you must conduct a loyalty test and confidentiality. The concern is that you have confidential
information that may be harmful to people. You can't really monitor people using the confidential
information.
Even though 1.9(c) stops a lawyer from using this confidential information, that is not
enough to say that we won't have a 1.9(a) problem--even though the lawyer cannot reveal it or
use it. If you are revealing it you make it known; however, if you can use it for strategy such as
in settlement (if they are going to run out of money) and it is for the betterment of lawyer's current
client, you can reveal it.
2. Whether the term "matter" refers to only litigated cases: Matters are not limited to
causes of action. Comment 2 of rule 1.9 shows that matter covers more than lawsuits. Matter is
not just limited to litigated cases. Comments say that it also applies to transactions.
B. Determining When Matters Are the "Same or Substantially Related"
2. Should the current med-mal practice case and the former adoption proceedings be
held to be substantially related matters? Comment 3 (pg 45) states that matters are substantially
related if they involve the same transaction or legal dispute, OR a substantial risk that confidential
factual information as would normally have been obtained in the prior representation would
materially advance the client's position in the subsequent matter. You look at the facts of the
cases and extent of the lawyer's involvement. Even in factually dissimilar cases, legal issues
might be the same. [Professor mentions Cardona v. General Motors Corp. case on page 235
where the court held that the cases were substantially related].
In this case the client is making a loyalty argument. The facts in this case are not the
same. Is it possible that a loyalty argument is enough to disqualify the attorney? No. If rule 1.9
were very strict, the conflict check would make it difficult to take cases.
3. Suppose attorney knows no crucial secrets (like Dr. Abraham operates while
intoxicated), but she does have general impressions of Abraham's personality and specific
knowledge of his financial situation. Could than make the cases "substantially related"?
Chugach- lawyer was disqualified; court said question not just whether he was connected to
former client, but whether he was in the position to acquire knowledge casting light on the
purpose of later acts and agreements (he left a company as general counsel when the majority
was taken out by minority and then later sued company representing a new client). But in Jelco,
it wasn't enough-court said law firm didn't have specific information that would let it prevail; just
gained general information about Jelco's attitudes towards negotiation and settlement.
This discusses the personality and the insight you gain in representing a former client
(i.e. they like to settle, have a bad temper). Chugach case says yes, the lawyer should be
disqualified. However in Jelco they said it was not enough. Under 1.9(a) Comment [3], general
knowledge of the client's policies and practices will not preclude a subsequent representation.
On the other hand, knowledge of specific facts gained in a prior representation that are relevant
to the matter will.
In order to make a disqualification claim the client does not have to reveal it to everyone.
"A conclusion about the possession of such information may be based on the nature of the
services the lawyer provided the former client and information that would in ordinary practice be
learned by a lawyer providing such services." Rule 1.9(b) and 1.10(b) are to be taken together
(note, however, that this is not relevant to this particular problem).
What if you were defended doctor in malpractice case 5 years ago and now you have
been hired by one of his patients suing same doctor. Does it matter if it is a different part of the
body? Professor mentions the lemon law case on page 235 Cardona. The question asked is
could the lawyer presumed to have learned something in the previous case. Even though they
are to different body parts you still know a lot about the doctor. Why would this lawyer even do
this?
What does materially adverse mean? It is not really defined anywhere, but just know that
it has to contain some kind of adversity. Professor says that they don't appear to be materially
adverse here b/c the matters are not substantially related. They are aligned adverse here b/c
they have been represented by the same attorney.
Are co-defendant's materially adverse? Most of the time you are going to have a loyalty
issue. They will think it hurts their case and that is adverse. Can you do it if you get consetn?
You don't need consent from your current client (Parker) b/c it would benefit her. You have to get
consent from the former client. You don't need consent from both parties; only the former client.
Go back to rule 1.7(b)(4). It says that you need consent from each affected client (but not current
client).
C. Other Situations in which Model Rule 1.9 May Require Disqualification
4. Suppose that instead of a lawyer who has represented the  changing firms, it is that
lawyer's non-lawyer assist (secretary) who does? Comment 4 in 1.10 refers you to 1.0(k)
("screened," denotes the isolation of a lawyer from any participation in a matter through the timely
imposition of procedures w/in a firm that are reasonably adequate under the circumstances to
protect information that the isolated lawyer is obligated to protect), provided that you detail how
you are going about the screening procedures when arguing the motion.
Rule 1.10 comment [4] says that it really applies to the lawyer and not to the
secretary or law student. The rules try to make it easy for secretary and law clerks to move
around. They throw this question into this problem but it really goes to 1.10-if someone in my
firm worked on this case, then no one else in my firm can. They still want to screen them from
working on the case.
5. Suppose you are a law student who worked on case for Firm A b/t summers and then
work for Firm B in the next summer? 1.10 comment 4 says that secretaries and law students are
okay so long as they are screened.
• Variation 1
• Heath represents Parker in a medical malpractice action against Abraham. Heath
assisted Abraham five years ago with the routine adoption of his wife’s children but has
not spoken to him since.
o Does this look like a RPC 1.7 situation or an RPC 1.9 situation? 1.9
o How do we determine whether two matters are “substantially related”? comment
3 if they involve the same transaction or legal dispute; ordinarily analyzed by
what information client would have given to old representation
o Same transasaction or legal dispute or
o A substantial risk exists that
 Is the client required to disclose what she told her lawyer for the
purposes of demonstrating the two matters are related? No comment 3
says the client is not required to disclose what she shared with her
former lawyer.
 If the client is not required to disclose, how does the court determone
whener there is a potential for unfair advantage? Assumption is made
given the issues involved in the earlier representation, and we presume
the information has passed.
• How can the passage of time potentially affect the analysis? The
issues may now be obsolete.
 Is the adoption matter substantially related to the malpractice matter?
No, focusing on comment 3 these two matters do not involve the same
transaction/legal dispute.
• Is this the same transaction or legal dispute?
• Is there a substantial risk that Heath learned something about
Abraham in the adoption matter that will give Parker an
advantage in the malpractice matter? Probably not, the issues
are different, a lawyer would probably not bring up the subject of
medical malpractice if an attorney is representing a adoption
case.
• Can heath help defend Abraham assuming parker would not give informed consent?
1.9(a) and (b) issue
• How would your answer change is an associate, who did not have anything to do with the
parker v. Abraham case, departed Heath’s firm and joined Black’s firm? Comment 5 says
if they have not aquired any information then there is no conflict
o 1.9(b): A lawyer who leaves one firm and joins another firm cannot thereafter
representa a person whose interest are materially adverse to those of a current
or former client of the prior firm in the same or a substantially related matter
when:
 The lawyer posses information protected by 1.6 or 1.9
 The information is not material to the representation
• How does RPC 1.9(c)(1) differ from RPC 1.8(b) a lawyer is precluded to the
representation to the client to the disadvanteage of the client, 1.9(c)(1) a lawyer is
precluded
o Why do we need RPC1.9(c)(1) given that we have RPC 1.8(b)?
• Why do we need RPC 1.9(c)(2) given that we have RPC 1.6(a)?

(Class Notes from Zena Week 10)


Variation Two
• You decide to speak with the vice president responsible for the manufacture and sale of
children’s pajamas to learn more about the situation.
• From a textual perspective, does RPC 1.13(b) permit you to approach the person
who may be the source of the problem or does it require you to approach a higher
authority in the first instance?
• Yes, you can approach the person who may be the source of the problem.
Second sentence of (b) “Unless the lawyer reasonably believes that it is not
necessary in the best interest of the organization to do so, the lawyer shall refer
the matter to highest authority in the organization, including, if warranted by the
circumstances, to the highest authority that can act on behalf of the organization
as determined by applicable law” Comment 4
• Does RPC 1.13(f) and Comment 10 suggest how you should begin your
discussion with the vice president?
• I represent the organization and not the individual
• You may want to get his/her own independent representation (counsel)
• The organization, and not you, controls the attorney-client privilege
• If the vice-president agrees there is a problem with the pajamas, and to correct it,
then is there any obligation to do anything else under RPC 1.13(b)?
• No

Variation Three
• The vice president tells you he does not believe regulators will recognize the problem. He tells you
experts believe only 1 in 50,000 children will suffer burns; that management consultants calculate
likely damages at $250K per burn victim; that annual profit is $1M even after payment of $1M in
civil damages each year, and disclosure might impact his chances of succeeding the president.
o Is the vice president violating a duty to the organization, or a law, or perhaps both?
 Both; Rule 1.13(b)
o Are you required to accept the vice president’s representation at face value?
 Not required to accept his representations. Comment [3]
 Has the vice president missed a significant type of potential injury to the
organization?
• Damage to the corporation because of the representation
 Has the vice president received a reasonable assessment of recoverable
damages?
o Are you limited to giving the vice president legal advice about the scope and effect of the
regulation or does RPC 2.1 allow you to have a broader discussion with the vice
president?
 No- other considerations such as moral, economic, social and political factors,
that may be relevant to the client’s situation
o Do you believe you should report what you have learned to a higher authority in the
organization?
 Yes but not required to do so

Rule 2.1: ADVISOR


In representing a client, a lawyer shall exercise independent
professional judgment and render candid advice. In rendering advice, a
lawyer may refer not only to law but to other considerations such as moral,
economic, social, and political factors, that may be relevant to the client’s
situation.

Variation Four
• You take the issue to the president. She tells you the vice president is
the smartest person in the organization and would never second-guess
one of the vice president’s decisions. She also tells you she wants the
vice president to succeed her and disclosing these matters would kill
the vice president’s chances.
o What do you do, if anything, after you receive the president’s
response?
 Go to the board- better interest of the organization.

Variation Five
• You take the issue to the board of directors. They listen to you and
then side with the vice-president and president.
o What do you do, if anything, after you receive the board’s
response?
 You may, but are not required, to report outside the
organization

Variation Six
• You consider going public with what you know
o Are you required to rethink your position or is your earlier
analysis sufficient to support a decision to go public?
 Analysis changes- reasonable certain injury to the
organization
 HINT- is there a difference between the conditions that
must exist for you to report up than the ones that must
exist for you to report out?
• Violation versus clear violation of law
• Likely to result in versus reasonable certain to
result in
o Is there any limitation on what you can say (or to whom you can
say it)?
 Protect the organization from injury; only make the
disclose to an appropriate authority who can protect the
organization from itself.

Variation Seven
• You rethink your position. You accept the expert’s conclusion that four
children might be severely injured each year but you conclude that the
applicable regulations are unclear. As such you cannot say that the
sale of the children’s pjs constituted a clear violation of law.
o Can you disclose what you know to an appropriate authority
pursuant to Rule 1.13(c)?
 No- absence of a clear violation of law
o Is there another rule that you might invoke if you felt strongly
about disclosing what you know to an appropriate authority?
 Rule 1.6(b)- allows a lawyer to make disclosures which are
adverse to the interest of the client
 Rule 1.6(b)(1)

Variation Eight
• Assume an altogether different scenario. You know about the pajama
problem and you take the issue to the corporation’s president. Much to
your surprise, she fires you for “rocking the boat.”
o What does Rule 1.13(e) require you to do in this circumstance?
 Obligated to inform the board of directors of your
termination and include a description of the
circumstances leading to your termination

Testing Traps for the Unwary


• Client is organization not constituents but
o Communication between the organization’s lawyer and a
constituent are protected from discovery by attorney-client
privilege but
o Organization controls privilege and can waive it without regard
to whether constituent agrees
• Obligation to make upward report attaches only when misconduct is
related to representation
• Conditions for upward report less stringent than conditions for outward
disclosure
o Violation of law vs. clear violation of law
o Likely to result in substantial injury vs. reasonably certain to
result in substantial injury
• RPC 1.13(c) ≠ RPC 1.6 (b)(1)-(3)
• Lawyer hired to defend organization or constituent, or to conduct an
internal investigation of wrongdoing, cannot make outward disclose
under RPC 1.13(c)

Variations on Problem 19
RPC 4.1, 4.2, 4.3

Rule 4.1: TRUTHFULNESS IN STATEMENTS TO OTHERS


In the course of representing a client a lawyer shall not knowingly:
(a) make a false statement of material fact or law to a third
person; or
(b) fail to disclose a material fact when disclosure is necessary
to avoid assisting a criminal or fraudulent act by a client, unless disclosure is
prohibited by Rule 1.6.

Rule 4.2: COMMUNICATION WITH PERSON REPRESENTED BY


COUNSEL
In representing a client, a lawyer shall not communicate about the
subject of the representation with a person the lawyer knows to be
represented by another lawyer in the matter, unless the lawyer has the
consent of the other lawyer or is authorized to do so by law or a court order.

Rule 4.3: DEALING WITH UNDERREPRESENTED PERSON


In dealing on behalf of a client with a person who is not represented by
counsel, a lawyer shall not state or imply that the lawyer is disinterested.
When the lawyer knows or reasonably should know that the unrepresented
person misunderstands that lawyer’s role in the matter, the lawyer shall
make reasonably efforts to correct the misunderstanding. The lawyer shall
not give legal advice to an unrepresented person, other than the advice to
secure counsel, if the lawyer knows or reasonably should know that the
interests of such a person are or have a reasonable possibility of being in
conflict with the interests of the client.

RPC 4.1(a) Questions


• What is the basic obligation imposed by RPC 4.1(a)?
o Be truthful
o Does RPC 4.1(a) prohibit a lawyer from telling a lie to her client?
No
o Does RPC 4.1(a) prohibit a lawyer from telling a lie to a friend at
a social function? No
 Rule 8.4(c) – applies in these two circumstances
• Does RPC 4.1(a) require a lawyer to disclose facts about which she is
aware but which her adversary appears to be unaware?
o No affirmative obligation to disclose what you know. Comment
[1]
o True or false- one exception exists when the client dies during
the course of the representation
• Does RPC 4.1(a) prohibit a lawyer from puffing and posturing about a
client’s willingness to settle, or the amount at which she is willing to
settle, during negotiations?
o No- Comment [2] not a material fact for the purposes of Rule
4.1(a)

RPC 4.2 Questions


• What is the basic obligation imposed by RPC 4.2?
o Cannot have ex parte communications
o Does RPC 4.2 prohibit ex parte communication with represented
persons or represented parties?
 Represented persons
o Does RPC 4.2 prohibit ex parte communication with respect to
all subjects or some subjects?
 Only to subject of the representation; not all subjects
o Does RPC 4.2 prohibit ex parte communication with persons who
are represented by another lawyer or person who are
represented in the matter by another lawyer?
 Represented in the matter by another lawyer
• Who is off limited when the represented person is a corporation?
o Comment [7]
o Current employees who are either
 Individuals who direct, supervise, or regularly consults
with lawyer about the matter
 Individuals who have the authority to bind the
organization with respect to the matter
 Individuals whose acts/omissions in connection with the
matter may be imputed to the organization for purposes
of civil or criminal liability
o Why are former employees typically not off limits?
 Former not current; do not do anything that the current
employees do

RPC 4.3 Questions


• True or false- a lawyer must always tell an unrepresented person that
she is lawyer before she can interact with the unrepresented person?
• True or false- a lawyer must always identify her client before she can
interact with an unrepresented person?
• What must a layer do and what must a lawyer not do when
interacting with an unrepresented person?
o The lawyer must not state or imply that she is disinterested in
the matter
o The lawyer must correct misunderstanding about the lawyer’s
interest in the matter
o The lawyer must not give legal advice, except for “you should
get a lawyer,” when if the lawyer knows or reasonably should
know that the interests of such a person are or have a
reasonable possibility of being in conflict with the interests of
the client.

10/20/10

Variation One
• A pedestrian is stuck by a Speedy Corp delivery truck. The pedestrian
thereafter retains Shabazz to represent him in a negligence action.
Shabazz interviews Barry Winters, a Speedy Corp accountant, and the
truck driver about the accident.
o Why would a lawyer like Shabazz want to interview witnesses
before initiating litigation?
 To determine if there is a cause of action (COA) RPC 3.1,
Rule 11 (rule of civil procedure)
 Why would a lawyer like Shabazz prefer to interview
witnesses even after initiating litigation?
• Determine if they would be good for trial, less
expensive then deposing a witness.
o Can Shabazz properly interview Barry Winters assuming Shabazz
does not know that Bentley represent Speedy Corp in the
matter?
 Yes, because he has no knowledge of the representation,
RPC 4.2
 Would your answer be any different if Shabazz knew that
Bentley represents Speedy Corp in the matter?
• No, because Barry Winters is not one of the 3
categorizes of employees under Comment [7]
o Can Shabazz properly interview the truck driver assuming
Shabazz does not know that Bentley represented Speedy Corp in
the matter?
 Yes, he does not know that there is a lawyer representing
the Speedy Corp in this matter
 Would your answer be any different if Shabazz knew that
Bentley represents Speedy Corp in the matter?
• Yes, he is the employee whose negligent actions
will be imputed to the organization for the purposes
of fixed liability in this case. He cannot talk to the
truck driver.

Variation Two
• Shabazz commences a negligence action and names Speedy Corp and
the truck driver as defendants.
o Can Bentley represent both Speedy Corp and the truck driver in
this matter?
 Yes- because there is no concurrent conflict of interest
 Why is there an RPC 1.7(a)(2) conflict between the
interests of Speedy Corp and the truck driver?
• Significant risk that the representation of one or
more clients will be materially limited by the layer’s
responsibilities to another client
• Which fact would Speedy Corp presumably want to
emphasize to minimize its overall exposure given?
o Only liable because of the negligent actions
of one of the employees that happened
during the scope of employment; vicarious
liability. Indemnification- can recover back
each and every dollar from the employee.
o Pin all responsibility on the employee to
preserve its right of indemnification.
• Which fact would the truck driver presumably want
emphasize to minimize his overall exposure?
o Take the position with purposes of complying
with the policy; it’s the policy’s fault not the
driving that is the proximate cause of the
pedestrian’s injuries. Other ∆ is solely liable
for the accident.
 Is this a concurrent conflict that can be waived so long as
both clients provide informed consent, confirmed in
writing?
• No- if there is a 1.7(a) violation look to (b)(1) and
therefore cannot ask for consent under (b)(4).
Reasonably believe that they can provide
competent and diligent representation to both
clients.

Variation Three
• Bentley serves and files a notice of appearance stating that she will be
representing Speedy Corp and the truck driver.
o Can Shabazz avoid knowledge about Bentley’s involvement in
the case simply by refusing the read the notice of appearance?
 Knowledge would be inferred in this situation- 1.0(f)- by
virtue of receipt
o Does Bentley’s notice of appearance prevent Shabazz from
conducting interviews of former employees of Speedy Corp
regarding the effect of the one-hour delivery policy on safety
issues?
 No, because they do not fall into one of the three
categories under Comment [7] of RPC 4.2 purposes
 What might Bentley consider doing to ensure that
Shabazz cannot interview the former employees on an ex
parte basis?
• Represent them
 What rule might prevent Bentley from implementing this
strategy?
• RPC 1.7 to determine whether there is a concurrent
conflict and if it can be waived

Variation Four
• Bentley decides not to represent the former employees but she wants
to speak with them about the impact the one-hour delivery policy has
had on safety issues?
o Which RPC should Bentley be mindful of when she speaks with
the former employees?
 RPC 4.3
• The lawyer must not state or imply that she is
disinterested in the matter
• The lawyer must correct misunderstanding about
the lawyer’s interest in the matter
• The lawyer must not give legal advice, except for
“you should get a lawyer,” when if the lawyer
knows or reasonably should know that the interests
of such a person are or have a reasonable
possibility of being in conflict with the interests of
the client.
o What must Bentley do if one of the former employees says that
he read about the accident and is thankful to have her as his
lawyer?
 Tell the former employee No, that he represents the
Speedy Corp.
o What must Bentley do if one of the former employees asks her
whether he is exposed to any claims as a result of the accident?
 Run through the analysis; Cannot give advice- be in
conflict with the client’s interest

Variation Five
• One month after Shabazz commences a lawsuit Mary Speedy decides
to telephone the plaintiff directly and offer to settle the case.
o Can Mary speak with the plaintiff consistently with RPC 4.2?
 Yes, RPC only applies to lawyers, nothing prohibits client
to client communication
o Can Bentley give Mary advice about what to say during the
conversation with the plaintiff?
 Nothing impermissible in advance of a client to client
communication
o How might your answer change if Bentley came up with the idea
of having Mary speak directly to the plaintiff?
 Take into consideration of whether Mary is an alter-ego of
Bentley. RPC 8.4(a) clause 3- Bentley cannot ask Mary to
have that conversation- Violate or attempt to violate the
RPC or do so through the acts of another

Variation Six
• Mary Speedy telephones Shabazz to speak with him about settling the
case.
o Can Shabazz speak with Mary because she initiated the call?
 No- RPC 4.2
 Would your answer change if Shabazz said he cannot talk
to her but she replied that she can fend for herself?
• No- there are no exceptions
 Would your answer change if Shabazz said he cannot talk
to her but she replied that she just fired Bentley and is
therefore underrepresented?
• Yes- can have a conversation with her

Variation Seven
• Shabazz telephones Mary Speedy to talk about the budget for a
homeless shelter for which they both serve on the board of directors.
o Is this communication prohibited by RPC 4.2?
 Only prohibits ex parte communications that relate to the
subject of the representation

Testing Traps for the Unwary


• There is no general duty to disclose facts about which an adversary is
unaware
• A lawyer can puff and posture during settlement negotiations
• The no contact rule applies to represented persons not just
represented parties
• The no contact rule applies only to communications regarding the
subject of the representation not all subjects
• The prohibition on giving legal advice to an unrepresented person,
other than “you should get a lawyer,” applies only when there is a
reasonable possibility of a conflict between the interests of the client
and the interests of the unrepresented person

Variations on Problem 22
RPC 1.2, 1.6, 2.3

Rule 2.3: Evaluation for Use by Third Persons


(a) A lawyer may provide an evaluation of a matter affecting a client
for the use of someone other than the client if the lawyer reasonably believes
that making the evaluation is compatible with other aspects of the lawyer’s
relationship with the client.
(b) When the lawyer knows or reasonably should know that the
evaluation is likely to affect the client’s interests materially and adversely,
the lawyer shall not provide the evaluation unless the client gives informed
consent.
(c) Except as disclosure is authorized in connection with a report of an
evaluation, information relating to the evaluation is otherwise protected by
Rule 1.6.

Evaluations for Use by Third Persons


• How would you summarize RPC 2.3(a)?
o Whenever there is a client need, to share information about the
client to a third person it is okay to do so, so long as reasonable
belief is compatible with aspect of relationship to the client.
o Who is the intended beneficiary of the evaluation?
 Third person
 Does a lawyer who provides an evaluation assume any
tort duties to the recipient?
• Yes- Duty to be honest with the third party
 Does a lawyer who provides an evaluation owe any ethical
duties to the client?
• Yes- compatible with the representation of the
client
• RPC 2.3(b)- knows or reasonably should know that
the evaluation is likely to affect the client’s
interests materially and adversely.
• RPC 2.3(c)- information is still protected by RPC 1.6
o Is an opinion letter a type of evaluation for use by a third
person?
 Yes- lawyer’s opinion about some matter (can be positive
or negative)
 Client is selling real property wants to give the buyer an
opinion letter from her lawyer stating that client has good
title and that there are no competing claims of ownership
 Auditor of client’s financial condition wants opinion from
client’s lawyer that client is not involved in litigation and
that lawyer is unaware of any facts that could give rise to
a claim.
• How does RPC 2.3(c) affect the way a lawyer proceeds when preparing
an evaluation, such as an opinion letter?
o Retain the client’s confidences unless he has informed consent-
RPC 1.6(a) that will be contained; disclose information- informed
consent/permission and so long as disclosure is impliedly
authorized.
o Does the lawyer typically need the client’s informed consent to
make disclosure when preparing an evaluation?
 Yes/No depends on the situation
 If the client asks the lawyer to prepare the evaluation,
then the lawyer is impliedly to make disclosure in order to
carry out the representation.
 But, RPC 2.3(b) will still require the lawyer to consider
whether the information would have a material or adverse
affect on the client’s interest.

Rule 1.6: CONFIDENTIALITY OF INFORMATION


(b) A lawyer may reveal information relating to the representation of a
client to the extent the lawyer reasonably believes necessary:
(2) to prevent the client from committing a crime or fraud that is
reasonably certain to result in substantial injury to the financial interests or
property of another in the furtherance of which the client has used or is using
the lawyer’s services;
(3) to prevent, mitigate or rectify substantial injury to the
financial interests or property of another that is reasonably certain to result
or has resulted from the client’s commission of a crime or fraud in
furtherance of which the client has used the lawyer’s services;

Permission to Disclose
• How would you summarize RPC 1.6(b)(2)?
o Does (b)(2) apply to situations in which the fraud is on-going or
completed?
 On-going; proactive disclosure to protect the client from
committing the crime or fraud
o Does (b)(2) permit disclosure if the fraud is unrelated to the
work the lawyer is doing for the client?
 No- Lawyer is not permitted; Must have assisted the client
in things that are setting up the fraud; advancing the
client’s fraudulent intent
o What is the purpose of disclosure in this context?
Prevent the use of legal services to carry out fraud; trying

to protect victim from the fraud
• How would you summarize RPC 1.6(b)(3)?
o Focused on completed fraud; disclosure is intended to mitigate
the damages of the client’s fraudulent acts
o Does (b)(3) apply to situations in which the fraud is on-going or
completed?
 Completed
o Does (b)(3) permit disclosure if the fraud is unrelated to the
work the lawyer is doing for the client?
 Only when the lawyer’s services have been used to
further the crime or fraud
o What is the purpose of disclosure in this context?
 Protecting the victim from the client’s fraudulent crime
and activities
• How does RPC 1.13(c) differ from RPC 1.6(b)(2) and (3) with respect to
o The intended beneficiary of the disclosure?
 1.13(c)- client
 1.6(b)(2) and (3)- victim
o The person whose injury is to be avoided or reduced?
 1.13(c)- protect the client who is the victim
 1.6(b)(2) and (3)- protect the victim’s financial interest
and property
G. Problem 15

Imputed Disqualification
(Page 244)
Facts
Charles and Burls, C & B, big wall street firm, represents national cliental including Worldwide
Containers Corp. in many matters one of which is a suit by National Gasket Co. against
Worldwide for contribution in a products liability case. Case is to be tried in New Orleans and C &
B is cooperating with Willis & Xeres, the law firm that World Wide uses as local counsel in New
Orleans.
Willis of W & X is only lawyer in that firm working on the case. His only role is to file papers,
motions, and other pleadings forwarded to him by C & B. National Gasket has sought to disqualify
C & B and W & X from acting as World Wide's lawyers b/c Xeres (while he had been a solo
practitioner before forming W & X) had represented National Gasket in various product liability
matters arising out of the same facts that led to the present suit. Xeres learned confidential
information that if disclosed would be useful to World Wide's defense in the present suit. C & B
has never represented National Gasket.
A. Imputation of Conflict Throughout a Law Firm
1. Whether Willis may continue to act as local counsel for World Wide: We know that
Xeres is disqualified under rule 1.9(a) b/c National Gasket was a former client. Since he is
disqualified under 1.9 (a), it does not that he is not doing anything in this case. Under rule
1.10(a), Xeres' disqualification under 1.9(a) imputes to the whole firm, thus including Willis under
1.10(a). If X is disqualified, then so is W. Comment 1 to 1.10 says that a firm is essentially one
lawyer. Comment [2] says that it only applies to lawyers who practice in a law firm.
If this is a personal conflict as opposed to the 1.7 and 1.9 conflicts (i.e. your personal
belief is that everyone involved in corporate wrongdoing is horrible), you do not have to be
involved in the litigation but someone else in your firm can. Personal conflicts are not subject to
this rule.
The premise of 1.9(b) is to allow a lawyer to be able to move around and switch firms.
1.9(b) operates to disqualify the lawyer only when the lawyer involved has actual knowledge of
information protected by rules 1.6 and 1.9(c). Under 1.0(f) "knowingly" means actual knowledge
of the fact in question. A person's knowledge may be inferred from circumstances.
B. Persons and Firms to Which Imputation Will Extend
1. If the court disqualifies the firm of W & X and all of its partners and associates, may C
& B simply get new local counsel? Will the disqualification of W & X required that C & B be
disqualified? Comment 1 says that "firm" means lawyers in a law partnership, professional
corporation, sole proprietorship, or other association authorized to practice law, or lawyers
employed in a legal services org., or the legal department of a corporation, or other organization.
(See 1.0(c)). W is not doing the research, they are just filing the papers. Comment [2] says that if
they hold themselves out to the public as one firm, then yes. Professor says that they don't
appear to be holding themselves out to be one firm. It is World Wide's local counsel and C&B
has their local firm. They do not seem to hold themselves out as one firm. Would there be any
harm in picking a different local counsel then W&X? No, since W is working on the case, and
C&B may not be imputed by X of W&X.
To determine if they hold themselves out to the public as one firm: Have to determine if
they are affiliated with each other by looking at if they have a close and regular continuing and
semi-permanent relationship (p249).
2. If Xeres' confidential knowledge about National Gasket is imputed to Sandra
Jones, an associate of W & X, and then Jones leaves to join a 2nd firm, what does Model Rule
1.9(b) say whether her new firm will also be disqualified? According to comment 5 of rule 1.9,
Jones must have actual knowledge. Where the lawyer has acquired no knowledge or information
relating to a particular client of the firm, and that lawyer later joined another firm, neither the
lawyer individually nor the second firm is disqualified. If Jones is disqualified, then it will impute
the whole firm. When we have
a lawyer that moves around we don't have 1.9(a) but 1.9(b). Rule 1.9(b) deals w/ where someone
was in a firm formerly associated with that client. The test is: 1.
Are her interest materially adverse? [Yes]
2. About whom the lawyer had acquired confidential information? [Does not appear that she has
any confidential information based on the facts we know].
This rule allows this b/c don't want to make it difficult for others to move around.
3. Now suppose Xeres resigns from the firm of w & X, Will W still be disqualified?
1.10(b)-when a lawyer has terminated an association with the firm, the firm is not prohibited from
thereafter representing a person with interests materially adverse to those of a client represented
by the formerly associated lawyer and not currently represented by the firm unless: 1. matter is
same or substantially related, and 2. any lawyer remaining in the firm has protected information.
What can the firm do? Rule 1.10(b) says it depends on whether W has any confidential
information. Comment [5] to rule 1.10 says that when X leaves he takes this taint with him and
unless W has confidential information, it is going to be ok. (This is the flip side to rule 1.9(b)).
Look at the lawyer who is leaving: are we inquiring what the lawyer can do who leaves and goes
to a new firm (1.9), or are we inquiring as to what the law firm the lawyer left behind can do
(1.10)?
D. The Use of "Screening" To Avoid Imputation
1. Does Rule 1.10 recognize the availability of screening to avoid imputation in the
case of lawyers like X who have moved from one private firm to another? Rule 1.0(k) states that
"screened" denotes the isolation of a lawyer from any participation in a matter through the timely
imposition of procedures w/in a firm that are reasonably adequate under the circumstances to
protect information that the isolated lawyer is obligated to protect. However, rule 1.10 does not
provide for screening in the private firm setting.
2. A number of cases have now said, in dictum or otherwise, that screening of
lawyers is permissible.

Rule 1.18
IF the client left confidential information with you,
Paragraph D allows you to do something different. Both the effective client and prospective client
give informed consent in writing. IF u can not get informed consent, the lawyer who received
confidential information.

H. Problem 16

Special Problems of Government Lawyers


(Page 260)

Rule 1.11

Government ---> Private 1.11(a)(b)(c) and 1.10 (comment 7)


Private ----> Government 1.11 (d)(2)(i) and 1.10 (comment 7)

Government Lawyer negotiate for job 1.11(d)(2)(ii)

4.2 Represented Persons 4.3 Unrepresented Persons


- Lawyer shall not communicate - If lawyer knows person is
with person lawyer knows is unrepresented, lawyer shall not
represented, unless other lawyer state or imply he is
consents or is authorzed by law disinterested.
or a court order.
- Protects person who has chosen - Lawyer shall clarify his role if
to be represented (see comment lawyer believes unrepresented
1) person misunderstands his role
- Applies even if represented -
person initiates or consents to
the contact (see comment 3)
- Lawter may communicate with
person represented w/matters
outside scope of representation
-

-
Government Lawyer subject to 1.7/1.9 1.11(d)(i)

A. The Rules Applicable to Former Government Lawyers


3. Rule 1.9 only prohibits representation that is materially adverse to the former
client. Is rule 1.11 so limited? How about 18 U.S.C. § 207? (Rule 207 just limits the amount of
information that a government official can reveal; it is a criminal statute that sets forth the periods
of time during which a government official cannot reveal any information).
4. Is Smithers barred from all activity w/in his own law firm in the case involving
Quick? Can he talk to Quick about what the FTC is likely to do? Rule 1.11(a) states that except
as law may otherwise expressly permit, a lawyer who has formerly served as a public officer or
employee of the government: (1) is subject to rule 1.9(c) (cannot reveal confidential information);
and (2) shall not otherwise represent a client in connection w/a matter (see rule 1.11(e) as to
definition of "matter") in which the lawyer participated personally and substantially as a public
officer or employee, unless the appropriate government agency gives its informed consent,
confirmed in writing, to the representation (except that a lawyer serving as a law clerk to a judge,
other adjudicative officer or arbitrator, may negotiate for private employment as permitted by
1.12(b). 1.12(b) states that a lawyer serving as a law clerk to a judge may negotiate for
employment w/a party or lawyer involved in a matter in which the clerk is participating personally
and substantially, but only after the lawyer has notified the judge or other adjudicative officer).
Thus, Smithers needs the informed consent of the FTC to be able to work on the
Quick case. It is likely that the FTC will not consent, and thus 1.11(b) kicks in, which states that
when a lawyer is disqualified from representation under (a), no lawyer in a firm w/which that
lawyer is associated may knowingly undertake or continue representation in such a matter
unless: (1) the disqualified lawyer is timely screened from any participation in the matter and is
apportioned no part of the fee therefrom; and (2) written notice is promptly given to the
appropriate government agency to enable it to ascertain compliance with this rule. However,
under comment 6, a lawyer is not prohibited from receiving a salary or partnership share
established by prior independent agreement, but that lawyer may not receive compensation
directly relating the lawyer's compensation to the fee in the matter in which the lawyer is
disqualified.
Comment 7 states that notice, including a description of the screened lawyer's
prior representation and of the screening procedures employed, generally should be given as
soon as practicable after the need for screening becomes apparent. Remember in this case,
however, that Quick only came to the firm because of Smithers, and now Smithers cannot do
anything to help Quick. Can Smithers at least take Quick out to lunch? Probably not, but he can
be cordial and say hello when he sees Quick in the firm.
Note that (a) and (b) do not prohibit representation based on adverse interests.
Rule 1.11(c), however, states that a lawyer having information that the lawyer knows is
confidential government information about a person acquired when the lawyer
A way around this is to screen the lawyer and have someone else in the firm do
it.
Comment 7 to rule 1.10 specifically states that where a lawyer has joined a
private firm after having represented the government, imputation is governed by 11.(b) and (c),
not 1.10. Under 1.11(d), where a lawyer represents the government after having served clients in
private practice, non-governmental employment, or in another government agency, former-client
conflicts are not imputed to government lawyers associated w/the individually disqualified lawyer
(if it were then the IRS could never get any work done). Rule 11.1(d)(2)(i) states that a lawyer
currently serving as a public officer or employee shall not participate in a matter in which the
lawyer participated personally and substantially while in private practice or non-governmental
employment, unless the appropriate government agency gives its informed consent, confirmed in
writing.
C. Negotiating for Post-Government Employment While Still in Government
1. Whether Smithers did anything wrong when he was negotiating for employment
prior to his retirement from the FTC: Rule 1.11(d)(2)(ii) states that a lawyer currently serving as a
public officer or employee shall not negotiate for private employment with any person who is
involved as a party or as a lawyer for a party in a matter in which the lawyer is participating
personally and substantially.
Here, we must decide Smithers' involvement and whether he improperly
negotiated for a private job in violation of rule 1.11(d)(2)(ii). Rule 1.11(e) defines "matter" as any
judicial or other proceeding, application, request for a ruling or other determination, contract,
claim, controversy, investigation, charge, accusation, arrest or other particular matter involving a
specific party or parties, and any other matter covered by the conflict of interest rules of the
appropriate government agency. (Remember, "matter" can relate to either litigation or
transactional).

V. Advising Clients

A. Problem 17

The Lawyer For an Individual Client


(page 276)

Facts
Mrs. Anderson, client, comes to you and complains welfare dept has taken her children away.
Investigation shows neglect, malnourishment, and abuse on two kids, Mary, 7, and Billy, 3. Social
workers at school became suspicious with Mary's bruises and malnourishment after several days
of absence. Mary said mom sometimes hit them and sent them to bed hungry. She also left
them for hours with no adults to care for them. Mrs. Anderson says husband John is abusive and
she often leaves home in fear of her life. John has job but is paid in cash and so cannot be
reliable. She wants her mother to live with her but husband says no and gets violent.

A. The Difference Between Advice and Advocacy


1. Whether the lawyer's job is only to return the client's children, or is she seeking
experience and judgment about how to deal with her situation? Rule 2.1 states that in
representing a client, a lawyer shall exercise independent professional judgment and render
candid advice. In rendering advice, a lawyer may refer to other considerations such as moral,
economic, social, and political factors. Comment 5 says that in general, a lawyer is not expected
to give advice until asked by the client. However, when a lawyer knows that a client proposes a
course of action that is likely to result in substantial adverse legal consequences to the client, the
lawyer's duty to the client under 1.4 may require that the lawyer offer advice if the client's course
of action is related to the representation.
Under 1.16, you could withdraw, but that is not always the best solution, b/c the
lawyer's business will suffer.

B. Limits on the Advice a Lawyer May Give


1. Rule 1.2(d) states that a lawyer shall not counsel a client to engage, or assist a
client, in conduct that the lawyer knows is criminal or fraudulent, but a lawyer may discuss the
legal consequences of any proposed course of conduct w/a client and may counsel or assist a
client to make a good faith effort to determine the validity, scope, meaning, or application of the
law. Comment 9 states that this prohibition does not preclude a lawyer from giving an honest
opinion about the actual consequences that appear likely to result from a client's conduct, nor
does the fact that a client uses advice in a course of action that is criminal or fraudulent of itself
make a lawyer a party to the course of action. These rules make a huge distinction between past
conduct and present or ongoing conduct.
Thus, would it be criminal or fraudulent for the attorney to help Mrs. Anderson
regain custody of her children? Well, there exists a duty to report when child abuse is present. If
the attorney believes that the client is committing the crime of child abuse, she cannot assist her
by fighting for her to get custody. Under 1.6, can she reveal that her client abuses her children?
2. Is it obvious what it means to "counsel or assist" wrongdoing? Under comment
9, there is a critical distinction between presenting an analysis of legal aspects of questionable
conduct and recommending the means by which a crime or fraud might be committed with
impunity. What if a client charged with murder asks you which South American countries have no
extradition treaties with the United States? The lawyer, under comment 9, may be able to tell the
client which countries do not have extradition treaties with the U.S. It might actually be beneficial
b/c the lawyer might be able to talk the client out of that course of action. If, on the other hand,
the lawyer recommends a particular country to the client, she may have crossed the line.
3. What if Mrs. Anderson's mother owns her own house and a few stocks but has
no other source of income. If she were to become ill, the house could be seized to reimburse
Medicaid for her care. Would it be wrong for the attorney to encourage Mrs. Anderson's mother
to give the house to Mrs. Anderson to avoid Medicaid being able to take it? If it is a crime to do
this, then the lawyer might have a 1.2(d) problem, because she is affirmatively assisting the client
in fraud.
4. Would you be more uncomfortable, or less, about helping a wealthy client buy an
expensive house in Florida or set up a trust in the Islands that would exempt those assets from
being seized by creditors if the client ever went bankrupt? The purpose is important here, even
though the client can move to Florida anytime he wants. If his purpose is to evade creditors, we
may have a 1.2(d) problem, but on the other hand, the laws in Florida were expressly set up to
protect a person's homestead from creditors. If it’s a tax planning thing and it is a loophole but
still legal – it’s ok.

C. The Client Suffering from Diminished Capacity


1. Rule 1.14 states that when a client's capacity to make adequately considered
decisions in connection w/a representation is diminished, whether b/c of minority, mental
impairment, or for some other reason, the lawyer shall, as far as reasonably possible, maintain a
normal client-lawyer relationship with the client. The lawyer may also, under subsection (b) seek
to take action to protect the client, such as by seeking appointment of a guardian ad litem,
conservator, or guardian. Note that under subsection (c), information relating to the
representation of a client w/diminished capacity is protected by 1.6. When taking protective
action, the lawyer is impliedly authorized under 1.6(a) to reveal information about the client, but
only to the extent reasonably necessary to protect the client's interests. Comment 6 states that in
determining the extent of the client's diminished capacity, the lawyer should consider and balance
such factors as: the client's ability to articulate reasoning leading to a decision, variability of state
of mind and ability to appreciate consequences of a decision; substantive fairness of a decision;
and the consistency of a decision w/the known long-term commitments and values of a client.
1.6 – you may reveal info when you are impliedly authorized. May not be able to get consent to
reveal confidential information because of their diminished capacity.  Take protective action.

B. Problem 18

Advising the Business Corporation


(Page 289)
Facts
Outside counsel to Sleepwear that makes children's pj's which are flammable and VP is aware of
this. VP says experts say that only 1/50K children would hold the matches to the PJs for a fire to
start. Management consulting firm said civil damages couldn't exceed 250,000 per victim. It is
more profitable to sell the pajamas and pay for any damages than not to sell them. You haven't
been asked for advice on whether to keep marketing them and VP is angry that you know and
has threatened your job.

A. The Client to Whom a Corporate Lawyer Owes Primary Loyalty


1. The client here is the corporation, not the corporation's VP. Although the VP is an
officer of the corporation and therefore the corporation acts through him (see comment 1 to 1.13,
which states that a organizational client is a legal entity, but it cannot act except through its
officers, etc.), if he wants to take a course of action that is not in the best interest of the
corporation, the attorney's duty is to the corporation, not to him.

B. Reporting Corporate Misconduct Within and Outside the Client Organization


1. What steps are you required to take in this situation? When does a lawyer's obligation
shift from taking orders to taking action? The attorney here is representing the corporation in an
unrelated matter, and the VP has told him to butt out. Should he just butt out? Rule 2.1,
comment 5 states that a lawyer is not expected to give advice unless asked for it; no duty to
investigate affairs but do it when it best interest of client. Additionally, comment 6 to rule 1.13
states that the authority and responsibility provided in rule 1.13 are concurrent w/the authority and
responsibility provided in other rules. Thus, this rule does not limit or expand the lawyer's
responsibility under rules 1.8, 1.16, 3.3, or 4.1. 1.13(c) supplements 1.6(b) by providing an
additional basis upon which the lawyer may reveal information relating to the representation. The
lawyer may reveal such information only where the organization's highest authority insists upon
or fails to address threatened or ongoing action that is clearly a violation of law (which appears to
be the case here w/the VP), and then only to the extent the lawyer reasonably believes necessary
to prevent reasonably certain substantial injury to the organization. It is not necessary that the
lawyer's services be used in furtherance of the violation (which it is not here, b/c the lawyer's
representation is unrelated to the pajamas), but it is required that the matter be related to the
lawyer's representation of the organization. This is a "scope" issue. There is a broad scope of
lawyer's employment; this rule is trying to protect the lawyer who learns something just because
he is a friend (like lawyer and VP on golf course when lawyer isn't representing co.)
Rule 1.13(b) is a two part test: the officer's engagement in action, intention to
act, or refusal to act must be (1) a violation of a legal obligation to the organization, or a violation
of law that reasonably might be imputed to the organization, and (2) likely to result in substantial
injury to the organization. At that point, the lawyer shall proceed as is reasonably necessary in
the best interest of the organization. Comment 4 states that the lawyer should give due
consideration to the seriousness of the violation and its consequences, the responsibility in the
organization and the apparent motivation of the person involved, the policies of the organization
concerning such maters, etc. If a constituent persist in conduct contrary to the lawyer's advice, it
will be necessary for the lawyer to take steps to have the matter reviewed by a higher authority in
the organization. Any measures should be taken to the extent practicable to minimize the risk of
revealing information relating to the representation to persons outside the organization.
Exception in 1.13(b): the lawyer does not have to proceed if the lawyer
reasonably believes that it is not necessary in the best interest to do so. Otherwise, the lawyer
shall refer the matter to a higher authority in the organization, including, if warranted by the
circumstances, to the highest authority that can act on behalf of the organization as determined
by applicable law. (Similar to Sarbanes Oxley)
Here, the lawyer has tried to speak to the VP, who has refused to take action.
Thus, the lawyer should continue above the VP to other authorities that can act.
1.13(c) states that if, despite the lawyer's efforts, the highest authority that can
act on behalf of the organization insists upon or fails to address an action or a refusal to act, that
is clearly in violation of law, and the lawyer reasonably believes that the violation is reasonably
certain to result in substantial injury to the organization, then the lawyer may reveal information
relation to the representation, whether or not 1.6 permits such disclosure, but only if and to the
extent the lawyer reasonably believes necessary to prevent sub
stantial injury to the organization.
1.13(d) states that the foregoing shall not apply w/respect to info relating to a
lawyer's representation of an organization to investigate an alleged violation of law, or to defend
the organization or an officer, employee, or other constituent associated w/the organization
against a claim arising out of an alleged violation of law. Cannot reveal information relating
specifically to what the lawyer was hired to do; a client is entitled to a defense w/o their
confidential information being "spilled." Comment 7 states that this rule is necessary to allow
organizational clients to enjoy the full benefits of legal counsel in conducting an investigation or
defending
against a claim.

C. Lawyer Liability for Failure To Take Appropriate Action


Since rule 1.13(c) states that a lawyer may reveal information relating to the
representation of the corporation, there is really no excuse for not doing so. If the lawyer does
not reveal information, he will not be disciplined under these rules, because the rule is
discretionary (uses the word "may").

• Overview of Rule 1.13


o Application to lawyers and to organizations
 Applies to outside counsel and in house counsel
• See RPC 1.13(a)
 Applies to all organizations (corporations, partnerships, labor unions,
governmental agencies, etc.)
• See comment [1]s reference to unincorporated associations
• See comment [9]s reference to governmental organizations
o Who is the client and the consequences of who is the client
 Client is organization and not its constituients (such as officers, employees,
shareholders, partners)
 Lawyer may separately or additionally represent a constituent (such as an
officer or an employee) but doing so may raise RPC 1.7 issues
 Lawyer must explain that client is organization and not constituent when
speaking with constituent if she knows or reasonably should know that
organiation and constituent have adverse interest
o Obligation to upwardly report – RPC 1.13(b)
 If
• A lawyer knows that a
• Person associated with the organization
• Is engaged in action, intendeds to act, or refuses to act in a
• Matter related to the representation
• That is either
o A violation of a legal obligation to the organization OR
o A violation of law that might reaonably be imputed to the
organization
• And that is likely to result in substantial injury to the organization
 Then
• The lawyer shall proceed as is reasonably necessary in the best
interest of the organization
o Speak with misbehaving person if this is reasonable
o Speak with person at next level of organizations hierarchy if
necessary
o Speak with person or group at highest level of organization’s
hierarchy if necessary.
o Discretion to ouwarly Report – RPC 1.13(c)
 If
• The highest authority in an organiation
• Insists upon or fails to address in a timely and appropriate manner
• A perons’s action or refusal to act
• That is clearly s violation of law and
• That is reasonably certain to result in a substantial injuury to the
organization
 Then
• The lawyer may reveal information relating to the representation of
the organization to an outsider
o Only so long as the lawyer reasonably believes doing so is
necessary to prevent substantial injury to the organization
o And then only to the extent the lawyer reasonably believes
necessary to prevent injury to the organization
• Rules 1.13(C) and 1.6(B0(1)-(3)
o Difference in application
 RPC 1.13(c) and RPC 1.6(b)(1)-(3) are separate, stand-alone rules that
must be viewed as such
 Lawyer may reveal information when the requirements of one rule are
satisfied without regard to whether the requirement of the other rule are also
satisfied.
• Information can be revealed under
o Difference in Intended beneficiary
 An RPC 1.13(c) disclosure seeks to protect the client from misbehaving
constituents
 An RPC 1.6(b)(1), (b)(2), or (b)(3) disclosure seeks to protect a third
person from a misbehaving client
o Similarity in non-obligation
 RPC
• Big picture questions
o What is the assumption underlying upward reporting? Eventually the higher you go
there is someone who does not have a stake in the misbehavior, and who will take
action to resolve it. A borard will normally not have any stake in the wrongdoing.
o Why is obligation to make an upward report limited to situations in which the wrong
doing is related to the work the lawyer is doing for the organization? No reason to
report personal mibehavior which has nothing to do with the organization. We limit a
lawyers obligation report misbehavior to the matters which he is working on because
he is uniquely knowledgeable in that are and b/c he is a credibe person to come
forward in that matter.
o Can a lawyer make an upward report when the lawyer knows of wrongdoing that is
unrelated to the work she is doig for the organization? Nothing in the rule that
specifically tells you that you cant make the upward report of misbehavior which
occurs in area unrelated to the work she is doing for the organization. So you can use
your discretion if ou want to.
o Why is outward reporting limited to situations in which there is a clear violation of
law? b/c of the damage to the organization an internal report can be handled by
insiders once there is an outside report the information is out in the public.
• Variation 1
• While working on an unrelated matter, you learn that sleepware tests confirm that its
children’s pajamas can ignite when matches are held against them for a few seconds. You
know that federal regualtions prohibit the sale of products known to cause burns to children.
o Does thsi variation contain a fact that eliminates any obligation to do anything under
RPC 1.13(b)? there is no obligation under (b) because the fact pattern says you are
not working on this matter but on a unrelated one.
o Does the absence of an obligation preclude you from taking action? You can still
make a report if you think appropriate but you are not required to make that report.
• Variation 2
• You decide to speak with the vice president responsible for the manufacture and sale of
childrens pajamas to learn more about the situation.
o From a textual persepective does RPC 1.13(b) permit you to approach the person
who may be the source of the problem or does it require you to approach a higher
authority in the first instance? You can go to the person who may be the source of
the problem in the first instance RPC 1.13(b)(2nd sentence) and [Comment 4] can be
properly constued in this fashion because it says unless the lawyer believes it is not
in the best interest in the organization to do so. (if you can fix the problem by going to
the source of the problem first and it is reasonable then you can do that)
o Does RPC 1.13(F) and comment [10] suggest how you should begin your discussion
with the vice president? In this circumstance you necessarily know the source of the
problem may have interest diverse from the organization of the enitity, and as the
organizations lawyer you should begin the discussion by disclosing 3 things (1) you
represent the organization not the individual; (2) tell the individual he/she may want to
get their own outside counsel; (3) tell the individual the organization controls the
attorney/client privilege.
 I represent the organization and not the individual
 You may want to get their own outside counsel
 The organization, and not you, controls the attorney/client privelege
o If the vice president agrees there is a problem with the pajamas and to correct it, then
is there any obligation to do anything under RPC 1.13(b)? no, if you correct the
problem there is no need to go any higher.
10/18/10

• Variation 3
• The vice president tells you he does not believe regulators will recognize the problem. He
tells you experts believe only 1 in 50, 000 children will suffer burns; that management
consultants calculate likely damages at 250,000 per burn victim; that annual profit is $1
million even after payment of $1 million in civil damages each year; and disclosure might
impact his chances of succeeding the president.
o Is the Vice President violating a duty to the organization, or a law, or perhaps
both? Both, there is a regualtion which may prohibit the sale of pajamas which
are not fire retardent and if the vice president keeps selling them there could be a
violation of the law; the vice president placing his interest ahead of the
organizations better interest is violating the fiduciary duty of loyalty of the
organization.
o Are you required to accept the vice president’s representations at face value? No
you do not have to accept them. Comment 3 says normally it is not your job to
second guess the vice president or the client in regards to business risks, but this
is different because this is not just business risks.
 Has the vice president missed a significant type of potential injury to the
organization? Yes, damage to the organization/companys reputation
 Has the vice president received a reasonable assessment of recoverable
damages? No a lawyer is more knowledgeable and more uniquely
qualified to judge damages in this kind of matter than would be a
management consultant since this is what lawyers do for a living.
o Are you limited to giving the vice president legal advice about the scope and
effect of the regulation or does RPC 2.1 allow you to have a broader discussion
with the vice president? There is nothing that prohibits the lawyer from discussing
all of the relevant factors. RPC 2.1 also says a lawyer can weigh in on moral,
economic, social, and political factors that may be relevant to the client’s
situation.
o Do you believe you should report what you have learned to a higher authority in
the organization? Yes, under 1.13(b) you are required to do so. If the president
looks past the issue and doesnt want you to say anything, because the president
would like the vice president to succeed her, then the President may place more
interest in seeing the vice president in succeed her then the very best interest in
the organization so since there is no reasonable response by the president you
must work your way higher up in authority.
• Variation 5
• You take the issue to the board of directors. They listen to you and then side with the
vice-president and president.
o What do you do, if anything after your receive the boards answer?
 You can consdier going public to protect the corporation from itself.
• Variation 6
• You consider going public with what you know.
o Are you required ti rethink your positiong or is your earlier analysis sufficient to
support a decisions to go public? Yes
 Is there is a difference between the conditions that must exist for you to
report up that there ones that must exist for you to report out. yes
• There must be a clear violationg of law
• That is reasonably certain to reult in injury to the corporation (as
opposed to inward reporting needing only “likely to reult in harm”)
 Is there any limitation on what you can say (or to whom you can say it?)
can only disclose if and then only to the extent you reasonably believe
necessary to protect the organization from substantial injury either in
whole or in part.
• Variation 7
• You rethink your position. You accept the expert’s conclusion that four children might be
severley injured each year but you conclude that the applicable regulations are unclear.
As such you cannot say that the sale of the clildren’s pjamas consititutes a clear violation
of law.
o Can you disclose what you know to an appropriate authoirty pursuant to rule
1.13(c)? no because there is no clear violation of law.
o Is there another rule that you might invoke if you felt strongly about disclosing
what you know to an apporpriate authority? 1.6(b) specifically (1)
• Variation 8
• Assume an altogether different scenario. You know about the pajama problem and you
take the issue to the corporation’s president. Much to your suprise she fires you for
“rocking the boat.”
o What does rule 1.13(e) require you to do in this circumstance? If you reasoanbly
beleive you have been terminated for the representation because of you wanting
to report upward, your are required to inform the highre up (the board) of your
firing.
• Testing traps for the unwary
o Client is organization and not the constituents but
 Communications between the organization’s lawyer and a constituent
are protects from discovery by attorney-client privilege but
 Organization controls privilege and can waive it without regard to
whether constituent agrees
o Obligation/duty to make upward report attaches only when the misconduct is
related to representation.
o Conditions for upward report less stringent than conditions for outward
disclosure.
 Violation of law vs. clear violation of law
 Likely to result in substantial injury vs. reasonably certain to result in
substantial injury
o RPC 1.13(c) is not equal to RPC 1.6(b)(1)-(3) (must take each rule into
consideration separately)
o Lawyer hired to defend organization or constituent, or to conduct an internal
investigation or wrongdoing, cannot make outward disclosure under RPC 1.13(c)

RPC 4.1(a) Questions


• What is the basic obligation imposed by RPC 4.1(a)?
o Does RPC 4.1(a) prohibit a lawyer from telling a lie to her client? No (doesnt
apply to a client, only applies to 3rd parties)
o Does it prohibit a lawyer from telling a lie to a friend at a social function? No it
doesnt apply to a lawyers personal life (applies only when in representing a
client)
• Does RPC 4.1(a) require a lawyer to disclose facts about which she is aware but which
her adversary appears to be unaware? No there is no affirmative obligation (may be a
discovery obligation but ethically speaking there is no obligation see comment 1 RPC
4.1)
o True or false – one exception exists when the client dies during the course of the
representation ex. if a lawyer represents a client in a personal injury action the
lawyer must inform the adversary. So if a client dies then the lawyer must inform
the adversary. If a client dies then the lawyer must get a new client in order
continue, and therfore the opposing counsel should be informed.
• Does RPC 4.1(a) prohibit a lawyer from puffing and posturing about a client’s willingness
to settle, or the amount at which she is willing to settle, during negotiations? No,
[comment 2] is not a material fact for 4.1(a).
RPC 4.2 Questions
• What is the basic obligation imposed by RPC 4.2? when representing a cleint in a matter
they cannot have ex parte discussions with persons represented in the matter by some
other lawyer.
o Does RPC 4.2 prohibit ex parte communication with represented persons or
represented parties? Persons
o Does RPC 4.2 prohibit ex parte communication with respect to all subjects or
some subjects? Just the subject of the representation, not all subjects.
o Does RPC 4.2 prohibit ex parte communication with persons who are
represented by another lawyer or persons who are represented in the matter by
another lawyer? Those who are represented in the matter.
• Who is off limits when the represented person is a corporation? [comment 7]
o Current employees who are either:
 (1) Consituents/individuals who direct, supervise, or regualrly
communicate with the organizations lawyer about the matter.
 (2) Those individuals who have the authority to bind the organization with
respect to the matter. (typically the president or someone who can say
yes we will settle and if yes, how much we will settle for.)
 (3) Individuals whose acts or ommissions may be imputed to the
organization for the purposes of establishing the organiations liability in
the matter. (the person within the scope of their employment which may
have been wrongful) (actions or inactions which expose the
organizations liability)
o Why are former employees typically not off limits?
 Because they will typically not be people who fulfill any of the three
things above. (1)-(3).
RPC 4.3 Questions
 A lawyer must always tell an unrepresented person that she is a lawyer before she can
interact with the unrepresented person? No, there is nothing in the rules that say you
must do this.
 A lawyer must always identify her client before she can interact with an unrepresented
person? No not always
 What must a lawyer do and what must a lawyer not do when interacting with an
unrepresented person?
o The lawyer must not state or imply that she is disinterested
o The lawyer must correct misunderstadnings about the lawyers role in the
situation the lawyer must take reasonable steps in correcting the
misunderstanding in the role the lawyer plays.
o The lawyer must not give legal advice, except for “you should get a lawyer,”
when there is a reasonable possibility of conflict bewtween the interests of the
client and the interest of the unrepresented client and the lawyers

C. Problem 19

Contact With Represented and Unrepresented Persons


(Page 302)

Lawyers frequently contact others on behalf of clients. Typically, the contacts are to obtain or
deliver information. These contacts are regulated, however, due to concerns that lawyers might
mislead an opposing party or interfere w/that party's relationship w/its own lawyer.

Facts
Speedy Corp. promises to deliver packages within an hour which makes drivers take dangerous
chances. President Mary stood with accountant Barry Winters when Speedy Truck hit a
pedestrian elderly man causing severe injuries. Employees of the company witnessed the
accident. Attorney Shabazz, representing the elderly man, interviewed Barry and truck driver
without telling Speedy's lawyer Barbara. Shabazz then contacted Mary for an interview and
Barbara told Mary to decline unless in formal deposition. Barbra wants to investigate to find out if
Speedy would be criminally negligent and Shabazz of course would like to talk to the same
people.

A. Contact With a Represented Opponent In General


1. Rule 4.2 states that a lawyer shall not communicate about the subject of the
representation w/a person the lawyer knows to be represented by another lawyer in the matter,
unless the lawyer has the consent of the other lawyer or is authorized to do so by law or a court
order. Thus, the lawyer must communicate with the opposing party's attorney.
3. This rule protects one's client, b/c an attorney can never be sure of what his/her client
will say to opposing counsel. This rule eliminates those concerns. Comment 8 states that the
prohibition on communications w/a represented person only applies in circumstances where the
lawyer knows that the person is in fact represented in the matter discussed. Must be actual
knowledge, but such actual knowledge may be inferred from the circumstances. Thus, the lawyer
cannot evade the requirement of obtaining the consent of counsel by closing eyes to the obvious.
4. May represented clients talk to each other w/o going through their lawyers? Yes.
Under comment 4, parties to a matter may communicate directly w/each other, and a lawyer is
not prohibited from advising a client concerning a communication that the client is legally entitled
to make.
However, this may potentially be problematic, b/c it is possible for the lawyer to
circumvent the rule by "speaking" to the other client through his own client. Comment 4 also
states that a lawyer may not make a communication prohibited by this rule through the acts of
another under rule 8.4(a). So, for example, the lawyer cannot get his secretary to communicate
w/opposing counsel's client at his direction and then report back to the attorney w/regard to what
the other client said. Note also that under 4.3, a lawyer is prohibited from interacting with persons
who are not represented at all.

B. Special Issues When Dealing w/Officers and Employees of an Organization


1. Whether lawyer has a right to interview the employees of company responsible for the
accident: In this case, there may be corporate counsel. Here, we have the company accountant,
the truck driver, and the president of the corporation. Comment 7 states that in the case of a
represented organization, this rule prohibits communications w/a constituent of the organization
who supervises, directs, or regularly consults w/the organization's lawyer concerning the matter
of has authority to obligate the organization w/respect to the matter or whose act or omission in
connection w/the matter may be imputed to the organization for purposes of civil or criminal
liability.
Thus, it is obvious that the lawyer cannot speak to Speedy (the president of the
corporation), b/c she supervises, directs, etc. The lawyer also cannot speak to the truck driver,
b/c his acts could impute criminal or civil liability to the corporation, and thus contact w/him would
not be proper under comment 7 either. With regard to the accountant, it seems that the lawyer
can speak to him, but is limited by comment 7 in methods of obtaining evidence. He cannot use
methods that violate the legal rights of the organization under rule 4.4.
Restatement third § 100(2) is very similar to the model rule. It says that opposing
counsel may not contact current corporate employees and agents without the other lawyer's
permission in three situations: (a) if the employee or other agent supervises, directs, or regularly
consults w/the lawyer concerning the matter or if the agent has power to compromise or settle the
matter; (b) if the acts or omissions of the employee or other agent may be imputed tot he
organization for purposes of civil or criminal liability in the matter; or (c) if a statement of the
employee or other agent, under applicable rules of evidence, would have the effect of binding the
organization w/respect to proof of the matter.
2. Whether the prohibition against contacting corporate employees should apply to
former employees: comment 7 states that consent of the organization's lawyer is not required for
communication w/a former constituent. If a constituent is represented in the matter by his/her
own counsel, the consent by that counsel to a communication will be sufficient. If the former
constituent is not represented, then 4.3 will kick in, b/c the lawyer is limited on communicating
with a person who is unrepresented.

C. Interviewing Client Employees and Other Unrepresented Persons


2. Should the defense attorney (for the corporation) take a "we're all in this
together" approach? She could, under rule 1.13(f) and (g). Under (f), in dealing w/an
organization's directors, etc., a lawyer shall explain the identity of the client when the lawyer
knows or reasonably should know that the organization's interests are adverse to those of the
constituents w/whom the lawyer is dealing. (G) states that a lawyer representing an organization
may also represent any of its directors, officers, employees, etc. If the organization's consent to
the dual representation is required by 1.7 (conflicts of interest), the consent shall be given by an
appropriate official of the organization other than the individual who is to be represented, or by
the shareholders.
However, comment 10 points out that there are times when the organization's
interest may be or become adverse to those of one or more of its constituents. Here, Speedy is
worried about defending the speeding policy to make its deliveries on time, and its best defense
might be to dump the liability on the truck driver and blame him for speeding. In this case, the
lawyer should advise any constituent (here, the truck driver), whose interest the lawyer finds
adverse to that of the organization of the conflict or potential conflict of interest, that the lawyer
cannot represent such constituent, and that such person may whish to obtain independent
representation. The lawyer will need to get informed consent under 1.7 from both affected parties
to represent them both, and if the lawyer does not get it, must recommend separate counsel,
especially if the other constituent would otherwise be unrepresented.

4. Whether there are limits on how opposing counsel may conduct his interviews of
the eyewitness, who are neither represented nor treated as represented by corporate counsel:
Can he take them to lunch? It depends. It doesn't look right, but it also is not prohibited by law.
The closest rule is 3.4(b), which states that a lawyer shall not counsel or assist a witness to testify
falsely or offer an inducement to a witness that is prohibited by law. This rule applies most
directly to actual bribes, however, which it does not appear is the case here. Still, it is
unprofessional for opposing counsel to take witnesses to lunch who are employed by the
opposing party.

D. Problem 20

The Ethics of Negotiation


(Page 318)

Facts
Young, 19, was at fault in a car accident with a person who suffered 25K in injuries and damages.
Young lied to cop and said he didn't drink but really had 3. You have plea negotiations with the
prosecutor and settlement discussions for potential criminal and civil claims which Young hasn't
discussed with you but insurance company gave you authority to settle for 20K in civil damages.

A. Authority to Participate in and Consummate Negotiations


1. There is no rule directly on point about negotiation. However, rule 1.2(a) gives
an attorney the authority to do so. It states that a lawyer shall abide by a client's decisions
concerning the objectives of representation, and under 1.4(b), a lawyer shall consult w/the client
as to the means by which they are pursued.

4. Whether a lawyer should be required to tell the client about all settlement offers
received from the other side: Yes! Rule 1.4, comment 2 states that a lawyer must promptly
consult with and secure the client's consent prior to taking action unless prior discussion w/the
client have resolved what action the client wants the lawyer to take. For example, a lawyer who
receives from opposing counsel an offer of settlement in a civil controversy or a proffered plea
bargain in a criminal case must promptly inform the client of its substance unless the client has
previously indicated that the proposal will be acceptable or unacceptable, or has authorized the
lawyer to accept or to reject the offer.

5. Whether the client should be bound by a settlement that the lawyer was not authorized
by the client to make (i.e. whether a lawyer may assume that the lawyer for the other side has
authority to make the settlement being proposed): NO. This goes back to agency law. The client
is not going to be bound, and the lawyer may face malpractice. Additionally, a lawyer who
affirmatively misrepresents his or her settlement authority is subject to liability for this
misrepresentation.

B. The Duty of Honesty in Negotiations


Note: Rule 4.1 is tied to Rule 8.4(c)
1. Whether the attorney may assert that his client had nothing alcoholic to drink on the
day of the accident: no. This question is narrow and specific. To state that the client had nothing
alcoholic at all to drink would be a lie. [Remember from evidence, however, that statements
made during negotiations or in a negotiation setting is inadmissible in court. This is to encourage
negotiation and the settlement of cases]. If the negotiations were to fail and the issue was to go
to trial, the lawyer could not then continue the lie and encourage the client to testify in depositions
and at trial that he had nothing at all to drink. Such would be a blatant violation of the rules 4.1
and 8.4, b/c it would be a material misrepresentation of fact. The best answer at negotiations
would perhaps be a question, "What proof do you have that my client was drinking?"
Comment 2 to Rule 4.1 states that under generally accepted conventions in negotiation,
certain types of statements ordinarily are not taken as statements of material fact. Estimates of
price or value placed on the subject of a transaction and a party's intentions as to an acceptable
settlement of a claim (e.g. "My client won't take a dime under $5000!" when the lawyer knows that
the client would take $4000) are ordinarily in this category, and so is the existence of an
undisclosed principal except where non-disclosure of the principal would constitute fraud.
Comment 1 also states, however, that a lawyer is required to be truthful when dealing with others
on a client's behalf, but generally has no affirmative duty to inform an opposing party of relevant
facts.
2. Are all lies created equal? No. Some lies will be tolerated (see comment 2 above),
while others are forbidden. It is ok, for example, to fake anger at a negotiation, mostly b/c it
would be impossible to monitor and sanction that type of behavior.

C. The Duty to Volunteer Information or Correct a Misapprehension


1. Whether a lawyer should ever be required to disclose facts affirmatively in a
negotiation: No. See comment 1 to rule 4.1. That does not mean, however, that a
misrepresentation cannot occur for which the lawyer may be disciplined. (E.g. A lawyer cannot
fail to disclose to the opposing party that his client is no longer living. Even though comment 1
states that a lawyer does not have an affirmative duty to disclose relevant facts to the opposing
party, in this case, failing to do so constituted a misrepresentation of material fact and the lawyer
was held in violation of 4.1).
4. Whether the rules about negotiation should be different in criminal cases: criminal law
is different because the defendant's liberty is at stake. How candid should a prosecutor have to
be about the strength of the evidence? Rule 3.8 governs the special responsibilities of a
prosecutor, and subsection (d) states that the prosecutor shall make timely disclosure to the
defense of all evidence or information known to the prosecutor that tends to negate the guilt of
the accused or mitigates the offense.
5. Whether in the plea-bargaining negotiations w/the prosecutor, a criminal defense
counsel may assert that he/she "knows" the client is innocent: technically, rule 4.1 does not
make exceptions based on the motivation for lying. However, there is a justification for it because
the defendant's liberty is at stake.
There seems to be a difference between lying to deceive someone into changing their
position and lying to see how they would honestly react to a set of facts. However, some courts
state that there are no exceptions to rule [4.1] against misrepresentations and that the prohibition
applies to all lawyers in all cases.

D. Possible Limits on the Results that Can Be Reached in Negotiations


1. May a lawyer agree that in exchange for a generous financial payment to the lawyer's
client, neither the lawyer nor the client will file criminal charges against the opposing lawyer for
conduct would justify the charges? The Model Rules are silent; however, the ABA issued a
formal opinion (page 332) stating that a lawyer may use the possibility of bringing criminal
charges in negotiations in a civil case if both the civil case and criminal violation are well-founded
in fact and law, the lawyer does not suggest improper influence over the criminal process, and the
threat would not constitute extortion under the law. The lawyer may even agree not to file
criminal charges as an element of settling a civil claim if that agreement would not violate a
provision of law that required reporting of crimes.
Thus, the formal opinion answers the question with "maybe," as long as doing so would
not hinder the criminal process.
Does it follow that a lawyer should be permitted to agree not to file disciplinary charges
against an opposing lawyer to induce settlement of a civil case? No! The ABA states that a
lawyer may not agree to fail to report any mater the lawyer is required to report under 8.3(a).
That rule states that a lawyer who knows that another lawyer has committed a violation of the
Rules of PR that raises a substantial question as to that lawyer's honesty, trustworthiness, or
fitness as a lawyer in other respects, shall inform the appropriate professional authority.
2.The Model Rules do not permit an attorney to settle a matter for his client by agreeing,
for example, never to defend another drunk driving case. Rule 5.6(b) states that a lawyer shall
not participate in offering or making an agreement in which a restriction on the lawyer's practice is
part of the settlement of a client controversy. This is b/c if you as a lawyer agree not to accept a
certain type of case, it denies the public access to your services for that type of case, and limits
the freedom of clients to choose a lawyer.

E. Problem 21

The Lawyer as Evaluator


Rule 2.3 Evaluation for Use by Third Persons
Lawyer hired by a client to do an evaluation on the client that will be used by a third party in a
transaction affecting the client

F. Problem 22

Rules: Assisting client in fraud, crime


1.6(b)(2)(3)
4.1(b)
1.2(d)
1.16
1.13(b)(c) - may
Obligations When the Client May be Engaged in Fraud

Facts
International Energy, publicly traded company on NYSE, is about to issue new securities.
Strength of company based on reputation for research which is making patents for energy saving
devices. Company's auditors have declared International Energy to be in outstanding financial
health. Director of research, who is a friend of yours, told you at lunch one day that devise hasn't
been sufficiently tested. Engineer told you that the production facility for the devise was
purchased by a shell corporation owned by company's new president (who is shady). He said
price paid was way high. Auditors didn't catch the problem and didn't footnote that purchase was
from company officer. So balance sheet of the corporation looks better than it would if the facility
were carried at its true value. You have been hired to write a favorable opinion letter for the
company about these devices.

A. Disclosure of a Client's Intended Crime or Fraud


1. How sure are you that what your friend has told you is true (i.e. do you really know that
your client's new product is unreliable)? Under rule 1.13(b), if a lawyer for an organization knows
that an officer, employee, or other person associated w/the organization . . . intends to act . . . in a
matter related to the representation that is a violation of a legal obligation to the organization, or a
violation of law that reasonably might be imputed to the organization, and that is likely to result in
substantial injury to the organization, then the lawyer shall proceed as is reasonably necessary in
the best interest of the organization. Rule 1.13 requires you to ascertain that this information is
accurate, and then to report it to someone "higher up" in the company.
However, this information is certainly confidential, and the company will not want it
exposed to the public, which gives rise to a 1.6 problem as well.
3. Other obligations the attorney has under the Model Rules that must be taken into
account: Rule 1.2(d) states that a lawyer shall not counsel a client to engage, or assist a client, in
conduct that the lawyer knows is criminal or fraudulent. Rule 4.1(b) states that a lawyer shall not
knowingly fail to disclose a material fact when disclosure is necessary to avoid assisting a
criminal or fraudulent act by a client, unless disclosure is prohibited by rule 1.6.
Could the lawyer simply withdraw and not take any action? What consequence would
the opinion letter then have? Everyone (the public, for example) could rely on the information in
that letter, and the lawyer has the duty to minimize the harm that the opinion letter could case b/c
he/she knows the information is false or inaccurate. This would be a misrepresentation of fact
under 4.1. Additionally, comment 10 to rule 1.2 states that a lawyer may not continue assisting a
client in conduct that the lawyer originally supposed was legally proper but then discovers is
criminal or fraudulent. The lawyer must, therefore, withdraw from the representation of the client
in the matter. In some cases, withdrawal alone might be insufficient. It may be necessary for the
lawyer to give notice of the fact of w/drawal and to disaffirm any opinion, document, affirmation, or
the like (see 4.1).
Comment 6 to rule 1.13 states that the authority and responsibility provided in rule 1.13
are concurrent w/the authority and responsibility provided in other rules. In particular, this rule
does not limit or expand the lawyer's responsibility under rules 1.8, 1.16, 3.3, or 4.1. Rule 1.13
states that the lawyer "may reveal." Does 1.6 require the lawyer to reveal? 1.6(b)(2)-(3) -
comment 8 says it may and not shall reveal. (b)(3) allows you to rectify the situation assuming
you had no idea there was misrepresentation or fraud. It allows the lawyer to reveal the
information that before they would not be allowed to reveal. Therefore if the corporation is not
going to do anything about it a lawyer may act.
What if the attorney is fired b/c the client sees that he is reporting the fraud "up the
ladder?" Under 1.13(e), a lawyer who reasonably believes that he/she has been discharged b/c
of the lawyer's actions taken pursuant subsections (b) or (c), or who w/draws under
circumstances that require or permit the lawyer to take action under either of those paragraphs,
shall proceed as the lawyer reasonably believes necessary to assure that the organization's
highest authority is informed of the lawyer's discharge or w/drawal.
C. Specific ABA and SEC Responses to a Series of Corporate Failures
3. Sarbanes-Oxley: does not require the lawyer to reveal confidential information
protected by 1.6. However, does require the lawyer to report "up" the corporate ladder.

Class notes 10/25/10


Withdrawal Due to Client Fraud

• How would you summarize 1.16(a)(1)?


Lawyers cannot take representation when doing so violates RPC
& once realize the representation violates RPC then lawyer must
withdraw

o Which rule tells a lawyer that she cannot counsel a client to


engage in or assist a client in conduct the lawyer knows is
fraudulent?
 1.2(d) → lawyer cannot counsel to engage in or assist
in conduct lawyer knows is fraudulent

o How does this rule relate to the lawyer’s obligation to


withdraw under 1.16(a)(1)?
 If lawyer finds herself counseling a client doing fraud
→ lawyer must withdraw
 If lawyer undertakes representation of legit client but
comes to learn that her representation being used to
further a fraud → lawyer duty bound to withdraw
because continuing representation of client would be
furthering the fraud
 No discretion → lawyer must withdraw

• How would you summarize 1.16(b)(2)?


A lawyer may withdraw when lawyer reasonably believes is
criminal or fraudulent

o What is different about the extent to which the lawyer


recognizes the client’s misconduct in 1.2(d) & 1.16(b)(2)?
 1.2(d) → Lawyer must know
1.0(f) → "Knowingly," "known," or "knows" denotes
actual knowledge of the fact in question --- A
person's knowledge may be inferred from
circumstances
 1.16(b)(2) → Lawyer reasonably believes
Reasonably Believes = subjective belief
1.0(i) → "Reasonable belief" or "reasonably believes" when used in reference
to a lawyer denotes that the lawyer believes the matter in question and that the
circumstances are such that the belief is reasonable.
1.0(h) → "Reasonable" or "reasonably" when used in relation to conduct by a
lawyer denotes the conduct of a reasonably prudent and competent lawyer.

• How would you summarize 1.16(b)(3)?


A lawyer may withdraw when client used lawyer’s services to
perpetuate crime/fraud

o What should we be looking for in a fact pattern to


determine whether the lawyer’s situation is governed by
1.16(b)(2) or 1.16(b)(3)?
 1.16 (b)(2) → conduct ongoing
 1.16 (b)(3) → conduct has already happened

o Why difference w/ (b)(3)?


 B/c Lawyer needs to separate herself from situation
even though conduct already occurred
 Used often when client used lawyer’s services for
fraud in past & now wants to use lawyer’s services
again → protects lawyer from being used again

Withdrawal Plus. . .

• A lawyer who withdraws from the representation of a client in


transactional context → the lawyer does not “quietly” w/o giving
notice to anyone but the client

• A lawyer who withdraws from the representation of a client due


to a client fraud or crime must consider whether more is
required.

o What must the lawyer do if she previously provided an


opinion letter or other document to the person who is the
target of the client’s fraud?
 If lawyer does not do anything → lawyer assisting
the ongoing fraud
 1st - Lawyer should sit down w/ client & tell client to
change course or lawyer must walk away → 1.4(a)
(5)
 2nd - If client does not want to come clean or change
course of action → lawyer must withdraw
 3rd - THUS → lawyer must disaffirm/disavow/revoke
her opinion letter
 Lawyer must contact target of the opinion letter &
say that she is walking away from the opinion letter

o What do you suppose will happen when the person who is


the target of the client’s fraud receives word that the
lawyer is withdrawing and/or is disaffirming her earlier
work?
 Sends red-flag to target that something is not right
& should start investigating

o Why do you suppose the comments imply that this sort of


“noisy” withdrawal is preferable to disclosing information
relating to the representation of the client?
 Allows target of the fraud to investigate & gather info
on its own & protects lawyer-client relationship →
lawyer does not have to breach client’s
confidentiality unless absolutely necessary

o What must the lawyer also consider if she or the client


made a material misstatement or omission to the person
who is the target of the client’s fraud?
 Under 4.1(b) → must decide whether is permitted
under 1.6(b)
 If yes → then must disclose
 If no → cannot disclose

o If a noisy withdrawal is sufficient to bring the fraud to a


stop – can the lawyer still make a disclosure under 1.6(b)
or 4.1(b)?
 Must do the least intrusive thing to bring fraud to halt
 If revoking opinion letter brings fraud to halt → then
not required to go any farther
 1.6(b) → lawyer needs to do what lawyer believes is
reasonably necessary
 THUS → if revoking opinion letter halts fraud then
lawyer is CANNOT disclose under 1.6(b) because
disclosure is not reasonably necessary
o What does the last sentence of Comment 3 to 4.1 suggest
about the propriety of disclosure if there is a less invasive
way to avoid assisting the client’s crime or fraud?
 Drafters of the Rules figured a tiered approach for
withdrawal:
• 1st Silent withdraw → Give notice to client
but no one else & often this is sufficient
EX: when lawyer not had any communication
w/ others or target of fraud prior to lawyer’s
discovery of client’s conduct
• 2nd Noisy Withdraw → alerting target of
withdraw
EX: disaffirm/disavow/revoke any docs lawyer
has assisted in providing to target (this may be
enough)
• 3rd Very noisy withdraw & Disclosure of
fraud to target
EX: disaffirm/disavow/revoke is not sufficient to
correct fraud → sometimes need further
disclosure must be made
 NOTE: Only use withdrawal tier for what lawyer
believes reasonably necessary to distance herself
from the client’s fraud

Variation 1:

• You prepare a draft opinion letter that your client intends to


submit to a bank for purposes of obtaining a loan to finance the
manufacture of new product. Letter states: I do not know of any
material facts inconsistent w/ my client’s positive statements to
you regarding the prospects for this product or the company’s
overall financial health.” The director of R&D tells you the
product has not been sufficiently tested, its reliability overstated
& client overpaid for a new production facility & its value on the
balance sheet is overstated.

o Can you sign & provide the opinion letter to the bank as it
is currently written? NO
 4.1 → Cannot make misstatement of material fact

o Can you revise the opinion letter & send it to the bank w/o
obtaining the client’s consent? NO
 Lawyer CANNOT unilaterally change opinion letter
w/o your client’s informed consent
 2.3(b) → if lawyer prepares an evaluation for a client
& the evaluation will have a material or adverse
affect on the client then lawyer cannot provide the
evaluation without the client’s informed consent
 If you were to change the opinion letter to include
the statements of the R&D director → then client not
likely to get the loan
 THUS → if sent revised opinion letter have material
adverse affect on client

o Are you required or permitted to withdraw in this


circumstance?
 YES → you are permitted to withdraw & may even be
required to withdraw

o Does 1.4(a)(5) suggest how you should proceed in the 1st


instance?
 Tell lawyer that when they find themselves in a
predicament w/ a crime → lawyer MUST speak w/
client before withdraw

o If you cannot convince your client to come clean, then


should you withdraw quietly or noisily if you have not had
any direct or indirect contact w/ the bank?
 Quietly → so far all dealing w/ is a DRAFT opinion
letter & not had any contact/communication w/ the
bank so noisy withdraw is NOT reasonably necessary
 Noisily → if draft opinion letter has been delivered to
the bank or the lawyer has been communicating w/
the bank for loan purposes then lawyer needs to
withdraw noisily

Variation 2:

• Assume the same facts except you speak w/ director of R&D


after you sign the opinion letter & after it is submitted to the
bank but before the bank disburses any funds.
o Are you required or permitted to withdraw in this
circumstance?
o Is there a 4.1 issue that should still consider before pulling
trigger & walk away?
YES → still obligation to communicate w/ client to

convince client to come clean
o If you cannot convince client to come client then should
you withdraw quietly or noisy? Noisy
 1.2(d) → Must not assist client in engaging in
criminal/fraudulent act
 1.16 → Must withdraw b/c otherwise your opinion
letter (thus you) are assisting a client’s
criminal/fraudulent act.

o Can you also share what you know w/ the bank? NO


 Financing has not occurred → so no actual damage to
bank yet
IE: Fraud not completed until company receives
financing from bank
 Need to notify bank that you
disaffirm/disavow/revoke your opinion letter & let
bank investigate why

o Which rules are applicable to your analysis?


o How does the bank’s reaction to your withdrawal


potentially affect your ability to share what you know?

o What does the last sentence of Comment 3 of 4.1 tell us


about the drafter’s perception regarding withdrawal in the
context of client fraud.
October 25, 2010
• Obligation to disclose
o How would you summarize RPC 4.1(b)?
 Does RPC 4.1(b) require a lawyer to correct her own material
misstatements or omissions to avoid assisting a fraudulent act? yes
 Does RPC 4.1(b) require a lawyer to correct the clients material
misstatements or omissions to avoid assisting a fraudulent act? Yes so
long as disclosure is not prohibited by RPC 1.6
o How does RPC 1.6 affect the lawyers obligations under RPC 4.1(b)? unless the
disclosure is prohibited by RPC 1.6 there is a mandatory obligation to disclose
under 4.1(b) and if 1.6 prohibits the disclosure there is not requirement to
disclose.
 What are the analytical steps a lawyer must follow for purposes of
determining whether she must make a disclosure under RPC 4.1(b)?
• Step one requires the lawyer to analyze whether the prior
misstatement or omission constitutes fraud under the governing
substantive law.
• Step two requires the lawyer to analyze
 This analysis results in two outcomes
• RPC 1.6(b) permits lawyer to make disclosure and therefore
lawyer make disclosure under RPC 4.1(b)
• RPC 1.6(b) does not permit lawyer to make disclosure and
therefore lawyer make disclosure under RPC 4.1(b)
• Withdrawal due to client fraud
o How would you summarize RPC 1.16(a)(1)?
 Which rule tells a lawyer that she cannot counsel a client to engage in or
assist a client in ocnduct the lawyer knows is fradulent?
 How does this rule relate to the lawyer’s obigation to withdraw under
RPC 1.16(a)(a)
o How would you summarize RPC 1.16(b)(2)? A lawyer may withdraw when the
clients persistent in course of action using the lawyers services and the lawyer
reasonably believes what the client is doing is criminal or fradulent.
 What is different about the extent wo which the lawyer recognizes the
client’s misconduct in RPC 1.2(d) and RPC 1.16(b)(2) 1.2(d) states the
lawyer must know (actual knowledge of the fact in question) the various
things described in (d) but in 1.16(b)(2) says the lawyer just must
reasonably believe (subjective belief which is objectively reasonable).
 How would you summarize RPC 1.16(b)(3)? Lawyer may withdraw when
the client has used the lawyers services to perpetuate a crime or fraud.
(b)(2) the client is following a path to the crime of fraud (3) the client has
used therefore the crime or fraud has already happened.
 What should we be looking for in a fact pattern to determine whether the
lawyer’s situation is governed by RPC 1.16(b)(2) or 1.16(b)(3)? if the
criminal activity or fraud that is not yet completed (b)(2) if the criminal
activity or fraud is already completed focus on (b)(3)
• Withdrawal plus...
o A lawyer who withdrawals frim the representation of a client in the transactional
context typically does so quietly without giving notice to anyone but hte client.
o A lawyer who withdrawals from the representation of a client due to a client fraud
or crime must consider whether more is required
 What must the lawyer do if she previously provided an opinion letter or
other document to the person who is the target of the client’s fraud? You
must say something so it is not ktaken as you facillitating your clients
fraud or crime Under 4.1(a)(5) talk to your client and revoke/disaffirm
your opinion letter by letter or phone
• What will happen when the person who is the target of the
client’s fraud receives word that the lawyer is withdrawing and/or
is disaffirming her earlier work? Send a message to the recipient
that here is something wrong and they should therefore start
asking some questions.
• Why do you suppose the comments imply that this sort of “noisy”
withdrawal is preferable to disclosing information relating to the
representation of the client?
 What must the lawyer also consider if she or the client made a material
misstatement or omission to the person who is the target of the client’s
fraud? 4.1(b)
• If a noisy withdrawal is suffieicent to bring the fraud to a stop,
can the lawyer still make a disclosure under RPC 1.6(b) or
4.1(b)? No you must do the most least intrusive thing to bring the
fraud to a halt. If the revocation of the opinion letter is enough to
bring the fraud to a halt then you cant do anything further,
because RPC 1.6 says he can only disclose what is reasonably
necessary.
• What does the last sentence of comment [3] to RPC 4.1 suggest
about the propriety of disclosure if there is a less invasive way to
avoid assisting the clients crime or fraud? If you can tell the client
to solve the problem and they do then that is good and you can
continue to represent the client; if not then you subtly withdrawal
and are walking away under 1.2(d) and 1.16(a)(1) and also
consider whether you have any documents out there and inform
the people with those documents you are withdrawling and then
there is mandatory disclosure
o A tiered approach to withdrawling form a representing a
lawyer
 (1) withdrawl silently (give notice to the client
you are walking away) the silent withdrawl may
sometimes be adequate even in regard to fraud
etc.
 (2) noisy withdrawl
 (3) disclosure
• Variation One
o You prepare a a draft opinion letter that your client intends to submit to a bank for
purposes of obtaining a laon to finance the manufacturer of a new product. The
opinion letter states “ I do not know of any material facts inconsistent with my
client’s positive statements to you regarding the prospects for this product of the
company’s overall financial health.” The director or R&D tells you the product has
not been sufficiently tested, its reliability overstated and that the client overpaid
for a new production facility and that its value on the balance sheet is overstated.
 Can you sign and provide the opinion letter to thebank as it is currently
written? No not as it is currently written RPC 4.1(a) (false statement of
material fact)
 Can you revise the opinion letter and send it to the bank iwhtout
obtaining the client’s consent? Unless your impliedly authorized you
cannot unilaterally revise the letter without your clients consent RPC
2.3(b) because including these additional statements will necessarily
influence the banks decision of whether to lend and on what terms.
 Are you required or permitted to withdraw in this circumstance? He is
permitted to withdral and may even be reqired to withdrawl 1.4(a)(5)
• Does RPC 1.4(a)(5) suggest how you should proceed in this first
instance?
• If you cannot convince your client to come clean, then should
you withdraw quietly or noisily g you have not had any direct or
indirect contact with the bank? Withdrawl quietly because it is
only a draft opinion which has not yet been shared with the bank.
In this circumstance it owuld be quiet and not be noisy. The
lawyers would have to withdrawl nosily if the lawyer sent the
letter to the bank or has spoken with the bank regarding same.
• Variation 2
o Assume the same facts except you speak with the director of R&D after you sign
the opinion letter and after it is submitted to the bank but before the bank
dispurses any funds.
 Are you required or permitted to withdraw in this circumstance. You are
permitted, but not required because you still have an opportunity to
correct the statement
• Is there an RPC 1.4 issue that you should still consider before
pulling the trigger and walking away? Yes, talk to the client and
see if you can correct the problem.
• If you cannot convince your client to come clean, then should
you withdraw quietly or noisly? Now you are required to withdraw
because if you do not withdraw, you will be assisting your client
in the crime or fraud. 1.6(2)(d) requires you to get out. Withdraw
noisily, but not share specifically the details regarding the
unreliability of the product.
 Can you also share what you know with the bank? no
• Which rules are applicable to your analysis?
• How does the bank’s reaction o your withdrawal potentially afect
your ability to share what you know?
o What does the last sentence of RPC 4.1 comment [3] tell
us what about the drafters perception regarding
withdrawal in the context of client fraud?
 “If the lawyer can avoid assisting a client’s crime
of fraud only by disclosing”
• Variation 3
o Assume the same facts except you learn the truth after you sign the opinion
letter, after it is submitted to the bank and after the bank dispurses funds.
 Can you share what you know with the bank? Yes
• Why isnt noisy withdraw at this point likely to be sufficient? The
bank has already been harmed
• Is RPC 4.1(b) triggered in this circumstance? Yes because you
have t o refer back to 1.6 to see if you can make the disclosure,
then you are required to disclose to the bank under 4.1(b)
o Is this an RPC 1.6(b)(2) situation or is this an RPC 1.6(b)
(3) situation. 1.6(b)(3) because it has already happened
 Are you permitted to withdraw from other, entirely legitimate, matters you
are handling for this client? Yes, pursuant to rule 1.16
• Which two portions of RPC 1.16(b) are aplicable in this
circumstance? Rule 1.16(b)(3) because the client has used the
lawyer’s services to perpetrate a crime or fraud, the lawyer may
withdraaw from the representation. Also, 1.16(b)(1) because it
allows the lawyer to walk away for a good reason, bad reason or
no reason at all as long as doing so will not prejudice the client.
• Testing tips for the unwary
o RPC 1.6(b)(2) and (b)(3) permit disclosure only when the lawyer’s services
havebeem or are being used to further the client’s crime or fraud
o RPC 4.1(b) requires disclosure only when RPC 1.6(b) permits disclosure
o RPC 4.1 comment [3] implies that disclosure is a last resort
 Three tiers of withdrawal and withdrawal “plus”
• Silent
• Noisy
• Disclosure of information relating to the representation
 Don’t pursue a higher tier when a lower tier is sufficient to allow you to
avoid assisting the client’s crime or fraud

VI. Ethical Problems in Litigation


A. Problem 23
The Decision to File a Civil Suit

Facts
Client produces wines which have a short bottle life so to travel better and maintain quality of life
for longer period of time client puts chemical substance which causes cancer in rats. FDA had
Delaney Amendment which determines that any substance consumed by any amount by man or
animal causing cancer must be banned. Reputable scientists support Amendment and others do
not. Client will go bankrupt and FDA doesn't provide compensation for him and changes of
Congress passing a private bill for client is remote. Shelf life for the wine is less than 6 months
so 95% of what he has shipped will be sold to consumers. New wines don't have the substance.
Attorney plans to file suit attacking factual basis for the FDA order and the constitutionality of the
Amendment. Attorney knows court dockets are so crowded likely to delay the effectiveness of the
FDA's order banning the wines. Several years ago, circuit court upheld the law against a similar
constitutional attack.

A. Ethical Standards Governing the Filing of a Civil Action


1. Whether it is proper to file suit in this case, considering that the delay obtained by such
a suit might serve to save the client from bankruptcy: Rule 3.1 requires that there be a basis in
law and fact which includes a good faith argument. The lawyer shall not bring or defend a
proceeding that is frivolous. There may be a problem here with the good faith requirement,
however, because it does not seem that the lawyer cares if he wins or loses, as long as he can
obtain that delay. Comment 1 does state that in determining the proper scope of advocacy,
account must be taken of the law's ambiguities and potential for change, b/c the lawyer is not
always clear and never is static. On the other hand, comment 2 states that an action is not
frivolous even though the lawyer believes that the client's position ultimately will not prevail.
However, comment 2 also states that the lawyer must inform themselves about the facts of their
clients' cases and the applicable law and determine that they can make good faith arguments in
support of their client's positions. An action is frivolous if the lawyer is unable either to make a
good faith argument on the merits of the action taken or to support the action taken by a good
faith argument extension, modification, or reversal of existing law.
2. Rule 11 of the Fed. R. Civ. Proc. states that by presenting to the court a pleading,
written motion or other paper, an attorney or unrepresented party is certifying that to the best of
the person's knowledge, it is not being presented for any improper purpose, such as to cause
unnecessary delay.
Rule 3.2, which governs expediting litigation, states that a lawyer shall make
reasonable efforts to expedite litigation consistent with the interests of the client. Comment 1
states that realizing financial or other benefit from otherwise improper delay in litigation is not a
legitimate interest of the client. On one hand, however, it may be that the lawyer is relying on the
dockets of the courts to be what cause the delay, and not a plethora of motions on his part. On
the other, comment 1 also states that it is not proper for a lawyer to routinely fail to expedite
litigation solely for the convenience of the advocates. Nor will failure to expedite be reasonable if
done for the purpose of frustrating an opposing party's attempt to obtain rightful redress or
repose. It is not a justification that similar conduct is often tolerated by the bench and bar. The
question is whether a competent lawyer acting in good faith would regard the course of action as
having some substantial purpose other than delay.
4. Are the same answers about what is frivolous appropriate when one is selecting issues
to raise on appeal, and should it matter whether the appeal is of a criminal conviction instead of a
verdict in a civil case? Yes. Rule 3.1 states that a lawyer for the defendant in a criminal
proceeding, or the respondent in a proceeding that could result in incarceration, may
nevertheless defend the proceeding as to require that every element of the case is established.

B. The Ethical Status of Delay as a Litigation Tactic


2. Whether it would be proper for the lawyer to recommend that the opposing party's key
witness be hospitalized for possible surgery for an injury that, while not life-threatening nor terribly
painful, limits his workday: Rule 3.4(a) states that a lawyer shall not unlawfully obstruct another
party's access to evidence of unlawfully alter, destroy, or conceal a document or other material
having potential evidentiary value. A lawyer shall not counsel or assist another person to do any
such act. Comment 2 seems to pertain mostly to documentary evidence. However, rule 3.4(f)
states that a lawyer shall not request a person other than the client to refrain from voluntarily
giving relevant information to another party unless the person is a relative or an employee of the
client. Here, the witness is an employee of the client. Comment 4 states that 3.4(f) permits a
lawyer to advise employees of a client to refrain from giving info to another party, for the
employees may identify their interests w/those of the client. Rule 8.4(d) states, however, that it is
profession misconduct for a lawyer to engage in conduct that is prejudicial to the administration of
justice. Whether this would rise to the level of prejudicing the administration of justice would be
for the court/judge to decide. If the FDA filed a motion to compel, the judge could require the
wtiness to testify. The courts have many remedies to stop this kind of conduct on the part of
lawyers.
C. Consideration of ADR Alternatives
1. Comment 5 to rule 2.1 states that when a lawyer knows that a client proposes a course
of action that is likely to result in substantial adverse legal consequences to the client, the
lawyer's duty to the client under rule 1.4 when the matter is likely to involve litigation may require
that the lawyer inform the client of forms of alternate dispute resolution that might constitute
reasonable alternatives to litigation [mediation, negotiation, etc.].
2. Why are there special rules restricting lawyers who act as third party neutrals that do
not apply to non-lawyers who occupy the same role? Rule 2.4 governs lawyers serving as third-
party neutrals. A third-party neutral is a person who acts as a mediator, arbitrator, conciliator or
evaluator to assist two or more persons who are not the lawyer's clients.
4. If a lawyer has been a third-party neutral in a matter, may that lawyer or one of his
partners later represent a client in a related matter? No, under rule 1.12(a), unless all parties to
the proceeding give informed consent, confirmed in writing. Subsection (c) states that if a lawyer
is disqualified by (a), no lawyer in the firm with which that lawyer is associated may knowingly
undertake or continue representation in the matter unless (1) the disqualified lawyer is timely
screened from any participation in the matter and is apportioned no part of the fee therefrom; and
(2) written notice is promptly given to the parties and any appropriate tribunal to enable them to
ascertain compliance w/the provisions of this rule. The rules want to promote mobility of lawyers
and enable them to be able to represent new clients.

B. Problem 24
Litigation Tactics

Facts
Rich lawyer Martin dresses shabby in court to get jury sympathy while helping insurance
companies avoid large tort judgments. He tells his associates not to pick young jurors because of
their social worker mentality. Get a diverse jury of races and classes because disunified grants
small awards. He justifies his tricks to counteract with what he calls the unfair advantage of the
plaintiff's lawyer in winning verdicts because of the sympathy and other reasons not connected
with the merits in the case.

A. The Ambiguous Line Between Creative Lawyering and Deception


1. Should the attorney be subject to criticism for wearing different clothes in court than he
wears to the office? There is no ABA rule on point. However, most courts have their own local
rules that require attorneys, jurors, etc. to dress appropriately for court. Thus, if the judge finds
his attire improper, he will demand that he dresses better. Here, it is clear that the lawyer is
dressing shabbily is to send a message to the jury that the company could only afford a lawyer
from a small law firm and that it cannot afford a large damage award against them. Again, this is
for the judge to control.
2. When, if ever, does a lawyer have a duty to the justice system to pick only jurors who
will be fair to both sides? Rule 8.4(d) and comment 3 state that the lawyer who, in the course of
representing a client, manifests by words or conduct, bias or prejudice based on race, sex,
religion, national origin, disability, age, sexual orientation, or socioeconomic status violates (d)
when such actions are prejudicial to the administration of justice. However, legitimate advocacy
respecting the foregoing factors does not violate (d). A trial judge's finding that peremptory
challenges were exercised on a discretionary basis does not alone establish a violation of this
rule. It may is unconstitutional, however, according to the Supreme Court.
In this case, whether the lawyer has violated 8.4(d) depends upon whether his jury
selection tactics are prejudicial to the administration of justice.
3. When an attorney cross-examines a truthful witness, may he properly try to get that
witness to express uncertainty about something that the lawyer knows is true? Rule 3.3(a)(1)
states that a lawyer shall not knowingly make a false statement of fact or law to a tribunal or fail to
correct a false statement of material fact or law previously made to the tribunal of the lawyer.
Rule 4.4(a) states that in representing a client, a lawyer shall not use means that have no
substantial purpose other than to embarrass, delay, or burden a third person, or use methods of
obtaining evidence that violate the legal rights of such a person. (This is known as the
"badgering" rule).
In this case, as long as the attorney is not badgering the witness or making false
statements to the tribunal, he is ok. A lawyer is allowed to test the knowledge of witnesses on
cross.
4. Whether there are ethical problems in putting the attractive secretary up to acting like
she was the defendant's new wife: Rule 3.5(a) states that a lawyer shall not seek to influence a
judge, juror, prospective juror or other official by means prohibited by law. This rule seems to
speak more to bribery, so it may not apply here. However, by putting the secretary up to it, his
intent was to communicate to the jury that the secretary was the defendant's new wife, even if he
wasn't the one who ultimately made the communication, b/c it was the secretary who patted the
defendant on the head. Rule 8.4(a), however, states that it is professional misconduct for a
lawyer to violate or attempt to violate the rules, knowingly assist or induce another to do so, or do
so through the acts of another.

C. Problem 25
Disclosure of Law or Facts Favorable to the Other Side

Facts
You are sure you will win your case on motion for SJ, but then hours earlier find dicta that is
adverse to your client which would make judge rule against you. You know of a witness whose
facts will also be adverse to your client and you know opposing counsel doesn't know of his
existence. You then read deposition where you see a witness testified about a fact in a false
manner. In another case, client is about to be sentenced and court has no record of prior criminal
records on client so judge says he will just get probation but you know he does have criminal
record and judge asks you "Anything to add counsel"

A. Candor About Adverse Legal Authority


1. Must you cite all relevant cases to the court, even those not favorable to your position?
What are the limits, if any, on that obligation? 3.3(a)(2) states that a lawyer shall not knowingly
fail to disclose to the tribunal legal authority known to the lawyer to be directly adverse to the
position of the client and not disclosed by opposing counsel. Comment 4 states that it constitutes
dishonesty to the tribunal. Honesty to tribunal is highest duty in these rules, even moreso than
confidentiality. The authority/case has to be from controlling jurisdiction and be directly adverse.
In this problem, it is dicta and is analogous, so it is not directly adverse. When you are unsure
whether a case is directly adverse to your case, you should disclose it to the court because
opposing counsel or the judge can always claim it is directly adverse, so just disclose it and
distinguish it.
2. Rule 3.4 your duty to the court at times is greater than the duty to your client. The
duty of the court to make an informed duty. It has to be a controlling jurisdiction in order for it to
apply. If you can distinguish the case from yours then you should disclose and distinguish it.
You have to determine whether the court will benefit from hearing a case. The lawyer that
doesn't find the case has a 1.1 problem because he is not competent and if you say that you
didn't know about the case on the other side then you also run a problem with 1.1.

B. Candor About Adverse Facts


1. Do the Rules treat disclosure of the newly discovered witness differently form the
disclosure of adverse legal authority? Why? Rule 3.3(d) comment 14 states that in ex parte
proceeding no opposing advocates so of course has to argue both sides here when there is
opposing counsel not bringing it up is not lying unless court asks if there are any other witnesses
that would know of this. This is not the same as disclosing legal authority because court concern
for establishing bad law. Typically in interrogatories they ask you to make a list of the witnesses
that have information about the case so you would have to disclose but in the problem you were
never asked. Since only one side is present it is important for everything to be disclosed.
2. When should a court conclude that a lawyer deceived it by not volunteering factual
information? Gives cases where it says you get in trouble for not doing what the prior question
says you don't have to-you should when for ex. client has died-Toledo because then you no
longer have the agency relationship

C. Candor About Incomplete or Inaccurate Discovery Responses


1. What should a lawyer do if, as in this Problem, she discovers that the client of a
material witness has given false testimony in a deposition? Rule 3.3(b) is in effect during
depositions; it doesn't matter whether the witness is physically in court. The witness cannot make
a false statement; if lawyer knows of its falsity he/she must take remedial measures-disclosure to
the court when you have come to learn you have submitted false information even if it requires
breaking confidentiality.
The best way is to get client to correct it. In Jones v. Clinton, the attorney for
Clinton later told court when he found out that Clinton lied during deposition he told the court.
Clinton had given evasive answer for the obstructions of justice. Clinton had said that he was
never alone with Monica and that he never had sex with her.
Duty of candor to the court is higher than the duty of confidentiality. Comment 1
states that it also applies to depositions.
Rule 3.4 is concerned with your fairness to the opposing side in Penthouse v.
Playboy in problem p. 412, the court will likely impose sanctions for this conduct and these rules
also say that you have to comply with these discovery request.

D. Candor as to Factual Matters That Are Not Easily Verifiable


1. Is this the third item not disclosed to the court in the Problem-the prior criminal record
of your client-a factual matter or as legal matter, i.e. is it more analogous to the situation in Part A
or Part B of the Problem?
Is silence a false statement? No, but when court asks if you have anything to add then
you must be honest. If the Judge wouldn't have asked the attorney do you have anything to add,
does the attorney have a duty to disclose? If the court asked the lawyer, the lawyer should say
that they should be excused from answering; if the court does rely on him then the lawyer must
disclose; if the court doesn't rely on him and the client doesn't lie then the attorney most likely
doesn't have to disclose. They have not made a clear decision on this.
Being honest could break rule 1.6 (confidentiality). ABA formal opinion 287 says you
should shows gray area not clear answer-says lawyer should reveal to the court about prior
criminal record to the court if the lawyer believes the court relies on him as corroborating the
correctness of client's statement-lawyer's duty of candor to the court requires him to disclose to
the court
D. Problem 26
Handling Physical Evidence

Facts
Hammer, man you have never represented before, comes into office and puts gun and money on
your desk telling you he just robbed a bank and killed a guard in the process and wants your help.
Police contact you about reports that man seen with gun has walked into your office. Also,
Wallace, man being investigated for price fixing asks got your help to prepare for interview with
reporter and gives you secret tapes of conversations over the last 3 years.

A. Confidentiality of a Client's Identity


1. Client identity is not normally confidential. The traditional rule is that no privilege
attaches either to the fact that someone has consulted a lawyer or to the general subject of the
representation. The general evidentiary rule is that the identity of the client, amount of the fee,
identification of payment by case file name, and the general purpose of the work performed are
not usually protected from disclosure by the attorney-client privilege, b/c such information
ordinarily reveals no confidential professional communications b/t attorney and client.
Might be different if the fact that a person consulted a lawyer is in itself incriminating or
embarrassing. This is known as the "missing link" theory. If the name is the missing link in
making the case against the client, then the client's name will probably be protected.
3. Is there an exception to rule 1.6? Yes; if the lawyer must defend him or herself in a
controversy b/t the lawyer and the client, the lawyer may reveal confidential information relating to
the representation of the client. If the court orders an attorney to reveal his/her client's name or
else go to jail, 1.6(b)(6) states that the attorney may reveal it (or other information relating to the
representation of the client) to comply with other law or a court order. Comment 13 states a
lawyer may be ordered to reveal information relating to representation of a client by a court or by
another tribunal or governmental entity claiming authority pursuant to other law to compel
disclosure. However, absent informed consent of hte client to do otherwise, the lawyer should
assert on behalf of the client all non-frivolous claims that the order is not authorized by other law,t
that the information sought is protected against disclosure by the attorney-client privilege or other
applicable law. (In other words, don't give the information up freely; fight for your client's right
under attorney-client privilege first).

B. Taking Possession of Physical Evidence for Testing of Safekeeping


1. The attorney should not be a depository for criminal evidence, which in itself has little if
any material value for the purposes of aiding counsel in the preparation of the defense of his
client's case. Attorney-client privilege does not apply to physical evidence.
In re Ryder, 4th cir 1967-Aug 1966 man robbed bank. Another man, Cook, rented safety
box and was later interviewed by FBI found with money that had same markings from earlier
robbery. Cook said he got it from the robber who offered him 500.00 to open a safety deposit box.
Cook asked attorney Ryder what he should do and Ryder told him to tell the truth. Ryder
conferred with bar association who told him to take the money and give it back to authorities while
letting Cook know what you are doing. Ryder instead created a power of attorney signed by Cook
letting Ryder take the money and give it back without telling him. Ryder took the money and gun
used in robbery and conferred with t a judge who said not violating ethical conduct. FBI took
Ryder's box. Holding: 4th Cir said not attorney client privilege. Ryder initiated the actions. He
was holding stolen property and had possession of a gun with the intent of protecting criminal
from consequence of the crime.
Rule 3.4(a) states that a lawyer shall not unlawfully obstruct another party's access to
evidence or unlawfully alter, destroy, or conceal a document or other material having potential
evidentiary value. A lawyer shall not counsel or assist another person to do any such act.
Comment 2 states that the right of the opposing party, including the government, to obtain
evidence through discovery or subpoena is an important procedural right. The exercise of that
right can be frustrated if relevant material is altered, concealed, or destroyed. In this case, if the
attorney even takes the gun from the client he/she is altering it [fingerprints]. 3.4(a) applies to
computerized information. It is an offense in most jurisdictions to destroy material for purpose of
impairing its availability in a pending proceeding or one whose commencement can be foreseen.
Comment 2 also states that applicable law may permit a lawyer to take temporary
possession of physical evidence of client crimes for the purpose of conducting a limited
examination that will not alter or destroy material characteristics of the evidence. Under this
comment, perhaps the lawyer could turn the gun in for his client w/o revealing the client's name.
To prevent this situation entirely (so that the attorney never sees evidence of the crime),
the lawyer can tell the client up front that he/she will not hold or examine any physical evidence.
However, whatever the client says about the physical evidence (e.g. "I just used this gun to kill a
man) is confidential and protected by attorney-client privilege and rule 1.6.

C. Nondisclosure of Evidence
2. Client tells you he killed certain people and that he buried their bodies, but when
lawyer checks out his story, he finds out that one of the victims is not yet dead. May he tell the
police her location? Yes, under 1.6(b)(1): a lawyer may reveal information relating to the
representation of a client to the extent the lawyer reasonably believes necessary to prevent
reasonably certain death or substantial bodily harm. However, with regard to past crimes, the
lawyer is prohibited from revealing the information b/c it is protected by attorney-client privilege
and rule 1.6.
Remember also that the lawyer is prohibited from altering evidence under 3.4(a).
Therefore, the attorney cannot touch any of the bodies or help the client bury any of them, etc.
The lawyer can seek assistance for the victim if one of the victims is still alive (see 1.6(b)(1)), but
he cannot touch her.

D. Destroying, or Failing to Retain, Physical or Documentary Evidence


2. Cannot advise the client to destroy them under 3.4(a), especially since comment 2
states that it is an offense in most jurisdictions to destroy material for purpose of impairing its
availability in a pending proceeding or one whose commencement can be foreseen. Here, the
proceeding can be foreseen. 1.2(d) also prohibits aiding or counseling the client in committing
criminal or fraudulent activity, but the lawyer does have a duty to tell him the legal consequences
of the incriminitating/damaging evidence.

Class Notes, October 27, 2010:


• Variations on Problem 26:

o Variation One: Neil Hammer comes to your office with a gun


and a bag of money and tells you he robbed a bank and shot
a guard. He asks for help about what to do so he won’t get
caught.

 Can you advise Hammer about what to do so that he


won’t get caught?

• No under Rule 1.2(d)  Cannot assist him in


covering up the criminal activity.
o You would be characterized as an
accomplice if you assist him.

o Variation Two: The police see their suspect enter your office
and then flee, getting away completely.

 What is the general rule regarding the applicability of


the attorney-client privilege to questions regarding a
client’s identity?

• General rule  cannot invoke the attorney-client


privilege to preclude disclosure of the client’s
identity. (it is normally not considered
confidential)

o What is the exception to the discoverability


of client identity that may be applicable in
this circumstance?

 Sometimes the very identification of a


client would disclose a privileged
communication

 Last Link doctrine  applicable in


rare situations.. The name of the
client is the last link that law
enforcement needs to link the client
to the commission of the crime.

• Extends the protection of the


attorney-client privilege to
nonprivileged information.

• Then the lawyer may assert the


privilege to hide the identity of
the client.

o Variation Three: Hammer flees your office but leaves the


gun and the money behind.

 What is the general rule regarding what lawyers should


do when they find themselves in possession of the fruits
or instrumentalities of a crime?
• General Rule  Restatement third §119/ comment
2 to RPC 3.4The lawyer may retain those items for
a reasonable amount of time to examine them
and sometimes test them, but then the lawyer
must turn the evidence over to law enforcement
or at the minimum alter law enforcement after a
reasonable time has expired.

o Can you perform tests on physical evidence


that is in your possession?

 Under RPC 3.4 comment [2] Yes so


long as you can do so without
unlawfully altering or concealing the
evidence. You cant perform test which
have the affect of altering the
evidence.

 Assume you decide to turn the money and the gun over
to the police.

• Are you required to explain to police how you


obtained the gun and money in this particular
circumstance?

o Not if it will incriminate your client If you


focus on the attorney’s duty of
confidentiality  lawyer would not have to
disclose this information. If you obtained the
gun and money from one of hammers
friends your answer would be different
because the duty of confidentiality would
not extend to that friend.

 You can invoke Attorney- client


privilege. Can tell the police you
cannot disclose that information.

o However, in the context of receiving


evidence from a non-client you can tell the
police where you received the evidence.
 Was not in the course of
representation. Merely turning over
evidence.

• There is no attorney-client
privilege. The privilege never
attaches. And the lawyer would
have to disclose where he
received the evidence from.

o Variation four: Hammer does not leave the gun or money in


your office but tells you where he hid them. you drive to the
location and see the gun and money but do not touch either.

 Do you have an obligation to share the location of the


gun and money with law enforcement?

• If he merely observes it, does not touch it, or take


it to his possession  he does not have to disclose
to law enforcement. He has not touched or
changed anything and therefore has not affected
law enforcement in any way, and it is still
available to law enforcement in its natural state.

o Rule 3.4(a)  A lawyer shall not unlawfully


obstruct another party’s access to evidence
or unlawfully alters, destroy or conceal
evidence.

 If you take possession of the gun and money, then do


your obligations change in any way?

• You must turn in each of the two items after a


reasonable time.

• You must disclose information about the evidence


if asked.

o He must turn it over and disclose to law


enforcement where he got it and what it
looked liked in its natural state. Because by
picking it up and taking it back to his office
has obstructed the natural state.

 Do the answers to these various questions make sense


to you in relation to the text of RPC 3.4(a)?

• Yes, so that law enforcement would have


information to the evidence in its natural state,
which they would have had access to if the lawyer
would have left it untouched.

o Variation Five: hammer shoots a person dumps the body in


a dumpster. He tells you the location of the dumpster and you
go there and see that the vistim is alive but barely breathing.

 Do the RPCs require you to disclose the location of the


victim to police?

• By telling the police, emergency personnel may


be able to save the victim and then the victim will
be able to identify your client.  it would be
adverse to representation.

o But rely on 1.6 (b)(1) may reveal but


imposes no obligation on the lawyer to
disclose this information.

o RPC simply says you MAY reveal information


relating to a person who is reasonably
certain to suffer severe injury or death. (not
required but are permitted)

o But if you wanted to disclose, you can in


order to prevent substantial harm.

 Why do you suppose the criminal defense bar concluded


that lawyers should not disclose in this circumstance?

• That the lawyer should not disclose and is


ethically prohibited from disclosing, because you
would be harming the client because of the
victims subsequent identification.
o Variation Six: hammer shows you the gun that he used in
connection with the robbery.

 Can you advise him to wipe the fingerprints off the gun
then dispose it?

• Which of the following rules are implicated in this


situation? All of them

• 1.2(d)  you would be assisting/counseling a


client in the crime… cannot tell him to do this

• 3.4(a)  you cannot counsel another person to


unlawfully obstruct alter or conceal (2nd sentence)

• 8.4(a)  first clause defines professional


misconduct. Because he has already violated
other rules (Second clause (Not applicable) says
you cannot knowingly assist or induce someone
but only applies to other lawyers not to non-
lawyers.) Third clause says if you cant do it then
you cant avoid the rules by getting someone else
to do it on your behalf (Engage in professional
misconduct).

o “… Or to do so through the acts of another.


“

• 8.4(b) Obstructing evidence/being an


accomplice is a criminal act and it reflects on his
“honesty, trustworthiness and fitness as a lawyer
in other respects”

• 8.4(c) by concealing evidence it is deceitful.

• 8.4(d) by tampering with evidence, it is


prejudicial to the administration of justice.

o Variation Seven: Hammer is arrested and you represent him


at trial. You want to call a fact witness to testify on his behalf
bu the witness lives in another state and does not have
enough money to pay for travel.
 Can you pay for any of the following?

• The amount necessary to reimburse the witness


for a train or bus ticket so that she can travel to
the courthouse?

o Yes. 3.4 Comment [3]  Nothing


inappropriate to pay train ticket.

• The amount for food and lodging while at trial?

o Yes

• The amount for lost wages?

o Yes

• An additional $1,000 in the event hammer is


acquitted?

o No, attempting to induce testimony in your


favor.

 How would you articulate a general rule based upon


your answers?

o Always permissible for a lawyer to


reimburse a witness for expenses. But
cannot pay any amount that would induce
the witness to sway one way or another.
RPC 3.4(b) and comment [3]

o Variation Eight: You also want to call an expert witness to


testify on Hammer’s behalf.

 Can you pay the following?

• Reimburse for time she spends investigation the


case, formulating her opinions and preparing to
testify?

o Yes.
• The amount necessary for out-of-pocket
expenses?

o Yes

• An additional $1,000 in the event hammer is


acquitted?

o No.

 How would you articulate a general rule based upon


your answers?

o Always permissible to reimburse for


expenses. But cannot pay any amount that
would induce the witness to sway one way
or another. 3.4 comment [3] says it is not
improper to compensate an expert witness
on terms permitted by law, but is
impermissible to pay an expert witness a
contingent fee.

o Variation Nine: JB Wallace tells you that certain companies


in the industry may be under investigation for price fixing in
violation of federal antitrust laws. He does not know which
companies are involved, or whether his company is among
them, but he has audio recordings of all the conversations he
has had in his office for the last three years. He asks whether
he should keep or destroy the recordings.

 What are the various considerations that should go into


your analysis?

• Under 3.4(a) First in foremost need to determine if


it is “unlawful” to destroy these recordings.

 What is the significance of the word “unlawfully” in RPC


3.4(a)?

• Do not have an issue unless it is unlawful to


destroy the evidence. Doesn’t say you cannot
alter or obstruct evidence in some cases, it states
simply that you cannot do so “Unlawfully”
o What if there is not a federal law prohibiting
the destruction of evidence prior to
commencement of a formal investigation or
judicial proceeding?

 He can destroy them. if there is not a


body of law that prohibits the
destruction at the time the client asks
the question, then it is not unlawful or
unethical from telling the client to
destroy the tape recordings.

• What is the practical consequence of destroying


the recordings even if not unlawful?

o It looks like you have something to hide.


From the perspective of the law or ethics
there would be no consequence. But from a
practical consequence  everyone on the
other side they will squawk on the cover up.
And if you destroy the tapes you may have
destroyed things which may be helpful to
you and/or your client.

 What does this suggest about the propriety of document


retention policies in which documents are routinely
shredded when no longer needed by an organization?

• Document retention policy  minimize the


amount of paperwork kept in storage help keep
documents out of the wrong hands.

o Notwithstanding the document retention


policy, if a subpoena is issued  the papers
cannot be destroyed.

o From a practical sense  inform client not


to destroy the evidence.

o Variation Ten: A district court rule says that unpublished


opinions do not have precedential value and therefore cannot
be cited in motions, briefs or supporting memoranda.
 Would a lawyer violate 3.4 if she cited an unpublished
opinion in a motion or brief in that district court?

• Yes she would violate 3.4(c).

 Would a lawyer violate RPC 3.4 is she cited an


unpublished opinion in a motion or brief for purposes of
challenging the validity of the district court rule?

• No 3.4(c) says that there is an exception for open


refusal based on an assertion that no valid
obligation exists, so by challenging this rule you
are asserting no valid obligation exists.

o Variation Eleven: A district court rule prohibits defense


lawyers from asking plaintiffs in wrongful death actions
whether they have remarried.

 Which two portions of 3.4 are implicated?

• 3.4(e) guilt or innocence of an accused/allude to


a matter that the lawyer does not reasonably
believe is relevant.

• 3.4(c)  it is not an open refusal (it is knowingly


disobeying a rule imposed by this tribunal)

o Variation Twelve: The plaintiff’s lawyer states during closing


argument that her client is the most honest, trustworthy and
credible witness she has encountered in thirty years of
practice.

 Which portion of 3.4 would the plaintiff’s lawyer violate


if she made this statement?

• 3.4(e)  lawyer is stating a personal opinion/will


not be supported by admissible evidence/and it is
in regard to the credibility of a witness.

o Variation Thirteen: The P’s lawyer instead states during


closing argument: “I know the defendant caused the accident.
I saw the position of the two cars right after the accident.”
 Which portion of 3.4 would be implicated if the P’s
lawyer actually made that statement?

• 3.4(e)  cannot say this if there has not been


evidence presented to support these facts. (cant
say it unless It will be supported by admissible
evidence)

o Variation Fourteen: Barbara Bentley represents Speedy


Corp. in an antitrust action brought by the US govt. Bentley
does not want any of the company’s current employees to
speak with lawyers for the DOJ outside her presence.

 What can she do consistently with RPC 3.4 to


accomplish this result even assuming the employees
are not deemed to be her clients?

• 3.4(f)(1) and comment [4]

• allows lawyers to ask organizational clients


employees to not voluntarily share information
with adverse parties outside the lawyers
presence.

• 3.4(f)(2) Must have to reasonably believe that


their non-cooperation with the govt on a voluntary
bases will not be adverse

E. Problem 27
The Client Who Intends to Commit Perjury

* THE DUTY OF CANDOR TO THE COURT IS HIGHER THAN THE DUTY OF


CONFIDENTIALITY TO YOUR CLIENT! *

Facts
Smith, D, in robbery prosecution, was represented by Hawley. Smith said he wanted to take the
stand and testify that he was at girlfriend's house during robbery which was a lie. Hawley said he
couldn't let him perjure himself and so Smith promised not to but did anyway.

A. Knowing When a Lawyer Knows Something


2. Under 1.2(d), the lawyer cannot assist or counsel the client to commit perjury. Under
3.3, comment 9, however, the lawyer must honor the client's wish to testify. The lawyer cannot
elicit testimony from the client that the lawyer knows (under 1.0) is false, however.

B. The Decision to Call a Witness Who May Testify Falsely


1. Must the lawyer refuse to call Smith's girlfriend if he knows she will commit perjury on
his client's behalf? Yes. Under 3.3(a)(3) the lawyer may refuse to offer evidence, other than the
testimony of a defendant in a criminal matter, that the lawyer reasonably believes is false. [Note
the difference: with the criminal defendant, the lawyer must know the evidence is false to refuse
it; with any other witness, the lawyer must only reasonably believe that the testimony will be
false]. According to comment 9, this is b/c of the special protections historically provided to
criminal defendants; unless the lawyer knows that the testimony will be false, the lawyer must
honor the client's decision to testify.
3. What if the lawyer discovers that his witness lied under oath, but the lawyer has an
argument that the factual misrepresentation is not material?
Material that can be used for impeachment is material!

C. The Decision to Call the Defendant Who May Commit Perjury


2. Whether a lawyer must sit by and do nothing if the criminal defendant wants to testify
falsely: Supreme Court stated in Nix that although counsel must take all reasonable lawful
means to attain the objectives of the client, counsel is precluded from from taking steps or in any
way assisting the client in presenting false evidence or otherwise violating the law. Under 1.2(a),
the client has the right to make the decision to testify. However, the right to testify does not
extend to perjured testimony. The argument that the lawyer counseled the client to testify
truthfully and that doing so was ineffective assistance of counsel will fail as well.
Once the lawyer feels that he/she has talked his/her client out of perjuring himself, but
then the client gets on the stand and lies, the lawyer must take remedial measures under 3.3,
comment 10. This includes disclosure to the tribunal, and 3.3(c) states that this duty to disclose
includes confidential information. 3.3 takes precedent over 1.6 in this situation. Before the
lawyer does this, however, he should try to dissuade his client. [One way is to ask for recess and
take the client aside and tell him that if he is not allowed to perjure himself, and that if he does not
correct himself, then the lawyer will]. If that is not successful, the lawyer will reveal that his client
lied, and may withdraw under 1.16.
Comment 6 to 3.3 states that if a lawyer knows that the client intends to testify falsely or
wants the lawyer to introduce false evidence, the lawyer should seek to persuade the client that
the evidence should not be offered. If the persuasion is ineffective and the lawyer continues to
represent the client, the lawyer must refuse to offer the false evidence. If only a portion of the
witness's testimony will be false, the lawyer may call the witness to testify but may not elicit or
otherwise permit the witness to present the testimony that the lawyer knows is false.
Comment 13 states that the duration of the lawyer's obligation to be truthful to the tribunal
is a practical time limit and that the conclusion of the proceeding is a reasonably definite point for
the termination of the obligation.

D. What to do When the Client Does Give False Testimony


• Variations on Problem 27

o Overview of RPC 3.3(a)(1)

 What is the basic obligation imposed by RPC 3.3(a)(1)?

• A lawyer cannot knowingly make a false


statement of fact or law to a tribunal, AND must
correct a false statement of material fact made
previously to the tribunal.
 Is the obligation to speak accurately the same as the
obligation to correct mistakes?

• No, you only have to correct statements that are


material after the fact.

• But you cannot knowingly make a false statement


of any sort.

 You ask the court to reschedule your client’s deposition


b/c she will be on vacation in san Francisco.

• Would you be violating 3.3(a)(1) if you believed


your client is head to SF but learn afterward that
she is actually headed to SD.

o No, it was an innocent mistake of fact.

 Are you required to go back and tell


the court where your client is going?

• No, because this is immaterial


to the case.

• Would you be in violation of RPC 3.3(a)(1) if you


knew at the time of the motion that your client is
actually headed to SD rather than SF?

o Yes, cannot knowingly speak inaccurately.

 Are you required to go back and tell


the court where your client is going?

• No you do not have to, because


it is not material. In the absence
of this “material” statement, no
correction needs to be made.

• In terms of policy, why do you think the rule is


written this way?

o If it wouldn’t have made a difference to the


decision maker, then there should not be an
obligation to report it.
o Higher std. in the first instance, but a much
lower std in the second instance.

o Overview of RPC 3.3(a)(2)

 How would you summarize RPC 3.3(a)(2)?

• Have an obligation to tell authority even if it is


disadvantageous to your client.

 What does “legal authority” refer to?

• Regulations, statutes, controlling authority, etc.


(any authority)

o Does the rule limit the obligation to disclose


to “controlling” legal authority?

 Legal authority in this rule means


rules from controlling jurisdiction…
does not have to be binding.

 When the rule refers to the “controlling jurisdiction” to


what is it referring?

• Refers to the substantive rule that provides the


rule. It is any law that would be controlling in that
jurisdiction

o A civil action pending in state court alleging


a violation of the state’s law?

 State law

o A civil action is pending in federal court


alleging violation of federal law?

 Federal Law w/in that Circuit.

o A civil action pending in federal court


alleging a violation of some state’s law?

 That particular state’s law will be


controlling.
 What is the point in time by which you must disclose
directly adverse authority to comply with the rule…

• If you are the moving party?

o If and when it is not disclosed by opposing


counsel

• If you are the opposing party?

o Anytime

o Overview of RPC 3.3(a)(3)

 How would you summarize the first sentence of RPC


3.3(a)(3)?

• A lawyer shall not offer evidence he knows is


false.

o How should you proceed if your client tells


you well before trial that she intends to
testify falsely?

 Sit down with the client and explain


the consequences of the false
testimony.

• Consequences are…

o She must tell the truth

o She will commit perjury


and can be prosecuted
under criminal law)

o Obligation on lawyer to
tell the tribunal.

o Lawyer may withdraw


pursuant to 1.16

o 1.16(c)  must ask the


court for withdrawal.
o Tell client that you cannot
offer false evidence, and
will not ask her the
questions that the lawyer
will know will have false
responses.

o How should you proceed if your client tells


you during trial that she intends to testify
falsely?

 The analysis is exactly the same. It is


not as easy to withdraw at this time as
in the earlier hypo.

 How would you summarize the second sentence of RPC


3.3(a)(3)?

• Lawyer, client, or witness gives a false statement


of material evidence, then the lawyer realizes it is
false, the lawyer must take reasonable steps to
correct it.

o How would you proceed if our client tells


you after testifying that she answered
several important questions falsely?

 First instance, try and get client to


come clean, but if client refuses, the
lawyer is duty bound to disclose to the
court.

o Are your obligations the same or different if


the client’s false testimony comes out on
cross examination rather than on direct?

 They are the same.

 How would you summarize the third sentence of RPC


3.3(a)(3)?

• Lawyer has discretion to offer evidence the lawyer


reasonable believes is false.
o You can offer evidence that you reasonably
believe that is false but that you do not
know is false.

 But must look at the information and


come up with the decision after giving
it much thought.

• How does it apply in the criminal defense context?

o Cannot deny the accused of the ability to


testify, even if you know that she will testify
falsely. (its their constitutional right)

o Overview of RPC 3.3(b)

 How would you summarize RPC 3.3(b)

• Lawyer representing a client in an adjudicative


proceeding who knows their client intends, is, or
has engaged in criminal or fraudulent behavior
has to take reaspnable remedial measures
including if necessary disclosure to the tribunal.

• What sorts of criminal of fraudulent conduct is this


portion of the rule getting at?

o Focuses on all the various criminal and


fraudulent activity that can happen in trial
except for those relating to evidence.

 Bribery

• Whose criminal or fraudulent conduct forms the


basis for the obligation?

o Any person, it doesn’t matter who is doing


the act.

• If a lawyer knows that a person has engaged in


witness intimidation in a lawsuit in which she is
not involved is she required to do anything?
o No, because the lawyer does not represent
anyone in the case. There is no duty.

o For the duty to be imposed, the lawyer has


to be related to the case.

o She may disclose, but she cannot be held


professionally responsible for failing to do
so.

November 3, 2010

o Overview of RPC 3.3(c)

 How would you summarize RPC 3.3(C)?

• Duties of (a) and (b) continue to the conclusion of


the proceeding. RPC 3.3(c) also states that this
rule trumps anything that may be barred by rule
1.6 therefore if you do a RPC 3.3 analysis and you
encounter a 1.6 problem RPC 3.3 will override 1.6.
(it is more important for a person to be honest
and truthful to a tribunal than to protect some
statements made by a client.)

 Which of the following is the “conclusion of the


proceeding” for purposes of this rule:

• Jury verdict

o No

• Entry of judgment

o No

• Expiration of the time for making post –trial


motions

o Yes

• Decision by court of appeals if an appeal is taken

o No if the ruling is remanded; Yes if the


ruling is affirmed
• Decision by supreme court if further review is
permitted

o Yes as long as the judgment is affirmed.

 Conclusion of the proceeding is when


the moment in time when further
procedural litigation is barred by time.

 The case is over when there is nothing


more you can do based on procedural
rules.

 Comment [13] states “a proceeding


has concluded within the meaning of
this Rule when a final judgment in the
proceeding has been affirmed on
appeal or the time for review has
passed.

• Overview RPC 3.3(d)

o How would you summarize RPC 3.3(d)?

 The duty to represent their client and disclose facts,


whether or not the facts are adverse, in an ex parte
proceeding.

• What is an ex parte proceeding?

o Proceeding in which only one party to the


lawsuit is represented

• Why do you suppose the duty of candor is so


broad in the context of ex parte proceedings?

o Because the other party is not there to


represent the other side. Because they are
the only ones speaking to the court.

• Variation One:

o Smith is the defendant in a criminal matter. Smith tells his


lawyer he wants to testify and also wants a friend to testify in
support of an alibi defense. The lawyer points out that Smith
has never raised the possibility of an alibi defense before.

 What inference can be drawn from the fact that Smith


has not brought up before?

• IT is fabricated

• “does he have a reasonable belief” that the


defense is fabricated?

o Yes the alibi would have been brought up


before.

• Does he “know” that it is fabricated?

o Has to have actual knowledge, but it can be


inferred.

 How does Smith’s right to testify affect the lawyer’s


obligation to call smith if…

• The lawyer has a reasonable belief that the alibi


defense is fabricated?

o Must try to talk him out of it. The lawyer has


the discretion to not call a witness unless he
is the accused in the case. The lawyer then
becomes duty bound.

• The lawyer knows that the alibi defense is


fabricated but there are other issues about which
Smith’s testimony is relevant and about which he
will testify truthfully?

o He can only ask about the information that


he knows the witness will testify truthfully.

o First clause of 3.3(a)(3) – cannot offer


evidence that the lawyer knows is false. So
he cannot illicit testimony that is false. He is
entitled to call Smith except precluded from
asking questions that he knows are
fabricated.
• The lawyer knows that the alibi defense is
fabricated but there are no other issues about
which smith’s testimony is relevant?

o Smith will not be able to testify at all.

o The lawyer shall not offer testimony that


she knows is false.

o The accused has the right to testify but does


not have the right to testify falsely. It is not
unconstitutional to not allow the defendant
from testifying falsely.

o A criminal defense lawyer cannot call a


criminal defendant if the lawyer knows the
defendant will testify falsely.

 How does the analysis change, if at all, when we


consider the friend’s potential testimony if…

• The lawyer has a reasonable belief that the alibi


defense is fabricated?

o The lawyer has a duty to not aid and abet


perjury.

o But the lawyer has discretion to call the alibi


witness. He does not have the obligation to
call the friend.

• The lawyer knows it is fabricated?

o Cannot call the friend.

o RPC 3.3(a)(3) – first clause  not knowingly


offer evidence that he know is false.

• Variation Two

o Smith convinces the lawyer that he will testify truthfully but,


unfortunately, testifies falsely on several important points.
• RPC 3.3(a)(3) says there must be false testimony
about an important point and not an unimportant
point; obligation only attaches when the lawyer
comes to know the lawyer, the lawyers client, or
the lawyers witness has given false testimony of
importance.

 How should the lawyer proceed.

• Must take reasonable remedial measures to


correct misrepresentation that are material.

o The lawyer has to remonstrate


confidentiality with the client and try and
convince the client to come clean.

 If the client does not come clean, the


lawyer would have to disclose the
information to the tribunal, even if he
is not required to withdraw.

 May be subject to criminal prosecution


for perjury.

o The lawyer cannot convince Smith to come clean and so


disclose the false testimony to the court.

 Have to disclose to the court that the client testified


falsely.

• What are the courts options?

o IN THE CONTEXT OF TRIAL, IT IS HARD TO


CONVINCE A TRIAL JUDGE TO LET YOU
WITHDRAW. (must requests are denied)

o The court can suppress the testimony.

o Order a mistrial

o Or a court can do nothing.

 If it would not change the outcome of


the trial.
o Would your analysis of the lawyers obligations be any
different if the false testimony came out at a deposition rather
than at trial?

 It operates the same.

 They are equal in the eyes of this rule. (must take


reasonable remedial measures.) see comments 1 and
10

• Variation three

o Smith acknowledges, in advance that he will lie to save his


skin. The case is pending in a jurisdiction that permits a
criminal defendant to testify by way or narration.

o What is the narrative form of testimony?

 A narration of events spoken by the witness in response


to open ended questions. Lawyer asks open ended
questions and the witness tells the story (witness tells
the entire story without asking any more questions).

o How would the lawyer go about questions Smith?

 Lawyer can still ask questions in the normal manner up


until the point of the lie. Then the lawyer brings up the
open-ended question which elicits a narrative story.

o How is the narrative form harmonized with the text of RPC


3.3(a)(3)?

 One possible answer may be if you have not asked a


precise questions which elicits false testimony, then you
have not really elicited false testimony.

 How does comment [7] resolve the conflict between the


use of the narrative form and the first sentence of RPC
3.3(a)(1)?

• In certain jurisdictions The U.S. constitution


trumps RPC and the Constitution allows a person
to testify on ones own behalf even if the lawyer
knows the testimony is false.
• Variation Four

o A lawyer is appointed to represent the defendant in a criminal


matter. The defendant tells the lawyer that he committed the
crime for which he is charged.

 Given that the lawyer knows the client committed the


crime, can the lawyer…

• Make a motion to suppress the evidence? Yes

• Cross examine the prosecution’s witnesses? Yes

• Argue during closing that the prosecution has not


satisfied its burden of proof? Yes

 How does RPC 3.1 operate when a lawyer represents


the defendant in a criminal case?

• Even though the lawyer knows that his client is


guilty, you are still required to represent your
criminal defense client zealously.

• Overview of RPC 3.5

o RPC 3.5 preservees level playing field during trial and ensures
fair treatment of jurors after trial

 No attempts to influence through unlawful means

• Bribery, coercion/intimidation

 No ex parte communications absent authorization

• Temporary restraining orders okay

• Scheduling matters okay

 No post- discharge communication with


jurors/prospective jurors when…

• Communication is prohibited by law or court order

• Juror tells lawyer to leave her alone


• Communication involves misrepresentation,
coercion, duress or harassment

 No conduct intended to disrupt a tribunal

• Overview of RPC 3.6

o RPC 3.6 fosters decision-making based solely on the evidence

• The general prohibition ()sweeps broadly)

• Lawyer who is or who has been involved in


investigation or litigation of matter

• Cannot make a statement outside court

• That she knows reasonably should know

• Will de disseminated by means of public


communication

• Will have a substantial likelihood of materially


prejudicing an adjudicatory proceeding in the
matter

 The safe harbor

• Lawyer can say enumerated things with complete


impunity and without regard to consequences

 The “fighting fire with fire” exception

• Lawyer can make statement prohibited by (a) and


not permitted by (b) when necessary to

o Protect client from substantial prejudicial


effect

o Recent publicity initiated by someone other


than lawyer or client

 The Imputation rule

• Overview of RPC 3.7


o RPC 3.7 attempts to eliminates potential about the role of
lawyer (witness advocate rule)

 Lawyer cannot be advocate and witness at trial unless…

• Uncontested issues

• Nature and value of legal services in case

• Disqualification would work substantial hardship


on client

 No imputation within firm

• RPC 1.7 and RPC 1.9 may preclude others in firm


from representing client

o Here’s a question for all of you


professionally responsible kids – is the
language following “unless” really
necessary?

 It is there only to remind a lawyer


about conflict of interests, if they do
not remember the rules.

 But from a drafting there was no need


to put this language in it. The lawyer
should be able to know that there
might be a conflict and the language
is not necessary.

 For Monday, what might be a RPC 1.7


concurrent conflict of interest if the
lawyer is a witness and advocate at
trial.

F. Problem 28
The Verdict that May Be Tainted

Facts: Talley represented Δ in products liability case which ended up in huge verdict against Δ.
Talley asked one juror what happened and juror said all were in favor of Δ but one juror made
them all turn the other way and they later found out this juror was paid by competitor of Δ to do
so. Juror didn't want to testify so Talley recorded conversation secretly and also watched the pay
off but she was the only witness. She moved for a new trial.
A.The Ethics of Contacting Jurors After Trial
1. Was it proper for Talley to talk to the juror and investigate the jury's deliberatively
process? It is okay, however under rule 3.5(c)(1), after the case is over an attorney can't talk if:
prohibited by court order or law, it is coercion, duress, or harassment, or the juror doesn't want to
speak to the attorney. Many states require the attorney to file a notice of intention. (Florida
requires an attorney to file a notice of intention to talk to jurors).
2. Now that Talley believes that a juror violated his oath, what should she do? Under
3.3(b), if an attorney knows a jury was bribed, 3.3(b) and comment 12 states that the attorney
must tell the tribunal, no matter what the source--meaning, even if it is not a lawyer influencing
jury, there is still a duty to report. 3.3(b) requires the attorney to make remedial measures by
pulling in the other side and the judge and making the report.
The duty stated here is higher than the duty of confidentiality. The lawyer has a special
obligation b/c it would undermine the integrity of the system. He/she must take some remedial
measures. Given this, the lawyer should have told the judge b/c he/she tried to get the juror to
come forward (and the juror will not). The attorney would not be able to talk to the judge alone
anyway b/c that would be ex parte contact. Once the attorney has disclosed it to the judge, it is in
the judge's hands. The attorney's obligation ends once he/she reveals it to the court.

B. CONTACTING JURORS BEFORE TRIAL; COMPENSATION OF A WITNESS


1. Would it be improper for Talley's opponent to try to bribe one or more of the jurors
before trial? Yes (obviously).
3. If there is to be prohibition on pre-trial communication or contact with jurors, how
broadly should the rule be stated? Picking jurors are complicated matters so it would be
somewhat intrusive in voir dire process so allows information to be obtained but cannot for ex.
follow them outside of the court room.
Page 457 3(b) of text talks about whether we should be concerned about lawyer's
intruding into the juror's personal lives (as they did in the OJ Simpson case with these long
questionnaires). Prof says it might be a more efficient use of time to do this in terms of worrying
about the prospective contact with the juror. 3.5(a) you cannot influence a prospective juror.
3.5(b) talks about during the proceeding you cannot communicate ex parte unless the judge
allows you to. 3.5(c) does allow you to talk to jurors after the trial unless there is a court order.
4. Is it proper to pay a witness for the time it takes to prepare and give testimony? Under
rule 3.4(b) and comment [3] a lawyer cannot pay a witness for the content of his or her testimony.
It is routine for experts to be paid based on time they spend preparing and testifying in a case.
Experts get paid and this is okay so long as not a contingency fee. You can't give the expert an
incentive to testify based on whether your side wins. However, eyewitnesses cannot be paid
because would be paying for them to say certain things.
You can never being paying someone and suggesting that you are paying them for what
they are saying. You are paying them for the value of their time. How would a lawyer know if you
are paying for the content of the testimony if you pay for their expenses and time? You have to
see if it is reasonable i.e. 1 million for a 10 minute testimony is unreasonable. That is the only
way they will ever be able to figure this out. In Florida, lawyer can pay reasonable expenses for
loss of compensation for preparing for, time not at job, traveling for being the witness but not if
you testify I will pay you.

C. SECRET TAPE RECORDING


1. Was it professionally responsible for Talley to tape record when it became clear that
the juror would not commit his testimony to paper or repeat it to any other person? It is not a
violation of federal law. State laws may vary but some states allow what is called consensual
recordings (1 party to the conversation knows it is being recorded). In Florida you can video tape
without consent but not use an audio tape. Other states you can do both.
2. If Tally's tape recordings did violate state law, would her conduct also make her subject
to professional discipline or other remedies? If it is violating state law then it violates many rules
including rule 4.4(a) anti-harassing rule. Also under 8.4(b), this rule has a problem with her
violating state law - she can do it if it is going to break the law. There might be a violation of rule
8.4(c) whether or not a crime has been committed, if circumstances indicate the lawyer deceived
the person into believing there would be no taping.
Is this secret tape recording a violation of these rules? Even though the outcome is good the
process by how she went about to get it is not, under rule 4.4(a) and 8.4(b). 4.4(a) says that in
representing a client, a lawyer shall not use means that have no substantial purpose other than to
embarrass ... or burden a third person, or use methods of obtaining evidence that violate the legal
rights of such person. 8.4(b) says commit a criminal act that reflects adversely on the lawyer's
honesty. 8.4(c) says engage in conduct involving dishonesty, fraud, deceit, or misrepresentation.
Page 461 (e) of the text says that if it is not a violation of state law, it is no longer inherently
deceitful to tape record. If you say nothing then it is fine however if you say "this conversation is
between you and me" then you might be violating rule 8.4(c).

D. THE LAWYER AS A WITNESS AT THE CLIENT'S TRIAL


1. Is any ethical problem presented if Tally testifies at the hearing on her motion for a new
trial? Under rule 3.7, a lawyer cannot testify as a witness only with exceptions like lawyer sees a
car accident and ends up representing the injured. It raises problems and is not okay unless
substantial hardship for party but other person in firm can represent him unless there is a 1.7 or
1.9 conflict.
The general rule is that an attorney cannot be a lawyer and a witness. There are exceptions
under rule 3.7. The reason for the rule is because it could cause some confusion. In this case, it
is not a problem for her to be a witness.
2. Should a party be able to "waive" the protections of Model Rule 3.7? No. Not even if
client says okay-there is no waiving! Of course, even though you can't be the witness, someone
else from your firm can unless it would give rise to a 1.7 or 1.9 conflict. The lawyers testimony
might be in conflict with the client's testimony if it is adverse to the client. (E.g. Say I witness a
car accident and the client says, "can you be my lawyer?" you can't represent that person b/c you
are a material witness. However, someone from your firm can). However, if it is going to be a
conflict (for example, if you say, "he ran the red light," and the client says he didn't, there will be a
conflict), the conflict will be imputed to the firm unless you get consent from the client.

• Variation One

o Marian Talley asks a juror to come to her office to discuss the


verdict in a product liability case that went badly for her. The
juror agrees

 What additional piece of information would we need to


know before deciding whether Talley is acting
consistently with RPC 3.5(c)?

• Whether the juror consented to the meeting

• Variation Two
o The juror says that another juror brought everyone around to
his way of thinking but apparently, this juror works for a
competitor of Talley’s client and will be paid handsomely for
causing the jury to return a substantial verdict against Talley’s
client.

 Does Talley have any obligations under RPC 3.3(b) at


the conclusion of this conversation?

• Variation Three
o The juror won’t agree to sign an affidavit testifying to all of
this so Talley goes over the juror’s story a second time while
secretly recording it. She can only do so under the state
statute.
 The ABA’s current position about recording
conversations…every state has a statute stating one of
the following
• § Lawyer can record so long as doing so is
consistent with state law
o Ø Single consent versus dual consent
jurisdictions
• § Lawyer cannot state that the conversation is
not being recorded
o Ø Lawyer can do it secretly just can’t say
its not being done
• § Lawyer should not record a conversation with a
client absent the client’s consent.

Variation Four
o Tally sees the corrupt juror paid for her work and intends to
prepare an affidavit about what the other juror told her and what
she saw. She will use the affidavit to support a motion for a new
trial.
o Is Talley precluded from making the argument for a new
trial given her testimony is key to the motion? If looking at
the rule then you cannot say that witness and advocate
can be the lawyer and the witness. Jury will not
understand what is happening. If it is a bench trial then
the judge would know who is a witness and who and
advocate is.
 § What can Talley do to eliminate any possible
problem if the judge is unusually persnickety?
Invoke rule 3.7(b) and have another lawyer in the
firm represent the client.
o If Talley’s motion is successful, can she represent the client
at the second trial or does RPC 3.7 get in the way? Yes
because if she is successful in the the testimony on jury
temporing, there is no longer relevant to the new matter.

G. Problem 29 (didnt cover in class)


The Crusading Prosecutor

Facts: "Clean Gene," a prosecutor for county where state legislature started special prosecutors
branch which looks for misdeeds of state legislatures. He holds weekly press conferences to tell
reporters of his findings and answers questions. Newspapers are reporting rumors of those
legislators likely to be indicated, which Gene claims no knowledge of. Special unit is working to:
46 unpaid parking tickets by chairman of DMV; failure to report a sale of race track stock on
legislator's ethics form; house chairman for taking 5,000 bribe to kill a bill. Although it is not
normal to prosecute for small amounts of marijuana, Clean Gene found out one legislator had a
marijuana joint and decided to prosecute him for it.
A. MEDIA RELATIONS IN MODERN LITIGATION
1. What limits do the model rules place on White's use of press conference and other
public statements to tell the public about his office's work? Rule 3.8(f) imposes rules on the
prosecutor. The general rule is that, except for information purposes, a prosecutor cannot make
statements that will be prejudicial to the accused or poison the jury. However, 3.6(a) says that a
lawyer may not make judicial statements. Lawyers should be very careful.
4. Did "Clean Gene" White violate rule 3.6 when he said there were a lot of skeletons
under the bed of some state legislators? Rule 3.6(d) states that if he stated something specific,
then it would be problematic. With regard to the reports on indictments, rule 3.6(b)(2) says that
an attorney may state things that are in the public record. However, rule 3.6 comment 4 and 5
says that these are likely to prejudice this process. Here, what Gene spoke about has not
occurred and is not in the public record. It is a possibility therefore that it is not good; can't do
this.
He denounces the rumor mill - if there were leaks in his office that would be problematic.
Talks about the motion and what was stated in the motion, "The judge's actions show that
he is soft on public corruption." According to rule 8.2(a), the lawyer shall not make a statement
the lawyer knows to be false or have reckless disregard as to its falsity concerning the ... integrity
of a judge. If he says something that is reckless and if its unsubstantiated he will have a problem
with rule 3.1, which prevents the filing of a frivolous motion or legal argument.
"We have just indicted Kyle the Killer, who calls himself "Kyle the Killer." Rule 3.6(b)(5)
and (b)(7) states that this is normally done to warn the public. This is not to have any material
prejudice. There is a good purpose in disseminating information to catch someone.
(d). May White respond truthfully that the commentator is simply repeating a false charge
that one of the suspects has made several times? Rule 3.6(c) is the fairness provision of this
rule. The lawyer may make a statement that he feels is required to protect client from the
substantial or undue prejudicial effect.
6. Suppose White prosecutes the state legislator with a marijuana cigarette in his car. If
publicity will help the legislator beat the charge, may the legislator's lawyer advise him of that?
Does it make any difference whether the state legislator is a lawyer? The state can prosecute
something that they usually do not prosecute. Rule 3.8(a) says that you can't prosecute unless
you have probable cause. If they have pc then they are not stopped from doing this. This is only
what 3.8 is worried about (whether or not there is pc).

B. STANDARDS GOVERNING A PROSECUTOR'S DECISION WHETHER AND WHAT TO CHARGE


C. LIMITS ON PROSECUTORS INVESTIGATING DEFENSE ATTORNEYS
1. Should a prosecutor be permitted to subpoena office records of lawyers known to do
criminal defense work? Rule 3.8(e) says that a prosecutor shall not subpoena a lawyer unless
the prosecutor believes there is no other feasible alternative. Defense lawyers are trying to say
that this is a violation of an ethics rule since they could not get around it as a rule of procedure.

D. FOCUSING ON THE FEES OF CRIMINAL DEFENSE COUNSEL


1. May the government seize the fees of privately retained defense counsel? The IRS is
trying to track the source of the money. If I am paid a fee, I can get in trouble w/ the law if I know
that the money is coming from an illegal act. This relates to 3.8 b/c the IRS requires the attorney
to report it. The general rule is that a prosecutor does not subpoena a lawyers records.
However there are exceptions (payments from client to lawyer in excess of $10k).

H. Problem 30 (Didint cover in class)


The Duty to See Justice Done
(page 486)

Facts
Attorney York represented insurance co, in workman's comp for lady who said she was sexually
assaulted by a black man in the course of her work duties. She was scared of all black men after
this and couldn't tell one from another. This all came from psychiatric records which were only
available to settle civil claim. Brooks, identified by her in line up, had corroborating alibi but jury
went with her testimony and he was sentenced to life. York, after hearing about the psychologist
reports in the course of the workers' comp claim, believes that an innocent man may be in prison
for the rest of his life unless he acts, but Brooks is not his client (the insurer is) and he does not
know how to proceed.

[Note: we said before the rules are at the lowest level of conduct you must achieve and here this
is saying a higher standard to reach].

A. Duties of a Lawyer Who Learns an Injustice May Have Been Done


1. To whom does the workers' compensation attorney owe duties in this situation? To the
insurance carrier and the hotel, York undoubtedly owes a duty of confidentiality and a duty of
loyalty. Rule 1.6(a) imposes a duty of confidentiality on him b/c the information about the victim
and her probably inaccurate i.d. relates to his representation of the insurer in the workers' comp
claim, and it does not matter what the source of the information is. Rule 1.9(c) states that a
lawyer who has formerly represented a client in a matter or whose present or former firm has
formerly represented a client in a matter shall not thereafter: (1) use info relating to the
representation to the disadvantage of the former client except as these rules would permit or
require w/respect to a client, or when the information has become generally known; or (2) reveal
info relating to the representation except as the rules would permit or require w/respect to a client.
Therefore, unless the insurer gives informed consent, the attorney cannot reveal any
information relating to the representation. However, the attorney may reveal those things listed
under the exceptions in rule 1.6(b). We may have a 1.6(b)(1) exception here b/c the man who
was wrongly convicted and was sent to prison for life will probably suffer serious bodily harm
(although this is not usually the situation that 1.6(b)(1) speaks to). This attorney has information
that may prevent a man from suffering substantial bodily harm in prison.
2. Might the attorney have a duty to the rape victim not to make her psychiatric studies
public? Rule 4.4 governs respect for rights of third persons, and states that a lawyer shall not use
means that have no substantial purpose other than to embarrass, delay, or burden a third person,
or use methods of obtaining evidence that violate the legal rights of such person. Thus, the
question is whether revealing the psychiatric studies violates the victim's legal rights. If the
attorney has a 1.6(b)(1) exception, however, he has the right to reveal this information. Plus,
these reports were already furnished during discovery for the purpose of settling the workers'
comp suit. However, she revealed that information for the limited purpose of settling.
3. Does the attorney have any duty to the court that tried the case, or to the man he
believes is wrongly convicted? Rule 3.3(a) does not apply b/c the attorney was not involved in
the criminal proceeding. However, 3.3(b) states that a lawyer who represents a client in an
adjudicative proceeding and who knows that a person intends to engage, is engaging, or has
engaged in criminal or fraudulent conduct related to the proceeding shall take reasonable
remedial measures, including, if necessary, disclosure to the tribunal. Here, York knows that
Brooks has engaged in conduct that was fraudulent. The flip-side of that , however, is that
Brooks may not have intended to commit a fraud--she truly convinced herself that the man on trial
was the man who raped her. Comment 12 states that lawyers have a special obligation to protect
a tribunal against criminal or fraudulent conduct that undermines the integrity of the adjudicative
process; however, the situations this comment speaks to (bribery, destroying or concealing
documents/evidence) do not seem to apply here.
Does anyone have an obligation to listen to York? Would the former prosecutor or
defense counsel have such an obligation, for example? How about the trial judge? Start with the
judge (see cannon 1 on page 586) to maintain the integrity of the judiciary he should do
something.
The defense lawyer (assume the appeal have been exhausted); If not exhausted, then
his duties continue because he is still considered as representing the defendant. There is a
reasonable time limit after which the defense attorney's duty or obligation to rectify false evidence
or false statements of law and fact expires. But say that the client gives informed consent and
allows the attorney to reveal this information to the defense attorney so that the innocent man can
be exonerated. If the defense attorney does nothing, is he incompetent under 1.1? Maybe.
Prosecutor (rule 3.8 and page 507): seek justice not merely to convict-he or she
when he finds out of things should seek remedies for. In real case based on this all 3 of them
said they didn't feel they had a duty and were not supposed to do something and were not
punished for inaction. This shows rules really are at the low standard (attorneys and judges see
these rules as floors not ceilings).

B. Concerns About the Reliability of Results in Our Legal System


1. DNA has helped to exonerate the innocent.

C. The Obligation, if any, to Respond to Concerns About an Injustice


1. Post-conviction remedies; may be an obligation in that regard.
2. What if no one listened to York? Should he try to interest "60 Minutes?" Oprah?
Geraldo? The news media's ability to raise questions and stimulate changes has made it a
serious venue for many accused and convicted people. The news media has often become the
"court of last resort" in the U.S. today. [In real life, the attorney did go the judge, prosecutor, and
defense attorney, and none of them did anything. Reinforces that they see the rules as floors and
not ceilings].

VII. The Delivery of Legal Services


A. Problem 31
Marketing Professional Services
(page 496)

Generally: The Model Rules provisions can be understood as following the Supreme Court's lead
in distinguishing advertising from solicitation. Careful reading of the case law has prompted the
ABA to use the concept of "direct contact" to indicate the area in which regulation is most
stringent. However, the ABA also limits advertising in the public media.

A. The Constitutional Context of the Regulation of Lawyer Marketing


1. What makes the state-regulated lawyer advertising a matter for the federal courts?
The first amendment; freedom of speech. Bates v. Arizona: The court found the postulated
connection b/t advertising and the erosion of true professionalism to be severely strained. The
argument against advertising was that it was considered undignified. The court disagreed and
found a legitimate reason for allowing advertising w/regard to legal services. However,
advertising that is false, deceptive, or misleading of course is subject to restraint. Time, place,
and manner may also be restricted as it is w/other types of speech.
Rule 7.1 states that a lawyer shall not make a false or misleading communication a/b the
lawyer or the lawyer's fees. A communication is false or misleading if it contains a material
misrepresentation of fact or law, or omits a fact necessary to make the statement considered as a
whole not materially misleading. Comment 2 states that truthful statements that are misleading
are also prohibited. A truthful statement is misleading if it omits a fact necessary to make the
lawyer's communication considered as a whole not materially misleading. A truthful statement is
also misleading if there is a substantial likelihood that it will lead to a reasonable person to
formulate a specific conclusion a/b the lawyer or the lawyer's services for which there is no
reasonable foundation.
Comment 3 states that an advertisement that truthfully reports a lawyer's achievements
on behalf of clients or former clients may be misleading if presented so as to lead a reasonable
person to form an unjustified expectation that the same results could be obtained for other clients
in similar matters w/o reference to the specific factual and legal circumstances of each client's
case. The inclusion of an appropriate disclaimer or qualifying language may preclude a finding
that a statement is likely to create unjustifiable expectations or otherwise mislead a prospective
client. (Note use of the word "may," which means such a disclaimer might not always be enough
to cure a misleading advertisement).
The important thing to note is that just b/c a statement is truthful does not mean it is not
misleading. The problem with disclaimers is that average members of the public

B. When Lawyer Advertising Becomes In-Person Solicitation


1. There is a difference between solicitation and advertising. Rule 7.3(a) states that a
lawyer shall not by in-person, live telephone (as opposed to audio-dial), or real-time electronic
contact (i.e. chat rooms; but emails are ok) solicit professional employment from a prospective
client when a significant motive for the lawyer's doing so is the lawyer's pecuniary gain, unless
the person contacted: 1) is a lawyer, or 2) has a family, close personal, or prior professional
relationship with the lawyer. [Note: these exceptions allow attorneys to contact past clients to
solicit business].
Comment 3 states that the use of general advertising and written, recorded, or electronic
communications to transmit information from lawyer to prospective client, rather than direct in-
person, live telephone, or real-time electronic contact, will help to assure that the information
flows cleanly as well as freely. The contents of advertisements and communications permitted
under rule 7.2 can be permanently recorded so that it cannot be disputed and may be shared with
others who know the lawyer. This potential for informal review is itself likely to help guard against
statements and claims that might constitute false and misleading communications, in violation of
rule 7.1.
This is the difference between advertising and solicitation: advertising can be "checked,"
while solicitation cannot. What might be said during solicitation often is not recorded in any way
and cannot be proven.
Ohralik: Unlike a public advertisement, which simply provides information and leaves the
recipient free to act upon it or not, in-person solicitation may exert pressure and often demands
an immediate response.
In re Primus:
Comment 4 to 7.3 states that 7.3(a) is not intended to prohibit a lawyer from participating
in constitutionally protected activities of public or charitable legal-service organizations or bona
fide political, social, civic, fraternal, employee, or trade organizations whose purposes include
providing or recommending legal services to its members or beneficiaries.
Note, however, rule 7.3(b), which states that a lawyer shall not solicit professional
employment from a prospective client by written, recorded, or electronic communication, or by in-
person, telephone, or real-time electronic contact even when not otherwise prohibited by 7.3(a) if:
1) the prospective client has made known to the lawyer a desire not to be solicited by the lawyer;
or 2) the solicitation involves coercion, duress, or harassment (e.g. sending a client 250 gazillion
emails). [So just b/c the attorney is ok under 7.3(a) does not mean he can solicit! Must be ok
under 7.3(b) too!]
Comment 5 goes so far as to say if, after sending a letter or other communication to a
client as permitted by 7.2 the lawyer receives no response, any further effort to communicate
w/the respective client may violate 7.3(b). [Thus, the client does not have to affirmatively tell the
attorney not to solicit him/her; not responding to an initial solicitation is enough to imply that the
lawyer should discontinue solicitation].
2. Do Jerry Harold's phone calls proposing a "legal checkup" present issue under Rule
7.3(a)? No, no issues, b/c 7.3(a) states allows an attorney to solicit business from current or
former clients. However, if he does not get a response, he cannot continue to solicit those
clients.
b. Is it proper for a lawyer to print color brochures extolling the law firm's virtues?
Is there any question the lawyer's motive would be to bring more business to the law firm? Do
the words "Advertising Material" have to appear on the brochure itself? Rule 7.3(c) states that
every written, recorded, or electronic communication from a lawyer soliciting professional
employment from a prospective client known to be in need of legal services in a particular matter
(e.g. when you get traffic ticket and then get tons of ads in the mail for attorneys who can get you
out of it w/o points on your license) shall include the words "Advertising Material" on the outside
of the envelope, if any, and at the beginning and ending of any recorded or electronic
communication (unless one of the exceptions in 7.3(a) apply).
Comment 7 states that certain communications to be marked "Advertising Material" does
not apply to communications sent in response to requests of potential clients or their
spokespersons or sponsors. General announcements by lawyers, including changes in
personnel or office location, do not constitute communications soliciting professional employment
from a client known to be in need of legal services w/in the meaning of rule 7.3.
c. May lawyer hold seminars or make public speeches a/b the law and legal
problems? May a lawyer represent a person who comes for advice after attending one of these
programs? Would you be troubled if the lawyer distributed his/her business card at the events?
There is nothing wrong with this under comment 4, which states that a lawyer may participate in
activities of public organizations, or bona fide social or civic organizations. It is ok to disseminate
information. Plus, in the hypo the Professor gave, AARP invited the attorney to come to their
meetings and speak about wills. If the attorney is invited, he is not soliciting business. It is also
ok for the attorney to say, "Please see me after the meeting if you have any questions," because
it leaves the prospective client free to leave. They will not feel forced to come up to the lawyer. It
is also ok for the lawyer to leave his business cards by the door so that people may take them.
C. Continuing Issues in the Field of Lawyer Advertising
2. Do illustrations in a lawyer's advertisement have the same constitutional protection as
the text message? Zauder: Yes. Illustrations are protected speech.
3. Should a state be able to require lawyers to tell clients they will have to pay litigation
costs even if they lose their case? Zauder: There are material differences b/t disclosure
requirements and outright prohibitions on speech. In requiring attorneys who advertise their
willingness to represent clients on a contingent-fee basis to state that a client may have to bear
certain expenses even if he loses, states have required them to provide somewhat more
information than they might otherwise be inclined to present.
Professor says that if the attorney does not put in a disclaimer making clear the
difference b/t attorney's fees and the ordinary costs associated with filing a lawsuit (filing fees,
administrative fees, etc.), then courts may hold that the client does not have to pay anything if
he/she loses at trial, b/c a reasonable person would have derived that to be the agreement from
an advertisement that says, "If you lose, you don't have to pay me anything."
5. Should a lawyer be entitled to make claims as to her quality? Her won-lost record?
How about publishing client testimonials? Under rule 7.1, the information cannot be misleading,
and under comment 3, information is misleading if it creates in a reasonable person an unjustified
expectation that the same results could be obtained for other clients in similar matters w/o
reference to the specific factual and legal circumstances of each client's case. Thus, while it
might be true that an attorney has never lost a jury trial, that information is misleading if the
attorney has never done a jury trial.
6. May a lawyer use a memorable trade name or internet web address such as "The
Winning Team" or "suethebums.com?" Rule 7.5(a) states that a lawyer shall not use a firm
name, letterhead, or other professional designation that violates rule 7.1 (i.e. it cannot be false or
misleading). A trade name may be used by a lawyer in private practice if it does not imply a
connection w/a government agency or w/a public or charitable legal services organization and is
not otherwise in violation of 7.1.
So, it would seem that a lawyer may use a web address such as "suethebums.com," b/c
it is not false or misleading; it is just distasteful. "Winning Team" might be misleading.
7.Does television advertising justify special regulation?

D. Continuing Issues of In-Person Solicitation by Lawyers


1. Is the personal note from Jerry ok? Yes, as long as it is marked "Advertising Material."
The prospective client is free to throw the letter away and not respond. However, if the person
does not respond, at that point it would be improper for Jerry to "follow up," b/c doing so would be
akin to in-person solicitation.
Sharpero: targeted mail (like the traffic ticket example); this is okay so long as it is not
misleading.
4. Florida prohibited lawyers for potential plaintiffs from sending any targeted direct mail
w/in 30 days of an accident. Model rules do not require a waiting period, but under 7.3(b), if the
person doesn't want to be contacted, then the attorney cannot contact them.

PROBLEM 31
Overview of RPC 7.1
o How would you describe the basic obligation imposed by RPC
7.1? establishes a general proposition that lawyers are required
to follow when talking about themselves or their services…
cannot make false or misleading statements. cannot mislead
affirmatively or by withholding pertinent information.
o Does Rule 7.1 sound an awful lot like RPC 3.3(a), RPC 4.1
and RPC 8.1? similar but different.
o How does RPC 7.1 interact with RPC 7.2 through 7.5? it is
an introduction to the other rules. Must speak truthfully.
o Comment (3) says that truthful statements about lawyer’s
achievements can be misleading because they can create
an unjustified expectation that the prior achievements can
be replicated.
 § Assume a lawyer puts an advertisement in the
local paper and truthfully states that she obtained
the two largest jury verdicts in state history 0 one for
$40 million and the other for $35million.
• Ø What is the potential problem with this
advertisement, even if the lawyer really did
secure the two largest jury verdicts in state
history? Problem is not that it is untruthful, the
lawyer is conveying the wrong message to the
client. If the lawyer did it for someone else
they could do it for me.
• Ø Can lawyer who wants to make a claim
about her two jury verdicts cure the problem
by stating “individual results may vary”? If you
include a disclaimer then it would be ok.
Something must be said that individual results
will vary.
o Comment (3) says that an unsubstantiated comparison of
one lawyer’s services and another lawyer’s services is
misleading when a reasonable person would conclude that
the comparison can be substantiated.
 Assume a lawyer handles personal injury cases.
• Ø Can the lawyer put an advertisement in the
local paper in which she states that she is the
best personal injury lawyer in town? No
because there is no objective standard to be
rated.
• Ø Can the lawyer put an advertisement in the
local paper in which she states that she is the
only personal injury lawyer in town rated
“superb” by the lawyer rating service AVVO?
Yes because you can go to the AVVO website
and substantiate the fact, so it is not
misleading.
o RPC 7.1 prohibits lawyers from making a statement that requires
the disclosure of some additional fact that is necessary to make
the statement, considered as a while, not misleading
o Assume a lawyer handles 25 straight uncontested divorces.
o § Can the lawyer put an advertisement in the local paper
in which she claims to have won her last 25 cases? No,
Victory is assured because the matter is uncontested.
Overview of RPC 7.2
o How would you summarize RPC 7.2(a)? a lawyer may advertise
services through written, recorded or electronic communication,
including public media.
o Which of the following qualify as a formal in which lawyer
advertising can appear?
o Television and radio? Yes
o Newspapers? Yes
o Mass mailings? Yes
o Pre-recorded telephone messages? Yes
o E-mail? Yes
o How would you describe the basic obligation of Rule 7.2(b)? can
pay for reasonable costs, but cannot pay for recommending
services.
o Assume a lawyer pays an ambulance driver $100 for ever
auto accident victim who becomes a client as a result of a
referral by the ambulance driver? Yes because he is giving
something of value to someone for recommending his
services.
o Has the lawyer violated RPC 7.2(b)? Yes.
o lawyer can “pay for a referral in certain situations
o Payments for advertising
o Payments for participating in a lawyer referral service
o Payments for all or part of another lawyer’s practice
o Reciprocal referral agreements
o Can be lawyer to lawyer to non-lawyer professional
o Trusts and estates lawyer to accountant (a vice versa)
o Personal injury lawyer to corporate lawyer (and vice versa)
o Cannot be exclusive
o Client must informed about existence and nature of
agreement
Overview of RPC 7.3
o How would you describe the basic obligation imposed by RPC
7.3(a)? a lawyer shall not by in-person, live telephone or real-
time electronic contact solicit professional employment form a
prospective client when a significant motive for the lawyer’s
doing so is the lawyer’s pecuniary gain.
o What four categories of individuals can a lawyer properly solicit?
Lawyer, family members, someone who is a close personal
friend, someone with whom you have a prior relationship.
o Why do you suppose lawyers can solicit work from these
folks? Comment (4) says lawyer will not be overreaching
(pressuring clients to hire the lawyer) or burden the client.
o To whom is the rule referring when it talks about persons
with whom the lawyer has a “prior professional
relationship”? Both current and former clients, because
the lawyer has prior professional relationship with them.
They could be protecting current and former clients if the
law should change and effect the client.
o How would you describe the basic obligation imposed by RPC
7.3(b)? with respect to individuals that lawyer may properly
engage in solicitation, this rule takes away that solicitation right
is the client has made know to the lawyer a desire not to be
contacted or the solicitation involves coercion, duress or
harassment. (Includes everyone, not just people in 7.3(b)).
o You can’t advertise or solicit when the target has said he
wants to be left alone
o You can’t advertise or solicit when your efforts
o How would you describe the basic obligation imposed by RPC
7.3(c)? lawyer must include “advertising material” (in written,
recorded or electronic communications) when soliciting a client
in need of legal services in a particular matter. Do not have to
include advertising material is (a)(1) or (a)(2).
o Does RPC 7.3(c) apply to targeted mailings? Yes whenever
a lawyer knows that they are in need of professional
services, they must include “advertising material”.
o Does RPC 7.3(c) apply to untargeted mailings? No because
it is not targeted to a specific class that is in need of legal
services in a particular matter.
Overview of RPC 7.4
o What does RPC 7.4(a) allow lawyers to do? A lawyer may
communicate the fact that the lawyer does not does not practice
in particular fields of law.
o What does RPC 7.4(d) allows lawyers to do? A lawyer can say
they are certified as a specialist in a particular field of law only if
the lawyer has been certified by an organization that has been
approved by an appropriate state authority or that has been
accredited by the American Bar Association and the name of the
certification is clearly identified.
o Does RPC 7.4 prohibit a lawyer from characterizing herself
as a specialist in a particular area of practice? You can say
that you are a specialist, but only specified as a certified
specialist if you are in fact certified.
 What risk does a lawyer run if she characterizes
herself as a specialist but has not received a
certification from an approved or accredited
organization? Imposes a high standard on herself.
The client will be expecting a higher quality of
service. Nothing wrong on its face, but Rule 7.1 may
create a problem because you have to be truthful in
advertising (may challenge the claim).
Overview of RPC 7.5
o How would you describe the basic obligation imposed by Rule
7.5(a)?
o Assume Smith and Jones form a law firm.
 Can they call the firm Smith & Jones? Yes so long as
they are practicing together.
 Can they call the firm “Berger & Frankfurter”
because these are the two former justices that they
most respect? No because comment 1 says you can
include current or former lawyers, but you cannot put
others in the firm name.
 Can they call the firm “Best Damn Firm Ever”? No
because they are subject to Rule 7.1 and you cannot
prove this.
 Can Smith continue to call the firm Smith & Jones
even after Jones even after Jones’ death? Yes
because comment one. After death of partner you
can.
o How would you describe the basic obligation imposed by Rule
7.5(b)? a law firm with offices in more than one jurisdiction may
use the same name or other professional designation in each
jurisdiction, but identification of the lawyers in an office of the
firm shall indicate the jurisdictional limitations on those not
licensed to practice in the jurisdiction where the office is located.
o Assume Smith is admitted to practice in Florida and Jones
is admitted to practice in Georgia.
 Can Smith and Jones form a law firm and call their
firm Smith & Jones in both states? Yes but they must
say where they are admitted to practice law.
 What must Smith and Jones say on the firm’s
stationary and on any other listing of the lawyers in
the firm to ensure compliance with Rule 7.5(b)?

Overview of RPC 7.5 (contd.)


• How would you describe the basic obligation imposed by RPC
7.5(c)? if someone in a law firm whose name is part of the firm
name is elected to public office there are some instances where
the law firm cannot use that name while they are in office; the
public officials name must be removed during any substantial
period of time during which the lawyer is not actively and
regularly practicing law.
o Assume Smith and Jones form a firm and call the firm
“Smith & Jones”
 Can smith continue to practice as “Smith & Jones”
after Jones is elected to the U.S. Senate? Probably
not because Jones’ obligation as a senator would
restrict his ability to actively and regularly practice
law while simultaneously serving as a US Senator.
(would be different if he served on the city council of
a small town and would probably not be a full time
job)
 What can they do if Jones is not re-elected and
returns to practice with Smith? They could return to
using the name Smith & Jones if he leaves the senate
and returns to practicing law.
• Variation One
o Jerry Harold has just moved to a new community and so he
places the advertisement on page 514 in a local paper
 Did jerry create a potential problem by stating that…
• He is a “doctor of law” or that his “doctor’s
degree in law is the… degree a lawyer normally
earns”? this statement on its face is technically
true. But there are other degrees someone
could get and therefore it is not technically and
thoroughly true. See comment [30 of 7.1
• He served his country for three years as a
“Judge advocate in the Army”? a phrase that
non lawyers might consider to mean that he
was an actual judge as it implies he is not only
a lawyer but a judge as well.
• He does not claim to be the best lawyer in the
state but some of his former clients have
agreed to serve as references? There is
nothing that prohibits former clients from
providing references. but when he says he is
not the “best” lawyer he is implying that he
may be cloise to the best lawyer. And there is
no way to prove the lawyers assertion.
• He is “Unusual in one respect” because he
charges a single hourly rate for all matters.
Conveying that he is the only lawyer in the
state that charges a single rate for anything.
Can do a review to determine whether this is a
true assertion, and the probability is that there
may be other lawyer who charge one hourly
rate for all their work so it is very likely a false
assertion. But if true then this would be a true
assertion because it can be proven.
• His clients have found that his $95 per hour fee
saves them “a great deal in legal fees,
particularly in routinematters like real estate
and probate. It may, it could be misleading. A
lawyer who charges a low hourly rate but if the
lawyer makes up the difference by billing a
larger number of hours then the clients would
be in the same economic position, as if the
clients went with a lawyer who did the work
faster but for a lower price. In regards to real
estate and probate matter it is difficult to say it
is a great deal because not eery real estate
and probate matter is the same. Some are
more complicated than others.
 Was Jerry required t include a legend indicating that
the advertisement is “advertising material”? in a
newspaper No. because he is directing this at people
he doesnt know and who may or may not even need
his legal help (see RPC 7.3(c)) (it would be different if
you are directing the advertisement at people you
know will need your legal services)
 Was Jerry required to include his office address in the
advertisement?
• Variation 2
o Jerry Telephones each of his former clients who is over the
age of 50 te inquire whether they are in need of a review of
their
 Has Jerry Violated any RPC by making the call?
Whenever jerry is communicating with a former or
current client even if soliciting new business he is not
prohibited by 7.3(a).
 Is Jerry required to leave these folks alone if they tell
him they are not interested in any additional
services? Under 7.3(b)(1) limits there ability to solicit
even former or current clients if they say they dont
want their services then the lawyer may not continue
to solicit that business.
• Variation 3
o Jerry learns of an airplance crash and decides to send a
note to the victims or their survivors expressing his
personal sympathy for what the airline’s possible
negligence has forced them ot bear.
 Is Jerry at risk of violating aby RPC if he sends the
note?
• Are any of the following RPC’s implicated- 7.1,
7.2, 7.3? yes all of them would be implicated
because it can be infered that the reason for
the communication is for business and
thereforem akes it an advertisement. 7.3(a)
only applies face to face, over the phone, or
real time electronic chat. And since he is not
doing it in these modes 7.3(a) doesnt apply.
7.1 applies but he has not said anything false
or misleading. 7.2 applies because this is a
written communication but he has not
problems so therefore 7.3 might apply but it is
not an issue because it is not face to face, over
the phone, or instant chat.
• Is Jerry required to place the legend
“advertising material” on the outside
envelope? Yes, because it is directed to a
person that the lawyer knows is in need of
legal services. 7.3(c) he would have to comply
with advertising material. Someone could
maybeattemptt oclaim that 7.3(b) might apply.
 Jerry also plans to follow up with a personal visit to
the victim or survivor, during which he plans to say
“its up to you to decide whether to retain me, but if
you do, Ill be here for you”
• Is jerry at risk of violating any RPC if hemeets
with the victims or their survivors? Yes, 7.3(b)
cant solicit in person if it involves coercion,
duress, or harrassment. Jerry cant even have
this encounter under 7.3(a) solicit employment
in person because none of these people qualify
for any exceptions to the no face to face
solicitation rule. If Jerry proceeds without
prompting by the victims he would violate
7.3(a).
Variation 4
• You are a new lawyer. Your firm gives you a box of business
cards that you have with you in your car. You drive by a multi-car
accident. There is blood and gore everywhere (and lots of
prospective clients as well). You grab a fist full of business cards
and fling them out the window, letting the wind do the rest.
o Have you violated any RPC? While it is in bad taste No RPC
7.3(C) violation so it would probably be allowed since the
lawyer didnt hand them to each person personally there
was no pressure or anything on the victim to accept since
they can merely throw them away. The client is never
confronted with a lawyer pressured to sign up with a
prospective lawyer, and a person can pick up your card
and discard it.

B. Problem 32 (did not cover in class)


The Ethics of Referral to A Specialist
(Page 517)

Anytime you think of a referral, you must think of it in conjunction with rules 7.2(b) v.
1.5(e). 7.2(b) states that a lawyer shall not give anything of value to a person for recommending
the lawyer's services except that a lawyer may (1) pay the reasonable costs of advertisements or
communications permitted by 7.2; (2) pay for
7.2(b) means that a lawyer cannot be paid for finding a case and "dumping" it on another
lawyer. Straight finders' fees are invalid! Lawyers may have reciprocal agreements (in which
lawyers exchange referrals) as long as the client knows about the agreement.
1.5(e) speaks about a division of fees, and states that a division of fee b/t lawyers who
are not in the same firm may be made only if: (1) the division is in proportion to the services
performed by each lawyer or each lawyer assumes joint responsibility for the representation; (2)
the client agrees tot eh agreement, including the share each lawyer will receive, and the
agreement is confirmed in writing; and (3) the total fee is reasonable.
Professor says that the reason for subsection (1) is so that each lawyer will be held liable
for his/her own malpractice. When lawyers make these agreements, they are agreeing to
substitute their own lawyering abilities for the other lawyer's. Comment 7 states that joint
responsibility for the representation entails financial and ethical responsibility for the
representation as if the lawyers were associated in a partnership. A lawyer should only refer a
matter to a lawyer whom the referring lawyer reasonably believes is competent to handle the
matter under 1.1.

v. 7.2(b)(4) + 5.4(a)

Facts
Ramirez has represented Peron family for several years. Peron's son was injured in car accident
and family wants to sue but Ramirez isn't familiar with PI work worked out deal with other attorney
to take 1/3 of the settlement from this attorney's rep or 10K whichever is less as the finders' fee.

A. A Lawyer's Duty When a Matter Requires New Skills or Raises Unfamiliar Issues
1. Does Ramirez have an ethical obligation not to handle this case entirely by himself?
Rule 1.1 states that a lawyer shall provide competent representation to a client. Competent
representation requires the legal knowledge, skill, thoroughness, and preparation reasonably
necessary for the representation. Comment 1 states that in many instances, the required
proficiency is that of a general practitioner, Expertise in a particular field of law may be required
in some circumstances.

B. Calling a Lawyer a "Specialist" In One or More Fields of Practice


1. Should the law be concerned a/b lawyers calling themselves specialists? Rule 7.4(d)
states that a lawyer shall not state or imply that a lawyer is certified as a specialist in a particular
field of law, unless: (1) the lawyer has been certified as a specialist by an organization that has
been approved by an appropriate state authority or that has been accredited by the ABA; and (2)
the name of the certifying organization is clearly identified in the communication.
The reason for this rule is b/c "specialist" implicates a higher level of competence. Under
comment 3, certification signifies that an objective entity has recognized an advanced degree of
knowledge and experience in the specialty area grater than is suggested by general licensure to
practice law (this is the difference between saying, "Specialist" and "Practice Limited to [area of
law]). Under comment 1, a lawyer is generally permitted to state that the lawyer is a "specialist,"
practices a "specialty," or "specializes" in particular fields, but such communications are subject
to the "false and misleading" standard applied in rule 7.1 to communications concerning the
lawyer's services.

D. Alternatives to Referral Fees Designed to Achieve Similar Benefits


4. Should sharing legal fees w/a recognized lawyer referral service be similarly
prohibited? Rule 5.4 governs professional independence of a lawyer, and subsection (a)(4)
states that a lawyer or firm shall not share legal fees with a non-lawyer, except that a lawyer may
share court-awarded legal fees w/a nonprofit organization that employed, retained, or
recommended employment of the lawyer in the matter.

Sharing Legal Fees 


In 1.5(e) you can do this but then each lawyer assume joint-responsibility of the representation (if
you want a division of fees)
7.2(b) – no straight referral fees
1.5(e)

When you see someone trying to split a fee with another lawyer, you have [7.2(b) v. 1.5(e)] v.
7.2(b)(4) + 5.4(a)

Question 3) What is the problem with sharing a fee with a non lawyer? – it may disturb the
lawyer’s independent and professional judgment as to what to do for this case and client. 
Could interfere with the professional relationship. Rule 5.4(a)
Rule 5.4(a) kicks in when you see a non lawyer

Problem 33: Roles and Responsibilities in a Modern Law Firm


(page 530)

Facts
Large firm-managing partner knows of one associate who lies about hours he is billing but since
client isn't complaining either is she. Sara is managing partner and associate Able.

A. The Obligations of Supervisory Lawyer and Those They Supervise


1. Is there any doubt that Able would be subject to discipline for billing hours he had never
worked? Of course not! Rule 1.5(a) states that a lawyer shall not make an agreement for,
charge, or collect an unreasonable fee or an unreasonable amount for expenses. It is
unreasonable to bill for hours that were never worked! Additionally, he violated rule 8.4(c), which
states that it is professional misconduct for a lawyer to engage in conduct involving dishonesty,
fraud, deceit, or misrepresentation. It is certainly dishonest to bill a client for hours never worked.
2. What responsibility did Smart have to assure that Able did not engage in such dishonest
billing? Rule 5.1(a) states that a partner in a law firm, and a lawyer who individually or together
w/other lawyers possesses comparable managerial authority in a law firm, shall make reasonable
efforts to ensure that the firm has in effect measures giving reasonable assurance that all lawyers
in the firm conform to the Rules of Professional Conduct. Comment 2 states that 5.1(a) requires
lawyers w/managerial authority w/in a firm to make reasonable efforts to establish internal policies
and procedures designed to provide reasonable assurance that all lawyers in the firm will conform
to the Rules of Professional Conduct. Such policies and procedures include those designed to
detect and resolve conflicts of interest, identify dates by which actions must be taken in pending
matters, account for client funds and property, and ensure that inexperienced lawyers are
properly supervised. Basically, the firm must have rules in place w/which to "check" their
employees. Similarly, 5.3 governs non-lawyer conduct (such as that of law student clerks).
Under 5.1(c), a lawyer shall be responsible for another lawyer's violation of the Rules if:
(1) the lawyer orders or, w/knowledge of the specific conduct, ratifies the conduct involved; or (2)
the lawyer is a partner or has comparable managerial authority in the law firm in which the other
lawyer practices, or has direct supervisory authority over the other lawyer, and knows of the
conduct at a time when its consequences can be avoided or mitigated but fails to take reasonable
remedial measures.
b. As far as we know, Smart did not "order" the over-billing or even "ratify" it, but does
that mean she is off the hook? No. She should be on the hook, because under 5.1(c) she found
out that the associate was over-billing and did not take reasonable remedial measures. She
could have corrected the bill, talked to the client, etc.
3.Is Andy Howe, Smart's co-managing partner, equally responsible for Able's over-billing? If
Howe is responsible, it would only be virtue of 5.1(a), b/c he is a managing partner and therefore
is responsible for whether the firm had some policy or procedure in place to catch this type of
conduct. On the other hand, a named-partner cannot be expected to know about what every
associate does and what goes into every client's bill. Thus, it seems that the only way to "get"
him for misconduct is to point out what he did not do. Perhaps they could have had some kind of
internal auditing system (e.g. if most associates worked an average of 2500 billable hours and
Able worked 3500, a red flag would go up). In re Wallman: other partner is converting funds,
court suspended the other partner for failing to catch and correct the problem. 5.1 imposes an
obligation to know.
c. Would it be enough for Howe to show that the firm's orientation video contains a two minute
section in which Howe tells the new lawyers that the firm expects them to bill honestly?
4. Would Able's liability be different if over-billing was not his idea; he was just following
orders? Probably not. There is no "good soldier" defense to commands by partners that can only
be answered one way (i.e. the only way to comply is to blatantly violate the rules). Rule 5.2(a)
states that a lawyer is bound by the Rules notwithstanding that the lawyer acted at the direction of
another provision. However, subsection (b) states that a subordinate lawyer does not violate the
rules of professional conduct if that lawyer acts in accordance w/a supervisory lawyer's
reasonable resolution of an arguable question of professional duty. Comment 2 states that when
lawyers in a supervisor-subordinate relationship encounter a matter involving professional
judgment as to ethical duty, the supervisor may assume responsibility for making the judgment. If
the question can reasonably be answered only in one way, the duty of both lawyers is clear and
they are equally responsible for fulfilling it. However, if the question is reasonably arguable,
someone has to decide upon the course of action. That authority ordinarily reposes in the
supervisor, and a subordinate may be guided accordingly.
5. Regardless of individual discipline that it might be imposed, should Smart & Howe be
subject to professional discipline as a law firm for the conduct of Able? NY is the first jurisdiction
to make a law firm itself subject to professional discipline.

B. The Rights of a Lawyer Who Refuses to Violate the Law


1. If a lawyer refuses a direction to do something illegal, does that lawyer have any
protection against being fired for his/her courage? This type of firing is a.k.a. retaliatory
discharge. Some jurisdictions have held that there is no cause of action, while others have held
that there is.

D. Doubts About Life in the Modern Large Law Firm


D. Problem 34
Leaving One Law Firm and Forming Another
(Page 545)

Facts
Eight women in big man firm feel not being treated fairly so want to leave and take with them the
estate part of the practice which they ran. The men were angry but instead of suit offered to give
them estate part for a price. Women also want to contact the union members of a union which
one of the women was a part of prior to the firm to offer estate planning services.

A. The Law and Ethics of Departing from a Law Firm


1. What duties do law firm partners owe to each other? Partners traditionally are said to
owe each other the fiduciary duties of loyalty and care.
2. To whom do a law firm's clients belong? What rights should departing partners and
associates have to try to cause clients to consult them at their new firm? Comment 1 to 1.17
(which governs the sale of law practice) states that the practice of law is a profession, not merely
a business. Clients are not commodities that can be purchased and sold at will.
3. Should there be any limits on the departing lawyers' contacts with firm clients to seek
to persuade them to follow the lawyers to the new firm? If the clients are the firm's clients, they
are not the attorneys, and for 7.3 purposes, they are prospective clients. Thus, the attorney may
have a 7.3(a) problem if he/she tries to call those clients (prohibited solicitation). However, if they
are the attorney's clients, then under 7.3 the attorney is permitted to write letters to those w/whom
they have or had a professional relationship. Of course, this letter is subject to 7.3(b)(2) and
cannot involve coercion, duress, or harassment. It is also subject to 7.1 and cannot be false or
misleading.
4. How should lawyers proceed when leaving a law firm?

B. Efforts by Law Firms to Inhibit Their Lawyers' Ability to Leave


1. Rather than relying on a suit for breach of fiduciary duty to inhibit departures, might a
law firm simply put a covenant not to compete in its partnership and employment agreement that
requires former partners and associates not to practice law in competition w/the firm for a period
of three years after leaving? No. Rule 5.6 states that a lawyer shall not participate in offering or
making: (a) a partnership, shareholders, operating, employment, or other similar type of
agreement that restrict the right of a lawyer to practice after termination of the relationship; or (b)
an agreement in which a restriction on the lawyer's right to practice is part of the settlement of a
client controversy. The reason for this under comment 1 is that an agreement restricting the right
of lawyers to practice after leaving a firm not only limits their professional autonomy but also limits
the freedom of clients to choose a lawyer.

C. Buying a Lawyer's or Law Firm's Practice


1. Would any ethical problem have been presented if the women associates had bought
their portion of the law practice from Bright & Learned? Rule 1.17 governs the sale of a law
practice, and states that a lawyer or a law firm may sell or purchase a law practice, or an area of
law practice, including good will, if four conditions are satisfied. One such condition is that the
seller ceases to engage in the private practice of law, or in the area of practice that has been
sold, in the geographic area or in the jurisdiction in which the practice has been conducted.
(Comment 2 states that the seller must make a good faith effort to cease practicing). Also, the
entire practice, or the entire area of practice, must be sold to one or more lawyers or law firms.
The seller must give written notice to each of the seller's clients.
What about confidential information? Comment 7 states that negotiations b/t seller and
prospective purchaser prior to Seller must get informed consent from client to disclose any
confidential information.
2. May a lawyer properly recommend that his/her clients retain a the lawyer who has
bought the practice? Rule 7.2(b)(3) seems to imply that such a recommendation is possible. It
states that a lawyer shall not give anything of value to a person recommending the lawyer's
services except that a lawyer may pay for a law practice in accordance with 1.17.
Rule 1.5(e) states that the forwarding lawyer has to remain liable for the new lawyer's
malpractice, unless they each want to split the work and be responsible for their own malpractice.

D. Establishing a Group Legal Services Plan as Part of a Law Firm


1. Rule 7.3(b)(2) states that a lawyer shall not give anything of value to a person
recommending the lawyer's services except that a lawyer may pay the usual charges of a legal
service plan or a non-profit or qualified lawyer referral service (i.e. a referral service approved by
the local bar association).
The lawyer must be aware of 7.3(d), which states that notwithstanding the prohibitions in
(a) (i.e. no live contact, etc.), a lawyer may participate w/a prepaid or group legal service plan
operated by an organization not owned or directed by the lawyer that uses in-person or telephone
contact to solicit memberships or subscription for the plan from persons who are not known to
need legal services in a particular matter covered by the plan.
Comment 6 states that the rule is usually addressed to an individual acting in a fiduciary
capacity seeking a supplier of legal services for others who may, if they choose, become
prospective clients of the lawyer. It is not intended to prohibit a lawyer from contacting
representatives of organizations or groups that may be interested in establishing a group or
prepaid legal plan for their members. Comment 8 states that the organization must not be owned
by or directed by any lawyer or law firm that participates in the plan.

Overview of RPC 5.4


• No fee sharing with non-lawyers except for...
o Payments to the estate or beneficiary of a deceased lawyer
pursuant to th terms of a firm;s operating or partnership agreement.
o Payments to the prepresentitive of a deveased, disabled or
disappeared lawyer following the purchase of that lawyers law
practice.
o Payments to non-lawyer staff to compensate them for their services
o Payments of court-awarded fees to pro-bono organizations
• No partnerships with non-lawyers when any of the activities consist of the
practice of law.
• No influence over professional judgment by third-party who pays lawyer
for representation of client client
• No non-lawyer owner, officer or director of professional corportation that is
authorized to practice law for profit.
Overview of RPC 5.6(a)
• What is the basic obligation imposed by rule 5.6(a)?
o Assume a law firm’s partnership agreement provides that a partner
who leaves the firm cannot provide legal services within the state
for a two-year period.
 Is this provision enforceable? No because it limits the
attorney ability to practice alw after disassociation (5.6(a)
o Assume a law firms partnership agreement provised that partners
who “retire” (defined as those who will discontinues practicing law
in the jurisdiction) will receive a retirement bonus pf 1% of the firms
net income for the preceding 24 months
 Is this provision enforceable? Yes because this deals with
benefits concerning retirement. The lawyer is retiring and
that decision already prohibits the public form hiring that
lawyer so the public is not disadvantaged.
o Does RPC 5.6(A) apply to restrictions on the right to practice
arising out of the sale of a law practice? Yes
 What does comment [3] say about this? That language does
not apply to the sale of a law practice and the purchaser can
seek to restrict that persons ability to do so.
• What is the basic obligation imposed by rule 5.6(b)?
o Assume Lawyer sues GM for injuries suffered by Client in auto
accident. Lawyer figures out that the car’s design is defective. GM
offers to pay $5 million to settle so long as lawyer agrees not to
take another case against GM?
 Can GM’s lawyer make the offer? No it would restrict the
lawyer from taking any other cases.
 What do you suppose is justification for this rule? Prevents
conflicts between the lawyer and the client
• What would happen at mediation if these sorts of
offers were permissible and Gm made one? Client
would want to settle, but the lawyer wants to be able
to represent other clients so the lawyer will not want
to take it so he can continue to represent other
clients.
• Which RPC would rear its ugly head? 1.7 (a)(2)
concurrent conflict problem between the lawyer who
wants to represent other clients and the client who
want the money.
 How can GM accomplish the same result of “blacking out”
the lawyer entirely properly- based upon what we have
learned this semester? GM could ask the lawyer to represent
them too, then the lawyer would no longer be able to take
any more cases against them.
• Overview of RPC 1.17(a) and (b)
o What does Rule 1.17 allow lawyers to do?
 Buy and sell law practices.
o How would you describe what the seller must stop doing for the
sale to satisfy the requirements of Rule 1.17(a)?
 Must stop private practice or in the area of practice the
lawyer has sold.
o How would you describe what the seller must do under 1.17(b)?
 The sale must cover the entire practice, or the entire area of
practice and can be sold to one or more lawyers of a firm.
o Assume selling lawyer has a personal injury practice.
 Can she sell all of the practice to another lawyer?
• Yes, she sells everything to another lawyer. (1.17(a)).
 Can she sell half to one and half to another?
• Yes, just as long as the entire practice or entire area
is sold.
o Assume the lawyer has a personal injury practice and an estate
planning practice.
 Can she sell the pi practice and continue to represent her
estate planning clients?
• Yes, she is just precluded from practicing PI.
 Can she put an advertisement in the newspaper for more PI
clients?
• No, she must discontinue practice of PI.
• Overview of RPC 1.17(c) and (d)
o How would you describe RPC 1.17(c)? The seller must give written
notice of the proposed sale, the client does not have to be part of
the transaction or can take their file, and their consent will be
presumed if they do not respond in 90 days.
 What happens if one of the seller’s client’s does not want to
be represented by the buyer?
• The transaction continues, but the seller cannot
transfer that client’s file.
 What happens if one of the seller’s clients creates a conflict
with one of the buyer’s existing or former clients?
• 1.7 or 1.9 is implicated. If consent could not be
obtained, the purchasing lawyer cannot represent that
client.
• It does not effect the sale of the practice. The client
will have to go find new representation.
o How would you describe the obligation under 1.17(d)?
 The fees should not be increased by reason of the sale.
 The purchaser is obligated to the agreement the selling
lawyer had with the client.
o How does the purchaser determine whether the seller’s practice is
worth buying giving the constraints imposed by 1.6(a)?
 The seller can disclose information to the purchaser to do a
conflicts check in order to see if this is a viable purchase.
 But the seller cannot give any information precluded by
1.6(a).
 A purchaser will want this information to see how well they
stand in current cases. But the selling lawyer cannot provide
it to them.
o Does comment [7] give an explanation?
 The purchaser is deemed to have access to the client’s
information after 90 days and can see if they want to go
forward with the purchase.
o Can you take the good clients and leave behind the bad ones?
 No have to take all or nothing.
• Variation One
• Lawyers in a trust and estate group decide to leave and they
would like to take as many clients with them as possible. They
ask you what they can permissibly do to set up their own
practice prior to leaving their current firm.
o What if anything, can they do prior to departure with
respect to things like locating office space, obtaining a line
of credit, determining a firm name and the like?
 Can do anything that would allow them to leave the
firm on Friday at 5pm, and show up at a fully
functional office on Monday at 9am.
o What if anything, can they do prior to departure with
respect to things in contacting the clients with whom they
are working for purposes of soliciting their business going
forward?
 They cannot bad mouth their current firm.
 Tell the other lawyers of the firm that they are
leaving with timely notice. So that the other lawyers
can solicit the clients as well.
o What if anything, can they do prior to departure with
respect to things in contacting the clients with whom they
have not worked with for purposes of soliciting their
business going forward?
 Cannot contact them if you have not done work with
them in the past.
 You have no right to solicit them before you leave
the firm.
• What can you do after you leave the firm?
 Can solicit and advertise with respect to 7.3 and 7.2.
• Variation Two
• The Trusts and estates department leaves the firm and they
convince several of their clients to go with them. The remaining
lawyers ask you whether they can sue the departed lawyers for
tortuous interference with the firm’s relationship with these
clients.
o What do you tell them?
 Clients are not constant, and the client ultimately has
the power to terminate the relationship at any time
for a good reason, bad reason or no reason at all.
• Variation Three
• The remaining lawyers don’t want a public fight. As such, they
consider offering to sell the estate planning practice to the
departed lawyers.
o Would you advise the remaining lawyers to make such an
offer?
• If the surviving lawyers decided to sell, would
only make sense if they would be willing to
cease practicing law in this area anymore
because, Doing so would preclude them from
practicing in this area under RPC 1.17
 Hint – what is the downside?
• They could not practice in this area anymore
and would lose the income from such area.
o Would you advise the departing lawyers to accept such an
offer?
 They would be paying for the right to solicit clients
that they already have the right to solicit already.
They do not need to purchase this right.
• Unauthorized Practice of law
• What is the practice of law?
o Giving advice or counsel to persons as to their legal rights
and responsibilities or to others
o Selecting, drafting or completing legal documents or
agreements that affect the legal rights of a person
o Representing a person before an adjudicative body,
including but not limited to, preparing or filing documents
or conducting discovery
o Negotiating legal rights or responsibilities on behalf of a
person.
o The practice of law at its foundation is IRAC.
• What is the authorized practice of law?
o Practicing in a jurisdiction in which you are not authorized
to. (not formally admitted to practice law)
o A non lawyer can engage in the unauthorized practice of
law
o 5.5  cannot engage in the unauthorized area of law.
• My mother advises me that I cannot lawfully advertise my
services as a lawyer to consumers in Florida
o Has my mother engaged in the unauthorized practice of
law?
 Yes, she cannot tell you what your rights are.
o Does it matter if her advice is correct?
 No it is still a violation (maybe criminal)
o Has she violated the RPCs?
 No because she is a nonlawyer
o My mother is stopped for speeding. She subpoenas the
officer and represents herself in traffic court.
o Has she engaged in the unauthorized practice of law?
 No she has the right to represent herself on a pro se
basis.
• Overview of RPC 5.5(a)
• How would you describe the prohibition imposed on lawyers by
RPC 5.5(a)? you lawyer engaged in the unauthorized practice of
law when you purport to provide legal services in a place where
you are not licensed or authorized to do so.
o Assume that a lawyer is admitted to practice in Fl only
 Can he practice in the other 49 states?
• The general rule is that the lawyer can only
practice in Florida. (unless another rule allows)
 Assume that the lawyer is admitted in Florida only.
She receives a call call from a friend in GA who is not
a lawyer but who has a friend in Georgia who has a
legal problem.
 Can the lawyer give the friend advice?
• Last clause of paragraph (a)  cannot help
others in the unauthorized practice of law.
o Assume my mother asks for my advice about what to do
when appearing in court to contest her traffic ticket. She
lives in Washington and I am still admitted to practice
there.
 Can provide this information because she is
representing herself. She is participating in an
authorized practice of law. And therefore the lawyer
is not assisting in the unauthorized practice of law.
(makes all the difference in a 5.5(a) analysis)
• Overview of RPC 5.5(b)
• How would you describe the basic prohibition imposed by rule
5.5(b)(1)? Cannot establish an office in a jurisdiction in which the
lawyer is not authorized to practice, and you cannot have a
systematic and continuous presence (anything more than
temporary) for the practice of law in that jurisdiction.
• How would you describe the basic prohibition imposed by rule
5.5(b)(2)? You cannot convey the impression in that other
jurisdiction that you are authorized to practice in that jurisdiction
if the lawyer is not authorized to practice there.
o Assume a lawyer is admitted to practice in Fl only
 Can the lawyer set up an office in Savannah, Georgia
for the practice of law?
• No.
 Can the lawyer advertise in savannah for purposes of
providing services in the area without specifying
where she is admitted to practice?
• No. It would be misleading. Most people would
assume and conclude that the person running
the ad is licensed to practice in Georgia or else
he wouldn’t be running the advertisement.
 Which rule is implicated in this circumstance?
• Yes RPC 7.2. and also 7.1. and 5.5(b)(2)
• Overview of RPC 5.5(c)
• How would you describe the basic opportunity provided by Rule
5.5(c)(1)? As long as the lawyer is admitted to practice in the US
and is not prohibited in any jurisdiction from the practice of law,
they can associate themselves with another lawyer from another
state where they are not formally admitted who is formally
admitted in that state so long as that lawyer is associated with
the case.
• How would you describe the opportunity provided by rule 5.5(c)
(2)? Applies only to litigators and tells litigators that they are
allowed to litigate away from home, notwithstanding the fact
that the lawyer is not permitted to practice in that jurisdiction as
long as they comply with local rules in that jurisdiction applying
to lawyers not admitted to practice in that state on a temporary
basis. Provides litigators with two key advantages (1) lawyers in
one state can handle matters in another state as long as they
comply with out of state rules(2) when handeling cases at home
they can croos state lines for the purpose of litigation things.
o What does it mean for a lawyer to be admitted in a
jurisdiction on a pro hac vice basis? Universal way in both
federal and state courts, a lawyer from one state is given
permission in another state to practice (including trial) in
another state on a temporary basis. (can handle one
specific case but not others)
o Assume a lawyer is admitted to practice in Florida but has
client who must bring suit in Georgia because the Florida
courts cannot exercise in personam jurisdiction over the
defendant.
 Will the Florida lawyer eventually have to be
admitted in Georgia on a pro hac hice basis if she
wishes to handle the case? Yes
 Can the Florida lawyer perform pre-litigation
activities in Georgia before she is admitted pro hac
vice? Yes (last clause of 5.5(c)(2)) you don’t have to
wait for your formal admission as long as you have a
reasonable belief you will eventually be admitted on
a pro hac vice basis.
 To whom does the clause “or a person the lawyer is
assisting” refer to? Other lawyers who are assisting
the lawyer who herself has been admitted on a pro
hac vice basis or reasonably believes they will be
admitted (junior associates who will provide
assistance to those senior attorneys) (subordinate
lawyer who is assisting a lawyer who has been or
reasonably will be admitted on a pro hac vice basis)
o Assume a lawyer is admitted to practice in Florida and has
a case that is pending in the Florida courts. The lawyer
would like to depose a witness in Georgia.
 Can the Florida lawyer take the deposition in
Georgia? The lawyer can take the deposition because
this is envisioned by 5.5(c)(2) (lawyer needs to cross
a state line regarding an at home litigation matter)
• How would you describe the opportunity provided by rule 5.5(c)
(3)? A litigation oriented rule and says they can go to other
jurisdictions and participate in alternative dispute resolutions,
without worry as long as ADR proceeding is in regards to a
matter in the lawyers home state.
o Assume Tony Piazza is the nation’s best mediator. He
travels from his home in Honolulu for one week each
month to conduct mediations from an office in San
Francisco.
 Can lawyers admitted in Florida, and who are
handling a case in Florida, travel to Tony’s office in
San Francisco to mediate their dispute? Yes because
the mediation in California is related to the issue in
Florida.
 What is the last clause, beginning with “and are not
services,” getting at? Drafters way of telling lawyers
before travelling to another state check the rules of
that jurisdiction to be sure that jurisdiction does not
require admission pro hac vice to do the ADR (no
state has this rule yet, but at some point they might)
so it is advised to be sure to be safe and avoid any
quirky rules.
• Rule 5.5(c)(4) is the transactional lawyer’s best friend because it
effectively allows the lawyer to provide services everywhere so
long as she does not violate rule 5.5(b).
o Clients legal problems are governed by federal law and
client wants a lawyer uniquely qualified and experienced in
federal law.
 Lawyer in state X advised corporation in state Y
about federal securities laws
o Client’s business activities are connected to lawyer’s home
state, at least in part
 Lawyer in state X advises corporation doing business
in states X and state Y about problem in state Y.
o Client contacted lawyer in lawyer’s home state
 Lawyer in state X receives inquiry from corporation in
state Y about regulatory obligations in state Y.
• Overview of RPC 5.5(d)
• How would you describe the opportunity provided by rule 5.5(d)
(1)? Normally talking about in house lawyers whose only legal
services are provided to the entity which hires the individual.
o Assume a lawyer is admitted to practice in New York and is
employed by IBM at its corporate headquarters in Armonk,
New York.
 Would the lawyer be required to gain admission to
practice in California if IBM sent the lawyer to work at
one of its facilities in san jose? No corporations can
send those lawyers wherever they are needed
without having to gain admission to the California bar
as a condition on assisting the corporation in this
other jurisdiction.
 Would your answer be any different if the IBM lawyer
opened an office where she does her work continued
to work there for twenty years while emploed by
IBM? No as long as she is still working for IBM.
• Is this consistent with rule 5.5(b)(1) no, but this
says unless otherwise authorized and 5.5(d)(1)
authorizes this. And they can stay in this
jurisdiction perpetually as long as they are
providing services to the corporation which
employs them.
o Assume the IBM lawyer is admitted in New York but is
working for the company in San Jose, California.
 Can the lawyer draft a contract for a transaction
between IBM and a California corporation? Yes,
because this is the situation 5.5(d)(1) is looking at,
he is doing work in that jurisdiction for that corporate
employer. (lawyer cannot go to court) (he needs to
be admitted to that state bar or be admitted pro hac
vice, he is only allowed to work there as a
transactional lawyer not a litigator)
 Can the lawyer draft a contract for a transaction
between an IBM employee in her individual capacity
and a California corporation. No, because the special
consideration is given to the corporation and not the
individual employee. The client is the corporation
and the employee is not the client.
• Overview of RPC 5.5(d)(2)
o Rule 5.5(d)(2) allows to practice in jurisdictions in which
hthey are not formally admitted so long as the practice is
limited to…
 Bankruptcy
 Immigration
 Patent
 Federal income taxation
o Rule 5.5(d)(2) allows lawyers to handle cases in a U.S.
District Court so long as the admission requirements of the
district court do not require prior admission in the forum
state.
• Overview of rPC 5.7
• How would you summarize the definition of law-related services
in rule 5.7(b)? like legalservices but are things a non lawyer
could do without violating RPC.
o What are examples that “might reasonably be performed
along with legal services” but can “properly be performed
by non-lawyers”?
 Tax advice(L) + tax return preparation (LRS) or
accounting services (LRS) (see comment 9)
• How would you describe the obligation imposed by RULE 5.7(a)
(1) ad 5.7 (a)(2)? A lawyer would want to get away from the
RPC’s let your client know that it is not covered under the
client/lawyer privilege. You have to specifically tell the client that
is you were to make the clients tax returns you have to say you
are doing this outside the attorney/client privilege and that you
are not acting as a lawyer etc.
o Assume a tax lawyer wishes to make some extra money
preparing tax returns.
 Which portion of rule 5.7(a) will likely make the RPC’s
applicable to the lawyer is she prepares a tax return
for a client
 Which portion of rule 5.7(a) will likely make the RPC’s
applicable to the lawyer if she prepares a tax return
for a non-client?
• What must the lawyer do to avoid application
of the RPC’s to the provision of legal services?
• Variation one
• Victor Dowd intends to create do-it-yourself estate planning and
administration kits that he intends to sell in states in which the
contents of the kits are fully consistent with state law.
o How would you describe the current state of the law
regarding the extent to which the do it yourself kits do or
do not constitute the unauthorized practice of law?
Appears that a lawyer can do that without problem but in
general there is nothing wrong with preparing these kits.
o What additional activity would assuredly cause Dowd to
engage in the unauthorized practice of law even if the sale
of the kits does not? If he is advertising there he may be
liable under RPR 7.1 and 7.2 if the lawyer purports to
answer specific questions it is impermissible
• Variation two
• Dowd wishes to set up a subsidiary, in which he will have a
controlling interest, that will work out of his office and provide
legal services as well as financial planning and insurance
services.
o Will dowd be required to comply with the RPC’s with
respect to any legal services he provides? Yes
o Will dowd be required to comply with the RPC’s with
respect to the financial planning and insurance services he
provides? Depends entirel on the extent he can forcefit his
situation into (a)(1) and (a)(2)
 Why is it important that the facts indicate that Dowd
will have a “controlling interest” in the subsidiary?
The lawyer must at some level control the entity
before you worry abot (a)(1) or (a)(2)
 What might Dowd do if he wishes to avoid the
application for the RPC’s to the provision of financial
or insurance services to individuals who want those
services only.
 What might Dowd try to do if he wishes to avid the
application of the RPC’s to the provision of financial
or insurance services to individuals who want those
services along with legal services?
• *** Final exam 1 essay (60-70% of exam score) and 8-15
multiple choice***
E. Problem 35
The Duty to Work for No Compensation
Bro Bono (page 562)

Rule 6.1 Voluntary Pro Bono Publico Service

Lawyer’s expectation to Earn a Fee


• Novak represents a client on an hourly rate basis. The client does not pay Novak’s bill, yet
according to Comment 4 of rule 6.1 he cannot consider this pro bono.
Lawyer’s Receipt of fee award
• Novak takes a civil rights case on a pro bono basis and is successful. The court awards 50,000
to Novak for his time on the case pursuant to a statute authorizing fee shifting. Novak can still
consider this as pro bono, and according to comment 4 tells lawyers there is nothing wrong with
claiming pro bono in certain contexts even if you get paid, but you should consider but are not
required to donate the fee reward to a charity or donate it to a cause for people with little or lesser
means.
Lawyer’s financial Contribution
• Novak can fulfill under 6.1(a) (1)(2) under comment 9 provide pro bono service through
financial services to the exclusion of actual legal services. Allows a lawyer who is to busy to do
actual pro bono work to make a contribution to a legal service in exchange.
Lawyer’s failure to perform pro bono
• under comment 12 even when a lawyer never does pro bono, failure to perform pro bono is not
an offense that is punishable.

Facts
State bar of GA give min hours 100 hrs per year in uncompensated legal services. Wright
appointed by state for D in murder case when D refused public defendant. Statutory
compensation is max of 750.00. He knows he is guilty but D said innocent and required 300
hours his time so he sued the state for 30K sine his fee is normally $100.00/hr.

A. The Moral Obligation to Provide Pro Bono Legal Services


1. Do lawyers have any moral obligation to provide free or reduced price legal services?
Maybe. The Model Rules seem to require it under rule 6.1. The rule states every lawyer has a
professional responsibility to provide legal services to those unable to pay. The rule then states
that a lawyer should (as opposed to "shall") aspire to render at least 50 hours of pro bono work
per year. Comment 12 states, however, that the responsibility set forth in this rule is not intended
to be enforced through disciplinary process. Thus, this is an aspirational rule.
Plus, not all lawyers are on the same playing field when it comes to pro bono work, b/c
lawyers do not all make the same amount of money the way doctors tend to. Not every lawyer is
qualified to do the work a pro bono case requires (e.g. a patent attorney might know nothing of
criminal trial work); however, under 1.1, such a lawyer would be required to bring him/herself up
to speed. This can be burdensome, which is probably why many lawyers might not do pro bono
work.
2. Is there a shortage of available legal help such that part of it must be allocated to poor
people as one might allocate food after a severe hurricane? Is there a market failure in the
provision of legal services? [Why don't we have a legal system akin to the Medicaid system in
the medical field?] Law is much different from medicine. Everyone will need a doctor in their
lives, while some people may go their entire lives w/o ever needing a lawyer.

B. Efforts to Translate a Moral Obligation into a Legal Requirement


2. In Florida, all attorneys are "encouraged" to complete either 20 hours of pro bono
work, or to pay $350 to a legal aid organization. All attorneys are also required to report whether
they did the work, paid the money, or did nothing. There is no disciplinary consequence for
reporting that the lawyer did nothing; however, it is a violation not to report. This requirement has
been upheld by the 11th Circuit.
Remember, pro bono makes lawyers "look" good to society, which is one of the
reasons it is so strongly encouraged.

C. The Tradition of Accepting Court Appointment in a Criminal Case


1. What is the basis, if any, of the distinction b/t the "should do pro bono service" in 6.1
and the "shall accept court appointments" in rule 6.2?
If the lawyer is not getting paid enough, he can petition the state for more money, but he
cannot just stop representing the client in the matter.

D. Sources of Possible Other Funding for Legal Services to the Poor


3. IOLTA (Interest on Lawyer Trust Accounts) programs are in existence in most
jurisdiction today, and even though client consent is rarely required or sought, the ABA has said
that lawyers may properly participate in such programs. Nothing prohibits a lawyer from placing
clients' funds in an interest-bearing account. It would seem that the client should be entitled to
the interest; however, the ABA has upheld allowing lawyers to take the interest b/c the expenses
incurred in creating notification, record-keeping, and accounting, lawyers may incur other costs in
attempting to place clients' funds at interest.

F. Problem 36
Problems in Class Action Representation
(Page 578)

Class actions generally must meet requirements of FRCP 23. See also 7.2 and 7.4 with regard to
advertising (and remember that target advertising is ok!).
Rule 1.8(g) ensures that settlements are for the entire class, and states that a lawyer who
represents two or more clients hall not participate in making an aggregate settlement of the
claims of or against the clients, or in a criminal case an aggregated agreement as to guilty or nolo
contendere pleas, unless each client gives informed consent, in writing signed by the client.
Rule 1.8(e) states that a lawyer shall not provide financial assistance to a client in
connection w/pending or contemplated litigation, except that: (1) a lawyer may advance court
costs/expenses of litigation, the prepayment of which may be contingent on the outcome of the
matter; and (2) a lawyer representing an indigent client may pay court costs and expenses of
litigation on behalf of the client. A lawyer should not, however, ever offer to pay such expenses
such as living expenses, b/c this may encourage out of town prospective plaintiffs to file the
lawsuit (class action) just to get a free stay.

G. Problem 37
The Future of the Practice of Law
(Page 595)

Facts
Dowd wants to advice clients about more effective estate planning by publishing do-it yourself kits
which he plans to sell in states which have been verified to legal accuracy. Nationwide insurance
company likes this and has offered to refer its customers to him. Clients could come from various
states where Dowd will draft documents for them. Fee will be paid by the insurance company.
Now Dowd wants to create a planning subsidiary that will offer service to 10 states and he is only
licensed to practice in 1 of them. They will all be billed a single hourly rate, no matter what type
of specialty is involved, and Nationwide proposes to advertise the service by putting ads on TV
called, "We've all go to go sometime. Might as well do it right."

Note: This is about practicing in one jurisdiction where you are not licensed whether you are
litigating in this jurisdiction or contacting clients in these jurisdictions. It is a first degree
misdemeanor to practice law without a license.

A. The Unauthorized Practice of Law


1. Rule 5.5 comment 2 states that the definition of the practice of law is established
by law and varies from one jurisdiction to another. Florida says the definition can vary. The ABA
task force tried to refine the definition by suggesting that a person is presumed to be practicing
law when engaging in any of the following conduct on behalf of another: (1) giving advice or
counsel to persons as to their legal rights or responsibilities or to those of others; (2) selecting,
drafting, or completing legal documents or agreements that affect the legal rights of a person; (3)
representing a person before an adjudicative body, including but not limited to, preparing or filing
documents or conducting discovery; or (4) negotiating legal rights or responsibilities on behalf of
a person. Comment 2 states that whatever the definition, limiting the practice of law to members
of the bar protects the public against rendition of legal services by unqualified persons.
5. Should Dowd's proposed sale of do-it-yourself kits constitute the unauthorized
practice of law. Disclaimers all over the product certainly help (i.e. "For your particular problem,
see an attorney"). NY County Lawyers' Assoc. v. Dacey: book advised people on how to avoid
probate by advising people to put all of their property in trusts. Court said it cannot be claimed
that the publication of a legal text which purports to say what the law is amounts to legal practice.
The mere fact that the principles or rules stated in the text may be accepted by a particular reader
does not effect his. This is the essential of a legal practice--the representation and the advising
of a particular person in a particular situation.
Quicken: computer program containing over 100 legal forms that were
represented to be valid in 49 states. Contained disclaimer stating that ti did not provide specific
information for the consumer's exact situation. Court held that both the preparation of legal forms
and the selection of which form to use constitutes the practice of law in TX. However, court did
say that the prohibition in that case did not burden commercial speech more than was necessary
(addressing the First Amendment issue), and it enjoined the sale of the software.
The disclaimer on the product is not a substitute for the advice of an attorney.
3. Does the insurance company w/which Dowd works involve him in assisting the
unauthorized practice of law? To the consumer, the product is a "package" deal by which the
consumer may draft his/her own will, even though Dowd is the only person actually doing the
legal work that goes into the product. 5.4(a) states that a lawyer or law firm shall not share legal
fees w/a non-lawyer. This takes away
Florida Bar v. Beach: Beach had contract with paralegal company who prepared
and sold legal documents for consumers, and who met w/clients but told clients there was no
attorney/client relationship, but attorney met with paralegal to discuss clients. The court
suspended Beach for 90 days for assisting company to do legal work while unlicensed.
4. Should filling in the blanks on prepared forms constitute the practice of law? In
many states, it does not. Think of real estate closings.
6. May a person represent herself in court? Of course. May she act as her own
lawyer in negotiating a contract? Yes. Why do we let people act pro se if unauthorized practice
rules are so important? B/c unauthorized practice, in short, involves assisting another w/a legal
matter, not handling a matter for one's self. [In criminal law, a defendant has a constitutional right
to represent him or herself]. An officer of a corporation cannot represent the corporation,
however, b/c it would be a conflict of interest.
Comment 3 to 5.5 states that a lawyer may provide professional advice and
instruction to non-lawyers whose employment requires knowledge of the law; for example, claims
adjusters, employees of financial or commercial institutions, social workers, accountants, and
persons employed in government agencies. Lawyers also may assist independent non-lawyers,
such as paraprofessionals, who are authorized by the law of a jurisdiction to provide particular
law-related services. In addition, a lawyer may counsel non-lawyers who wish to proceed pro se.

B. The Multi-Jurisdictional Practice of Law


1. In what jurisdictions may a licensed lawyer regularly practice? Generally, only in
the state in which that person took and passed the bar. That is what 5.5 is meant to enforce.
Some states have reciprocity agreements w/other states, but Florida does not. If an attorney has
a continued presence in a state in which he/she is not licensed, and holds him or herself out as
an attorney licensed to practice in that state, he/she is committing the unlicensed practice of law.
However, it is permissible for an attorney to associate w/attorneys from other
states in which he/she is not licensed. Comment 9 to 5.5 states that lawyers not admitted to
practice generally in a jurisdiction may be authorized by law or order of a tribunal or an
administrative agency to appear before the tribunal or agency. This authority may be granted
pursuant to formal rules governing admission pro hac vice or pursuant to informal practice of the
tribunal or agency.
According to comment 10, under 5.5(c)(2), a lawyer rendering services in this
jurisdiction on a temporary basis does not violate 5.5 when the lawyer engages in conduct in
anticipation of a proceeding or hearing in a jurisdiction in which the lawyer is authorized to
practice law or in which the lawyer reasonably expects to be admitted pro hac vice. However,
this reasonable expectation usually must be based on work the lawyer is conducting in a
jurisdiction in which he is licensed (e.g. pending litigation in his state that is connected to
legislation in another state in which the lawyer is not licensed).
Comment 13 states that 5.5(c)(4) permits a lawyer admitted in another
jurisdiction to provide certain legal services on a temporary basis in this jurisdiction that arise out
of or are reasonably related tot he lawyer's practice in a jurisdiction in which the lawyer is
admitted but are not w/in paragraphs (c)(2) or (c)(3). These services include both legal services
and services that non-lawyers may perform but that are considered the practice of law when
performed by lawyers.
3. Should transactional lawyers have a similar way to follow their clients to other
jurisdictions? Birbrower v. Superior Court:
Comment 16 to 5.5 states that 5.5(d)(1) applies to a lawyer who is employed by a
client to provide legal services to the client or its organizational affiliates. This paragraph doe
snot authorize the provision of personal legal services to the employer's officers or employees.
This paragraph applies to in-house corporate lawyers, government lawyers, and others who are
employed to render legal services to the employer.
VIII. Ethical Conduct of Judges

A. Problem 38
Judges' Disqualifying Conflicts of Interest
(Page 618)

Facts
Judge Baxter got personal loan for down payment of house from lawyer who he knows that
represents the bank that gave the mortgage. She is representing the bank over lien priorities.
Judge told both sides of his potential conflict. Also, has niece who invests in company that D is in
for the lien case. Also, he was on committee for assessment policy where he is overseeing a
case on taxpayer who is challenging his assessments.

A. Financial Interests That May Create Disqualifying Bias


1. Was it improper of Anderson to lend money to Judge Baxter to help him buy a
house, or for Judge Baxter to accept the loan? Yes, it was improper. Canon 4(D)(1)(b) states
that a judge shall not engage in financial and business dealings that involve the judge in frequent
transactions or continuing business relationships w/those lawyers or other persons likely to come
before the court on which the judge serves.
Canon 4(D)(5)(f) states that a judge shall not accept, and shall urge members of
the judge's household not to accept, a gift, bequest, favor, or loan from anyone, except for a loan
from a lending institution in its regular course of business on the same terms generally available
to persons who are not judges. Comment 1 on page 606 states that 4D(5)(h) prohibits judges
from accepting gifts, favors, bequests, or loans from lawyers or their firms if they have come or
are likely to come before the judge; it also prohibits gifts, favors, bequests or loans from clients of
lawyers or their firms when the clients' interests have come or are likely to come before the judge.
As for the attorney, she will be subject to discipline under 3.5(a), which states
that a lawyer shall not seek to influence a judge by means prohibited by law (i.e. bribery), and
8.4(f), which states that it is professional misconduct for a lawyer to knowingly assist a judge or
judicial officer in conduct that is a violation of applicable rules of judicial conduct or other law.

3. Under canon 3E(1) a judge shall disqualify himself or herself in a proceeding in


which the judge's impartiality (denotes absence of bias or prejudice in favor of or against a
particular party or class of parties, as well as maintaining an open mind in considering issues that
may come before the judge) might reasonably be questioned [and gives specific instances in
which a judge should disqualify him or herself]. Comment 1 states that a judge is disqualified
whenever the judge's impartiality might reasonably be questioned, regardless of whether any of
the specific rules in 3E(1) apply. Comment 2 states that a judge should disclose on the record
information that the judge believes the parties or their lawyers might consider relevant to the
question of disqualification, even if the judge believes there is no real basis for disqualification.
Canon 3F allows a party and his/her attorney to remit (i.e. waive) disqualification.
Under 3F a judge disqualified under 3E may disclose on the record the basis of the judge's
disqualification and may ask the parties and their lawyers to consider, out oft he presence of the
judge, whether to waive the disqualification. If following disclosure of any basis for
disqualification other than personal bias or prejudice concerning a party, the parties and lawyer,
w/o participation by the judge, all agree that the judge should not be disqualified and the judge is
then willing to participate, the judge may participate in the proceeding.
B. Financial Interests of the Judge's Family
1. What obligation does Judge Baxter have to know his niece's financial holdings?
Canon 3E(1)(3) states that a judge shall disqualify himself or herself in a proceeding in which the
judge's impartiality might reasonably be questioned, such as when the judge knows (denotes
actual knowledge of the fact in question; knowledge can be inferred from circumstances) that
he/she, individually or as a fiduciary, or the judge's spouse, parent, or child wherever residing, or
any other member of the judge's family residing in the judge's household (denotes any relative of
a judge by blood or marriage, or a person treated by a judge as a member of the judge's family,
who resides in the judge's household), or has an economic interest (see page 589) in the subject
matter in controversy or in a party to the proceeding or has any other more than de minimis
interest (denotes an insignificant interest that could not raise reasonable question as to a judge's
impartiality) that could be substantially affected by the proceeding.
Here, Judge Baxter does not know of his niece's finances. However, he does not
have a duty to know under the Code. He would only have such a duty of knowledge w/regard to
his spouse and minor children. See 3E(2), which states that a judge shall keep informed about
the judge's personal and fiduciary economic interests, and make a reasonable effort to keep
informed about the economic interests of the judge's spouse and minor children residing in the
judge's household. Judge Baxter's niece, while a member of his household, is not a minor child--
she's 19.
Thus, the next inquiry is whether Judge Baxter's niece has a de minimis interest
in the insurance company that is the defendant in the lien priorities case before him. She owns
10 shares, a 1/100,00 interest. Whether this is de minimis depends upon what kind of impact his
ruling against the insurance company (in which she owns 10 shares) would have on his niece's
finances.
3. If Judge Baxter had a daughter who practices federal tax law in a local firm, must
he recuse himself whenever his daughter's firm enters an appearance in a case? If the judge's
daughter appears before the judge, he must recuse himself. Canon 3E(1)(d) states that a judge
shall disqualify himself or herself in a proceeding in which the judge's impartiality might
reasonably be questioned, such as when the judge or the judge's spouse, or a person w/in the
third degree of relationship (great-grandparent, grandparent, parent, uncle, aunt, brother, sister,
child, grandchild, great-grandchild, nephew, or niece) to either of them, or the spouse of such a
person is . . . acting as a lawyer in the proceeding.
However, he will not be obligated to recuse himself if the law firm entered an
appearance. The commentary on page 600 states that the fact that a lawyer in a proceeding is
affiliated w/a law firm w/which a relative of the judge is affiliated does not of itself disqualify the
judge.
4. Should a judge be disqualified when someone from the judge's former firm (or
the judge's former law clerk) appears before the judge? Yes. Canon 3E(1)(b) states that a judge
shall disqualify himself or herself in a proceeding in which the judge's impartiality might
reasonably be questioned, such as when the judge served as a lawyer in the matter in
controversy, or a lawyer with whom the judge previously practiced law served during such
association as a lawyer concerning the matter.
Plus, 1.12(a) states that a lawyer shall not represent anyone in connection w/a
matter in which the lawyer participated personally and substantially as a judge or other
adjudicative officer or law clerk to such a person. This rule paralles rule 1.11.

C. Bias Arising From Personal Views Rather than Financial Interest


1. Does Judge Baxter's prior involvement in the Committee for Responsible
Assessment Policy require his recusal from the case challenging the plaintiff's assessment? Not
necessarily. Canon 2B states that a judge shall not allow family, social, political or other
relationships to influence the judge's judicial conduct or judgement. Canon 3E would also govern
in this situation with regard to whether a judge should disqualify him or herself. [Note: Canon
4C(3) states that a judge may serve as an officer, director, trustee, or non-legal advisor of an
organization or governmental agency devoted to the improvement of the law, the legal system or
the administration of justice or of an educational, religious, charitable, fraternal, or civic
organization not conducted for profit].
Two things the judge must ask himself: 1) Should the judge disqualify himself,
and 2) must he disclose that he was involved in the committee that was involved in assessment
policy? (Remember, he is no longer a member). Even though the judge is no longer a member
of the committee, comment 2 states that a judge should disclose on the record information that
the judge believes the parties or their lawyers might consider relevant to the question of
disqualification, even if the judge believes there is no real basis for disqualification.

B. Problem 39
The Judge as a Political Candidate and Public Figure
(page 635)

Facts

A. The Content of Judicial Campaign Speech and Activities


1. What does the Model Code of Judicial Conduct say about Kraft's advertisement
in his campaign to become a state judge? Should it matter that Kraft was not yet a judge at the
time the advertisement was run? 8.2(b) states that a lawyer who is a candidate for judicial office
shall comply w/the applicable provisions of the Code of Judicial Conduct. Canon 5A(3)(a) states
that a candidate for judicial office shall maintain the dignity appropriate to judicial office and act in
a manner consistent w/the impartiality, integrity, and independence of the judiciary, and shall
encourage members of the candidate's family to adhere to the same standards of political
conduct in support of the candidate as apply to the other candidate. Canon 5A(3)(d) states that a
candidate for judicial office shall not: (i) with respect to cases, controversies, or issues that are
likely to come before the court, make pledges, promises, or commitments that are inconsistent
w/impartial performance of the adjudicative duties of the office; or (ii) knowingly misrepresent the
identity, qualifications, present position, or other fact concerning the candidate or an opponent.
Remember that the judge cannot mislead or be untruthful in his ad campaigns,
even though they are not ads to solicit business. These requirements are all encompassed by
Canon 5.
4. What other limits are there on other political activities in which a judge may
engage? Canon 5A(1)(b) states that except as authorized in 5B(2), 5C(1) and (3), a judge or
candidate for election or appointment to judicial office shall not publicly endorse or publicly
oppose another candidate for public office.
Exceptions: 5C--judge or candidate subject to public election () is allowed to buy
tickets for and attend political gatherings, contribute to a political organization, distribute
pamphlets supporting his/her candidacy.

B. Judges Soliciting and Receiving Campaign Contributions


2. J.R. Kraft accepted campaign contributions pressed into his hand? Was it proper
for him to do so? No. Canon 5C(2) states that a candidate shall not personally solicit or accept
campaign contributions or personally solicit publicly stated support. However, a candidate may
establish committees of responsible persons to conduct campaigns for the candidate by means
not prohibited by law. Such committees may solicit and accept reasonable campaign
contributions. Such committees are allowed to solicit and accept reasonable campaign
contributions and public support from lawyers. A candidate's committees may solicit contributions
and public support for the candidate's campaign no earlier than one year before an election and
no later than 90 days after the last election in which the candidate participates during the election
year. [Professor states this is b/c the candidate will likely be in debt from the campaign, and
allowing them to accept contributions after the election will help them to alleviate some of this
debt].
4. Would a merit system of judicial selection (appointing judges) remove all ethical
issues from the selection process? Commentary 1 on page 613 states that there is legitimate
concern about a judge's impartiality when parties whose interests may come before a judge, or
the lawyer who represents such parties, are known to have made contributions to the election
campaigns of judicial candidates. This is among the reasons that merit selection of judges is a
preferable manner in which to select the judiciary.

C. Lawyer Criticism of Judges


1. Should the state attorney be subject to discipline for making critical remarks
about Judge Kraft at her press conference? Rule 8.2(a) states that a lawyer shall not make a
statement that the lawyer knows to be false or w/reckless disregard as to its truth or falsity
concerning the qualifications or integrity of a judge, adjudicatory officer or public legal officer, or of
a candidate for election or appointment to a judicial or legal office.
Sandlin case: refused to read an "actual malice" standard into the "reckless
disregard as to its truth or falsity" language of rule 8.2(a). The test should be what the
reasonable attorney, considered in light of all his professional functions, would do in the same or
similar circumstances.
We cannot shield judges from all criticism; we can only sanction for reckless or
false statements.
2. If another judge believed that the state attorney's charges against Judge Kraft
were true, would that judge have an obligation to report Judge Kraft? Rule 8.2(b) states that a
lawyer who knows that a judge has committed a violation of applicable rules of judicial conduct
that raises a substantial question as to the judge's fitness for office shall inform the appropriate
authority. Canon 3D(1) states that a judge who receives information indicating a substantial
likelihood that another judge has committed a violation of this Code should take appropriate
action. A judge having knowledge that another judge has committed a violation of the code that
raises a substantial question as to the other judge's fitness for office shall inform the appropriate
authority.
Note that judges are much less likely to report lawyer misconduct than lawyers
are to report judicial misconduct.
4. Would the state attorney be subject to any different form of discipline if she made
her remarks about Judge Kraft to his face or in a letter to him? The judge has his own remedies
for misconduct in his courtroom. See Williams v. Williams: lawyer told on other lawyer for saying
bad words about judge then judge held him in contempt but the appellate court said it wasn't a
significant disruption.

D. The Judge Rewarding His Friends


1. May Judge Kraft testify as a character witness on behalf of his friend? Canon
2B, which states that a judge shall avoid impropriety and the appearance of impropriety in all of
the judge's activities, precludes the judge from testifying voluntarily as a character witness. A
judge should discourage the party from calling the judge as a character witness under
commentary 5. Here, Judge Kraft told his friend to subpoena him. However, the subpoena is just
a sham, b/c the judge wants to testify for his friends, as most character witnesses do.
Commentary 5 on page 592 states that a judge may not testify voluntarily as a
character witness b/c to do so may lend the prestige of the judicial office in support of the party
for whom the judge testifies. Plus, when a judge testifies as a witness, a lawyer who regularly
appears before the judge may be placed in the awkward position of cross-examining the judge. A
judge may, however, testify when properly summoned.
C. Problem 40
The Active Federal Judge
(Page 651)

Facts
The way judge manages his court room is interjecting himself into trials by always questioning
witness himself and being sexist towards female lawyers. Also, his extra judicial activities include
running a homeless shelter and teaching at law school where he plans to ask professors to write
drafts of opinions in cases ha has before him. Also, caught drunk driving with stripper in the car
coming home from Athletic Club that accepts white members only.

A. A Judge's Performance of Official Duties


1. How much should Judge Richardson insert himself into a trial? Canon 3B(3)
requires a judge to require order and decorum in proceedings before the judge. Subsection (4)
states that a judge shall be patient, dignified, and courteous to litigants, jurors, witnesses,
lawyers, and others w/whom the judge deals in an official capacity, and shall require similar
conduct of lawyers, and of staff, court officials and others subject to the judge's direction and
control.
2. Are Judge Richardson's sexist remarks a particular cause for concern? Yes.
Canon 3B(5) states that a judge shall not perform judicial duties w/o bias or prejudice. A judge
shall not, in the performance of his judicial duties, by words or conduct manifest bias or prejudice.
Commentary 2 states that this includes facial expression and body language. A judge must be
alert to avoid behavior that may perceived (by jurors, the media, and others) as prejudicial.
Canon 3C(2) mandates a judge to require staff, court officials, and others subject
to the judge's direction and control to observe the standards of fidelity and diligence that apply tot
he judge and to refrain from manifesting bias or prejudice in the performance of their official
duties.

B. A Judge's Participation in Non-Judicial Activities


1. Is it proper for Judge Richardson to be an officer of a civic organization that
serves the homeless in his community? Canon 4C(3) states that a judge may serve as an officer,
director, trustee, or non-legal advisor of an organization or governmental agency devoted to the
improvement of the law, or of an educational, religious, charitable, fraternal or civic organization
not conducted for profit.
However, under subsection (a) a judge shall not serve as an officer, director,
trustee, or non-legal advisor it if is likely that the organization will be engaged in proceedings that
would ordinarily come before the judge. Note, however, that the rule does not preclude the judge
from being a member of such an organization.
Here, as long as the organization does not frequently come before the Judge, it
will be ok that he is an officer.
2. May Judge Richardson seek to raise money to help the homeless? May he
testify before a public body about the need for more public funds to provide homeless shelters
around the state? Canon 4C(1) precludes a judge from appearing at a public hearing before, or
otherwise consult w/, an executive or legislative body or official except on matters concerning the
law, or except when acting pro se in a matter involving the judge or the judge's interests.
With regard to raising money, 4C(3)(b) states that a judge may assist in planning
a fundraiser, but shall not personally solicit funds unless from other judges over whom the judge
does not exercise supervisory or appellate authority. The judge can make recommendations to
public and private fund-granting organizations on projects and programs concerning the law, but
shall not personally participate in membership solicitation if it might reasonably be perceived as
coercive. A judge also shall not sue or permit the use of the prestige of judicial office for
fundraising or membership solicitation (i.e. cannot be the guest of honor at a fundraising event).
3. May Judge Richardson work as a part-time law professor? Yes. Canon 4B
allows a judge to speak, write, lecture, teach and participate in other extra-judicial activities
concerning the law. However, this is subject to Canon 4G, which states that a judge shall not
practice law.
4. W/regard to Judge Richardson's plan to ask outside experts to write drafts of his
opinions: Canon 3B(7)(b) allows a judge to obtain the advice of a disinterested expert on the law
applicable to a proceeding before a judge if the judge gives notice to the parties of the person
consulted and the substance of the advice, and affords the parties reasonable opportunity to
respond. Commentary 4 states that an appropriate and often desirable procedure for a court to
obtain the advise of a disinterested expert on legal issues is to invite the expert to file a brief
amicus curiae.
What about opinions written by the judge's clerk? Canon 3B(7)(c) allows a judge
to consult w/court personnel whose function is to aid the judge in carrying out the judge's
adjudicative responsibilities or w/other judges. A judge's clerk is certainly court personnel, and
therefore it is ok for them to write opinions for the judge.

C. A Judge's Obligation to Model Proper Behavior


1. Is it proper for Judge Richardson to belong to a private club that only admits
white males to membership? 2C prohibits a judge from holding membership in any organization
that practices invidious discrimination on the bases of race, sex, religion, or national origin.
Discrimination is invidious if it is arbitrary against a certain group. The U.S. Senate has passed a
non-binding resolution declaring that is is inappropriate for nominees to the federal bench to
belong to such clubs.
2. Should Judge Richardson's arrest for drunk driving and the fact that a young
dancer was in the car at the time be considered bases for judicial discipline? Canon 1 (which is
difficult to enforce b/c it is aspirational) states that a judge should participate in establishing,
maintaining, and enforcing high standards of conduct, and shall personally observe those
standards so that the integrity and independence of the judiciary will be preserved.
However, Canon 2A states that a judge shall respect and comply with the law
and shall act at all times in a manner that promotes public confidence in the integrity and
impartiality of the judiciary. Commentary 2 states that actual improprieties under this standard
include violations of law, court rules or other specific provisions of this Code. The test for
appearance of impropriety is whether the conduct would create in reasonable minds a perception
that the judge's ability to carry out judicial responsibilities w/integrity, impartiality and competence
is impaired.
Here, the fact that the judge broke the law is significant and is not merely a part
of his private life. The stripper thing and the fact that he is married probably is private.

1.18 (into)
1.2 (a)(b)(c)
1.4
3.1

Cannell-client
Broker churned account- throw book at him, get liscence suspenced, tie up bank account

Limit scope of representation=

Engagement letter
Accept settlement on the spot

Vous aimerez peut-être aussi